You are on page 1of 103

1000+ Error Detection for All Exams

Last 20 years (Previous Year Papers)


English By Vishal Parihar

Direction: The following question carries

1000+
one statement with one blank. Choose the
most appropriate word from the options
to fill the blank.

Q1
As a result of limited supply, the prices of
spices __________________.
A Bludgeoned
B Plummeted

ERROR DETECTION
C Fell
D Skyrocketed
E Remained stable

Solution
As supply is limited, prices must increase.
Unacademy Top Banking English Educator
Only option D implies that prices increased.
Hence, option D is the right answer.

VISHAL PARIHAR
Bludgeoned: to attack or overcome by
aggressive argument
Plummeted: to fall suddenly and quickly from
a high level or position
Skyrocketed: (of prices, an amount, etc.) to
increase rapidly to a much higher level than
before
Highlights:
➢ Best Faculty of English Language in 2017
Q2
➢ Best Editorial Analyst on YouTube in 2019The entire class ___________ into laughter on
➢ Super30 Teacher’s Award in 2019 hearing the Joke.
A Fell
➢ Best Content Delivery Award in 2020 B Burst
➢ 7+ years experience in Bank Exams C Wheeled
D
➢ Highest number of selection through Youtube Broke
E Screamed
➢ No.1 The Hindu Editorial session on Youtube
➢ Approx. 300 Million Views on Youtube Solution
“burst into laughter” is the correct usage.
➢ Most trending videos of English & Editorial
Burst: to break open suddenly and violently,
➢ Ex. Top Online Educator at Mahendras, Adda247
usually because there is too much pressure
& Wifistudy inside; to cause this to happen

Use Code: (AVP10) for 10% Bankers Way 1


India’s No.1 Teacher in Bank Exams for English Language and Editorial
additional discount on All Website: www.vishalparihar.com | Follow on Instagram: @vishalthetrainer
Unacademy courses YouTube Channel
1000+ Error Detection for All Exams
Last 20 years (Previous Year Papers)
English By Vishal Parihar

1. Another vital reform is to create/(A) a thriving


debt market to let long-gestation infrastructure/(B) 7. The most elegant (a)/ distinction I have been
projects be funded through bonds /(C) and prevent encountered (b)/ is the one drawn (c)/ by the late
the future buildup of bad loans. (D) Harvard jurist and philosopher Ronald Dworkin.(d)/
A.B and D B.Only A No Error (e)
C.Only D D.C and D A.a B.b C.c
E.all correct D.d E.no error

2. Bank boards must be fully professionalized, /(A) 8. The trial protocol for Bharat Biotech candidate
empowered with taking all/(B) bank-related a)/shows that phase I studies were to be conducted
decisions and hold/(C) accountable for their b)/through administration, with monitoring of
performance. /(D) adverse events c)/happening after two hours of the
A.Only C B.Only B first dose and then seven days d)/
C.A and C D.Only D A.only d B.both c and d
E.all correct C.both a and b D.both a and c E.no error

3. Regulatory oversight should see/(A) banks not just 9. He then returned to the hospital to get to work
separate,/(B) but and in relation with one another a)/, expecting to be busy stitching up patients b)/
/(C)and non-banking financial companies (NBFCs). injured in the blast and saving lives but c)/he
/(D) discovered that the hospital, too, was in rubble d)/
A.Only C B.Only B C.B A.both a and b B.only d
and C D.all are wrong E.all correct C.only a D.both b and c E.no
error
4. The Reserve Bank of India’s (RBI) reported
suggestion/(A) to the government to low/(B) the 10. Bayesian reasoning is a principled way to
government’s equity stake in the banks/(C) it owns integrate a)/what you previously thought with what
to 26% make eminent sense. /(D) b)/you have learned and come to a conclusion that
A.A and C B.B and C C.B c)/incorporates them both, give them appropriate
and D D.Only A E.All correct weights d)/
A.both a and b B.both c and d
5. Privatization would help these banks/(A) to C.only c D.only d
overcome restraints placed by State ownership such E.no error
as/(B) low pay, short tenures, political interfere
and/(C) the fear of being pursued by investigation 11. While TikTok isn’t immune to the challenges that
agencies. /(D) all a)/platforms face, the fact that our users come to
A.B and D B.C and D C.B express their b)/creative sides immediately makes
and C D.A and C E.All correct. TikTok a much more uplift c)/ environment than one
might experience on other platform d)/
6. Though Facebook, YouTube and Twitter have A.only b B.both a and c
deployed a)/artificial intelligence software and hired C.both c and d D.both b and d
thousands of b)/content moderators to take in vile E.no error
or misleading c)/posts, dangerous content inevitably
slips through d)/ 12. The risk of a political backlash (a)/ from the
A.only c B.only d middle class cannot rule (b)/ out in case oil prices
C.both a and b D.both a and c keep (c)/ climbing— feeding into general
E.no error inflation.(d)/ No Error (e)
Use Code: (AVP10) for 10% India’s No.1 Teacher in Bank Exams for English Language and Editorial 2
additional discount on All Website: www.vishalparihar.com | Follow on Instagram: @vishalthetrainer
Unacademy courses
1000+ Error Detection for All Exams
Last 20 years (Previous Year Papers)
English By Vishal Parihar

A.a B.b C.c A.Only III B.Both I and III


D.d E.no error C.All, I,II and IV D.Both II and IV
E.No error
13. The downtrend in private investment (a)/ must
recover (b)/ to increase (c)/ value-generating 18. The insolvency regulator has also amended
capacities.(d)/ No Error (e) (A)/the regulation to allow certain creditors, (B)/
A.a B.b C.c who do not want to wait for the liquidation process
D.d E.no error to be over, (C)/ for exit the process by assigning or
transferring the debt due to them, to other creditors
14. Regulatory oversight should see/(A) banks not of the company (D)/.
just separate,/(B) but and in relation with one A.The insolvency regulator has also amended
another /(C)and non-banking financial companies B.the regulation to allow certain creditors,
(NBFCs). /(D) C.who do not want to wait for the liquidation
A.Only C B.Only B C.B process to be over,
and C D.all are wrong E.all correct D.for exit the process by assigning or transferring
the debt due to them, to other creditors of the
In this question, a sentence has been divided into company.
four parts marked as I, II, III and IV. You need to find E.No error
which part/parts does not/do not have an error in
terms of its grammatical or contextual usage. If the 19. The financial creditors can also (A)/ attach a copy
sentence is absolutely correct, mark (E) as your of any court or tribunal’s order (B)/ which has,
answer. through an order, (C)/established the company had
defaulted on debt payments (D)/.
15. I. The democratic barbarism, there every issue is A.The financial creditors can also
II. now thought of through the prism of partisan B.attach a copy of any court or tribunal’s order
combat, C.which has, through an order,
III. not public reason, has now infected assessment D.established the company had defaulted on debt
in the judiciary partly payments
IV. as a result of its own inability to project that it is E.No error
above the fray.
A.Both I and III B.Both II and IV 20. The Insolvency and Bankruptcy Board of India
C.All II, III and IV D.Only IV (IBBI) has amended (A)/the regulations for
E.No error liquidation under the Insolvency and Bankruptcy
Code (IBC) (B)/by which they effectively allowed the
16. I. In seeking our minor policy victories liquidator to assign or transfer a “not readily
II. from the court, we in some senses realisable asset” (C)/ to any person in consultation
III. end up legitimising its major with the stakeholders’ consultation committee (D)/.
IV. infraction on constitutional principles. A.The Insolvency and Bankruptcy Board of India
A.Only II B.Both I and III (IBBI) has amended
C.Only IV D.All, I, II and III B.the regulations for liquidation under the
E.No error Insolvency and Bankruptcy Code (IBC)
C.by which they effectively allowed the liquidator to
17. I. We have gone past the stage assign or transfer a “not readily realisable asset”
II. where the highest court’s infirmities D.to any person in consultation with the
III. can be captured at the policy stakeholders’ consultation committee
IV. wonk-ish language of institutional reform. E.No error
Use Code: (AVP10) for 10% India’s No.1 Teacher in Bank Exams for English Language and Editorial 3
additional discount on All Website: www.vishalparihar.com | Follow on Instagram: @vishalthetrainer
Unacademy courses
1000+ Error Detection for All Exams
Last 20 years (Previous Year Papers)
English By Vishal Parihar

21. During the Diwali period messages attributed to 25. Biden is engaged in a grand relaunch (A)/ of his
deputy chief minister Nitin Patel (A)/ and minister of administration after (B)/ a disaster-prone 11 months
state of health Kumar Kanani began to circulate (C)/ from the election. (D)/No error
suggesting various (B)/ kinds of lockdowns that A.A B.B C.C
could be coming in the state, (C)/ especially in cities D.D E.No error
like Ahmedabad and Surat that have witnessed a
surge in cases (D)/. 26. As the public, governments and (A) / the
A.During the Diwali period messages attributed to investors wake up to (B) / climate change, the clean-
deputy chief minister Nitin Patel energy (C) / industry is gaining momentum. (D)/ No
B.and minister of state of health Kumar Kanani error (E)
began to circulate suggesting various A.A B.B C.C
C.kinds of lockdowns that could be coming in the D.D E.No error
state,
D.especially in cities like Ahmedabad and Surat that 27. Mr Biden’s most daunting immediate (A)/
have witnessed a surge in cases. domestic tasks will surely be dealing (B)/ with covid-
E.No error 19—despite this week’s further (C)/ good news
about vaccines.(D)/ No error
22. Despite the massive overcrowding in old city (A)/ A.A B.B C.C
areas where people thronged markets for Diwali D.D E.No error
shopping a week before the festival, (B)/ on
November 11, the AMC extended the timings of 28. Chhattisgarh Chief Minister Bhupesh Baghel will
popular commercial (C)/ areas from 10 pm till launch two key (A)/ schemes related with
midnight citing ‘ the festival season’ (D)/. distribution of fortified rice and (B)/ setting up of
A.Despite the massive overcrowding in old city Swami Atmanand Government English medium
B.areas where people thronged markets for Diwali schools on (C)/ Sunday during ''Rajyotsav''
shopping a week before the festival, statehood celebrations.(D)/ No error
C.on November 11, the AMC extended the timings A.A B.B C.C
of popular commercial D.D E.No error
D.areas from 10 pm till midnight citing ‘ the festival
season’. 29. In a bid to strengthen capital base, (A)/ the union
E.No error government has provided Rs 670 crore (B)/ to
Regional Rural Banks considering their importance
23. As oil demand dwindles, (A) / they will face a (C)/ in agriculture finance during these difficult
vicious fight for market share (B) / which will be won times. (D)/ No error
by the countries (C) / with the cheapest and cleanest A.A B.B C.C
crude. (D)/ No error (E) D.D E.No error
A.A B.B C.C
D.D E.No error 30. The Niti Aayog has released a draft model Act
(A)/ and rules for states on conclusive land title with
24. More than Most leaders, (A)/ Boris Johnson (B)/ an aim to reduce litigations and ease (C)/ the
relies upon the people around him, (B)/ for his land acquisition process for infrastructure projects.
positive qualities (C)/ are counterbalanced by (D)/ No error
negative ones (D)/ No error A.A B.B C.C
A.A B.B C.C D.D E.No error
D.D E.No error
Use Code: (AVP10) for 10% India’s No.1 Teacher in Bank Exams for English Language and Editorial 4
additional discount on All Website: www.vishalparihar.com | Follow on Instagram: @vishalthetrainer
Unacademy courses
1000+ Error Detection for All Exams
Last 20 years (Previous Year Papers)
English By Vishal Parihar

31. I. The Brit have amassed 92 pole positions and everyone wants (C)/ “every child in school”
has 163 podium finishes to his name, eventually (D).
II. eight more than Schumacher, both him and A.All in India,
Hamilton won their seventh title at the B.not only children, teachers or parents, but family
III. age of 35 and if Hamilton again signed with members
Mercedes for the new season, chances are he may C.and communities, everyone wants
IV. very well overtake the German’s championship D.“every child in school” eventually
title count. E.No errors
A.I and II B.III and IV C.II and IV D.I
and III E.No errors 36. Prime Minister Narendra Modi Saturday chaired
a meeting (A)/ for review the ongoing Covid-19
32. I. Since most cars pitted multiple times in order pandemic situation in the country (B)/ and
II. to counter the wet conditions in Turkey on suggested developing a vaccine delivery system on
Sunday, the lines of conduct of polls (C)/ and disaster
III. Hamilton was one of only two drivers who management while involving all levels of
IV. ran there races on single-stop strategies and government and citizen groups (D).
intermediate tyres. A.Prime Minister Narendra Modi Saturday chaired a
A.I and II B.II and III C.I and IV meeting
D.III and IV E.No errors B.for review the ongoing Covid-19 pandemic
situation in the country
33. I. From being a pedal-to-the-metal driver who C.and suggested developing a vaccine delivery
went through six years of system on the lines of conduct of polls
II. not winning a title during Red Bull’s era in D.and disaster management while involving all
dominance, levels of government and citizen groups
III. to achieving the level of mastery he has in E.No errors
managing his Mercedes’ tyres –
IV. Hamilton recognised the direction the sport was 37. Many considers it wrong (a)/ to blight youngsters
taking and changed their style accordingly. by recruiting (b)/ them into armed (c)/ forces at a
A.I and III B.II and IV C.All, I, II and III young age.(d)/
D.All, II, III and IV E.No errors A.a B.b C.c D.d
E.no error
34. Available data since years (A)/ of the ASER
(Annual Status of Education Report) indicate that 38. The circumstances that went into the making/(A)
(B)/ at least in rural areas, for many children,(C)/ of this incident and the response of the
basic skills of reading and arithmetic are worryingly administration/(B) all point to the same morbid
low (D). affliction which/(C) can be fatal to any democratic
A.Available data since years society. /(D) No error/ (E)
B.of the ASER (Annual Status of Education Report) A.C B.D C.Both A and C D.B
indicate that E.E
C.at least in rural areas, for many children,
D.basic skills of reading and arithmetic are 39. On May 20, the eastern part of India/(A) was
worryingly low battered by Amphan, and/(B) Many a people of
E.No errors West Bengal /(C) have been highly affected by the
devastating cyclone. /(D) No error/ ( E)
35. All in India,(A)/ not only children, teachers or A.A B.B C.C
parents, but family members (B)/ and communities, D.Both C and D E.E
Use Code: (AVP10) for 10% India’s No.1 Teacher in Bank Exams for English Language and Editorial 5
additional discount on All Website: www.vishalparihar.com | Follow on Instagram: @vishalthetrainer
Unacademy courses
1000+ Error Detection for All Exams
Last 20 years (Previous Year Papers)
English By Vishal Parihar

46. However the reader who(a) /understands the


40. The number of people/(A) of our country are broad(b) / implications of the content and (c) /
under the thumbs/(B) of danger because of ignorant allusion finds greater pleasure. (d) /
attitude/(C) towards the concept of social A.a B.b C.c D.d
distancing. /(D) No error/(E) E.no error
A.Both A and C B.B C.D D.C
E.E 47. Instead, these educationists (a) / should suggest
lowering of (b) / burden of homework for the(c) /
41. In India, it is the normal probusiness upper commission for educational reforms. (d) /
class/(A) that wants to keep stringent/(B) A.a B.b C.c D.d
containment measures in place for/(C) as long as it E.no error
takes to control the virus. /(D) No error/(E)
A.A B.B C.D D.C 48. If a country does not learn to (a) / master these
E.E new realities of life, all (b) / our aspirations to ensure
the prosperity(c) / of our peoples may come to
42. How can it be true when (a)/ we know a lot of naught. (d) /
rich people (b)/ complaining about their (c)/ A.a B.b C.c D.d
unhappy states of mind.(d)/ E.no error
A.a B.b C.c D.d
E. no error 49. The measures include yoga (a) / classes with
emphasis (b) / on physical and mental exercises (c) /
43. In order to test them he (a)/ took them to a field and also change in food habits. (d) /
(b)/ and gave a piece of land (c)/ to each one of A.a B.b C.c D.d
them. (d)/ E.no error
A.a B.b C.c D.d
E.no error 50. Along with contemporary and global issues, (A)/
the Prime Minister said the University is also
44. In order to test them he (a)/ took them to a field expecting (B)/ to encourage local culture, local art
(b)/ and gave a piece of land (c)/ to each one of and promote (C)/ its tradition on research on social
them. (d)/ issues (D).
A.a B.b C.c D.d A.Along with contemporary and global issues
E.no error B.the Prime Minister said the University is also
expecting
45. The decision by the Paris-based watchdog (A)/, C.to encourage local culture, local art and promote
the Financial Action Task Force, (B)/ last week to D.its tradition on research on social issues
retain Pakistan on its greylist (C)/ has clearly E.No errors
disappointing the Imran Khan government (D).
A.The decision by the Paris-based watchdog DIRECTIONS (51-100) In the following questions a
B.the Financial Action Task Force, sentence is provided which is divided in different
C.last week to retain Pakistan on its greylist parts. One of these parts may or may not be
D.has clearly disappointing the Imran Khan grammatically or contextually correct. Choose the
government Option corresponding to the error as part of your
E.No errors answer.

51.(A)The efforts of the government will/(B) not


only serve the purpose of
Use Code: (AVP10) for 10% India’s No.1 Teacher in Bank Exams for English Language and Editorial 6
additional discount on All Website: www.vishalparihar.com | Follow on Instagram: @vishalthetrainer
Unacademy courses
1000+ Error Detection for All Exams
Last 20 years (Previous Year Papers)
English By Vishal Parihar

successfully/(C)conducting Kumbh Mela but will 60(A)Across India, a devastating/(B) drought


contribute/(D) in the overall development of the threatens/(C) to spell boom for the/(D)already
Prayagraj and its people/(E)No error. distressed hinterlands/(E)No error.

52.(A)‘’According to estimates, apparel is produced 61.(A)The 'One Nation, One Poll' plan is/(B) laden of
by/(B) about 77,000 small-scale units./(C) One of logistical challenges/(C) and could have
biggest challenges is the threat/(D)of imports from undemocratic consequences,/(D)But it could be very
neighbouring countries’’ Rahul Mehta said./(E)No good for the ruling party/(E)No error.
error.
62.(A)Analysts also argue that if/(B) these assembly
53.(A)Not only this, the farsighted approach/(B) of polls are held/(C) ahead of the Lok Sabha polls, the
the Chief Minister Yogi Adityanath has enabled BJP's prospects are more/(D) likely to dim by the
an/(C) overall development of Prayagraj which will time the national polls roll out. /(E)No error
not only render world class facilities to the visitors at
Kumbh/(D)but will also serve the local residents and 63.(A)India spends little/(B) on R&D, making us
upcoming tourists of the city. /(E)No error. lag/(C) in innovative products in/(D) export
markets/(E)No error
54.(A)Various new initiatives were/(B) launched and
these never-seen-before/(C) measures are fuelling 64.(A)Private sector Universal Sompo Health
rapid development, growth and/(D) prosperity Insurance/(B) offers one of the cheapest health
around the State of Telangana. /(E)No error. covers of Rs 5 lakh /(C) for a family of five for Rs
12,800 per annum. This includes/(D) pre- and
55.(A)With India soon becoming/(B) the top three posthospitalisation and critical illnesses, among
automotive manufacturing/(C) countries in the other benefit./(E)No error.
world, it is high/(D) time this sport be given its due.
/(E)No error. 65.(A)The BJP's stand on banning cow slaughter/(B)
had led to huge public outrage in(C) the North
56.(A)Air India disinvestment, a crucial/(B) promise eastern state where eating beef/(D) is intrinsic to
made by the Modi government,/(C) is also symbolic the region food habits./(E)No error
of their commitment to deliver on its promise of/(D)
minimum government and maximum 66.(A)In Nagaland, the BJP's 15-year-long/(B)
governance/(E) No error alliance of Naga People's Front/(C) snapped, with
57.(A)Alternative schemes needed/(B) for export both parties failing to/(D) agree on seat distribution.
promotion/(C) to counter the US challenge(D) on /(E)No error
existing ones/(E)No error
67(A)India's unemployment rate(B) had risen to 6.1
58.(A)According to reports, India/(B) witness about per cent /(C)in 2017-18, the highest/(D) in 45 years.
10 million weddings/(C) every year and the wedding /(E)No error.
industry here is/(D) estimated to be worth more
than $50 billion./(E) No error. 68.(A)In Maharashtra, the Shiv Sena sees space for
the party/(B) in the backdrop of growing dissent
59.(A)A number of India's export/(B) promotion against the BJP, /(C) specially after Maharashtra
schemes has/(C) been challenged/(D) by the chief minister Devendra Fadnavis's bungling/(D) of
US/(E)No error loan waivers to farmers and handling the anger
among the Dalits. /(E)No error

Use Code: (AVP10) for 10% India’s No.1 Teacher in Bank Exams for English Language and Editorial 7
additional discount on All Website: www.vishalparihar.com | Follow on Instagram: @vishalthetrainer
Unacademy courses
1000+ Error Detection for All Exams
Last 20 years (Previous Year Papers)
English By Vishal Parihar

69.(A)NPAs of banks and HFCs in/(B) home loan is


comparatively/(C) low except in housing loans up 76) A study conducted by the World Health
to/(D) Rs. 2 lakhs which of course is a/(E) small Organisation country office for (a)/ India in
fraction value-wise. /(E)No error collaboration with the Johns Hopkins Bloomberg
School of Public Health across (b)/ seven States in
70.(A)As the nation tries to make/(B) sense of the India show that banning gutka, (c)/ a form of
meagre information/(C) in the ground, the chewing tobacco, helped users kick the habit. (d)/
ambitious health protection plan/(D) provides more No error. (e)
questions than answers./(E)No error
77) Matchboxes, little auxiliaries to our lives that
71.(A)Maharashtra pollution control board/(B) has have endlessly (a)/ fascinated collectors and
issue a circular for/(C) randomized risk based chroniclers of pop art, have (b)/ fast replaced almost
inspection and submission /(D) of inspection reports entirely by (c)/ gas lighters, electric stoves and
within 24 hrs as mandated by DIPP. cigarette lighters. (d)/ No error. (e)

72.(A)States may choose an intermediary/(B) such 78) The rally, launched as part of social outreach
as an insurance company, based /(C)on the premise programme (a)/ to connect children and youth along
that the government pays the insurance/(D) both coasts of the country, (b)/ is initiative of the
company premium on behalf for the beneficiary./(E) Southern Naval Command organised as part (c)/ of
No error. celebrations to mark the platinum jubilee of INS
Valsura. (d)/ No error. (e)
73.(A)Auto sales has slumped/(B) to a near
twodecade low, across/(C) all categories. The 79) In a move that would benefit thousands of
abysmal figures for /(D)July only confirm this trend property buyers, (a)/ the State Registration
./(E) No error. Department have (b)/ proposed to reduce the
guideline value (c)/ for registration of sale deed of
74.(A)In Christian-majority Meghalaya, the National properties in Tamil Nadu. (d)/ No error. (e)
People's Party,/(B) which is part of the NDA
government at the Centre and in Manipur, 80) Speak to reporters after the announcement,
declined/(C) to enter into a pre-poll alliance with the Raju Murugan, (a)/ director of Joker, said that he
BJP, in /(D)anticipation of a negative public hoped (b)/ the national award would open (c)/ up
mood./(E)No error space for films with political content in future. (d)/
No error. (e)
75.(A)In the long run, the gains on holding/(B)
elections simultaneously will outweigh/(C) the 81) Another senior leader said that the idea was
negatives, particularly from the viewpoint of/(D) the being strongly (a)/ pushed by several senior leaders,
political parties, funds and the manpower (b)/ since they think it would give the party (c)/ the
require./(E)No error much-need visibility. (d)/ No error. (e)

82) Citing the question asked in the SSLC social


Read the following sentences carefully and select science exam (a)/ on who is the chairman of the
the correct option. Each sentence is split into four Planning Commission, he said this (b)/ government
parts namely (a), (b), (c) and (d). Select the part does not even know (c)/ that the Commission was
which is grammatically wrong. If all the parts of the replaced on Niti Ayog in 2015. (d)/ No error. (e)
sentence are grammatically correct then, choose
‘No error’ (e) as the option.
Use Code: (AVP10) for 10% India’s No.1 Teacher in Bank Exams for English Language and Editorial 8
additional discount on All Website: www.vishalparihar.com | Follow on Instagram: @vishalthetrainer
Unacademy courses
1000+ Error Detection for All Exams
Last 20 years (Previous Year Papers)
English By Vishal Parihar

83) In 2017, the State was witness to a series of 90) Elliot hopes the exhibition—which will be on
protests (a)/ by students, members of Tamil show (a)/ for over a year—will eventual help reveal
organisations and others who (b)/ demand that (b)/ the stories of the creators and (c)/ spark
jallikattu, the traditional adventure (c)/ sport of discussion and debate on these communities. (d)/
taming the bull, be permitted unconditionally. (d)/ No error. (e)
No error. (e)
91) Eventually, through its characters, the film
84) Tuesday began on a celebratory note but ended teases out the many (a)/ nuances of arranged
with (a)/ sad for child rights activist and Nobel Peace marriage: the contradictions (b)/ between the
(b)/ prize winner, Kailash Satyarthi, who came to his growing education, employment to financial
ancestral (c)/ home here for the first time after opportunities for women, (c)/ and the inescapable
receiving the coveted prize. (d)/ No error. (e) pressures of “settling down” into matrimony. (d)/
No error. (e)
85) These receipts will not only certainly contain the
names of those (a)/ attacked but also their 92) Initially conceptualised as a year-long project,
addresses, in which (b)/ case it is for the corporation (a)/ the documentary eventually took almost four
authorities to explain (c)/ as to why they sold cattle year to film, (b)/ and six to be completed, resulting
to out of State people,” the team said. (d)/ No error. in 90 sharp (c)/ minutes cut from 750 hours of
(e) footage. (d)/ No error. (e)

86) The present method not only detects pregnancy 93) Chief Justice of India Jagdish Singh Khehar on
(a)/ at an early stage, but also makes it possible (b)/ Saturday said (a)/ political parties should be held
for breeders to independent carry (c)/ out the (b)/ accountable for the routinely unfulfilled (c)/
process of detection. (d)/ No error. (e) promises made in its election manifestos. (d)/ No
error. (e)
87) The research states that “people working for
bosses (a)/ who display psychopathic and 94) Though depression affects all demographic
narcissistic tendencies” not only (b)/ feel depressed groups, (a)/ it is more common within adolescents
because of constant bullying but are (c)/ also likely and (b)/ young adults, women of child bearing (c)/
to engage in counterproductive behaviour. (d)/ No age (particularly following childbirth), and adults
error. (e) over the age of 60. (d)/ No error. (e)

88) The current flag-bearer of women’s cricket in 95) An increase in educational enrolment among the
India, Mithali Raj, (a)/ recalls her 214, the then (a)/ younger cohort, attainment of socio-economic
highest score in women’s Test history, (b)/ against status, (b)/ and household composition large (c)/
England at Taunton in 2002: (c)/ Those days, the set- contributed to the drop. (d)/ No error. (e)
up was very amateurish, and there were hardly any
money involved. (d)/ No error. (e) 96) In his sharpest comments yet, the Dalai Lama
compared (a)/ China’s actions in Tibet to the regime
89) Because the pieces emerged from the museum’s of Cambodian dictator Pol Pot, (b)/ who was held
archives, (a)/ often with scant information about the responsible to the death (c)/ of more than a million
creators, (b)/ the exhibition has had to rely much in Cambodians in the 1970s. (d)/ No error. (e)
the voices of (c)/ those who brought the pieces to
Britain. (d)/ No error. (e) 97) Despite that, some commentators say that the
U.S.-China (a)/ agreement has shift the focus to (b)/

Use Code: (AVP10) for 10% India’s No.1 Teacher in Bank Exams for English Language and Editorial 9
additional discount on All Website: www.vishalparihar.com | Follow on Instagram: @vishalthetrainer
Unacademy courses
1000+ Error Detection for All Exams
Last 20 years (Previous Year Papers)
English By Vishal Parihar

developing countries, especially India, (c)/ which is scientist on Copenhagen (c)/ when actually he is a
the fourth largest emitter. (d)/ No error. (e) dentist from Karaikudi. (d)/ No error. (e)

98) The exfoliation was carried at room temperature 106) The desperate reliance on deep ground water
(a)/ but under vacuum and the electrical and farming is (a)/ symptomatic of the traumatic
conductance was (b)/ measured at the time of shift experienced (b)/ by the Banjaras since the
exfoliation before (c)/ the pristine nature of the British government criminalised them, (c)/ like the
edge was affected. (d)/ No error. (e) Mahua tree itself, in this part of the country. (d)/ No
error. (e)
99) There is a simple bench on the Indian side of the
border, (a)/ where bags are inspected, and a (b)/ 107) Towards the centre of the pole, Hart
covered spot where the women (c)/ are frisk and acknowledges one of the most disturbing (a)/ aspect
question. (d)/ No error. (e) of Canadian history, the system of residential (b)/
schools where children were forcibly sent,
100) Alongside such contemporary pieces, also on separated from their (c)/ families, and where they
display is (a)/ over a 100 historic pieces ranging from suffered abuse, disease and death. (d)/ No error. (e)
paintings (b)/ to photographs, metal and wooden
artefacts (c)/ found in the museum’s archives. (d)/ 108) The overall BSE Small Cap Index valuation,
No error. (e) though, (a)/ is still not very expensive compared to
historical averages – an indication (b)/ that earnings
101) “Focusing on minority communities was a (a)/ of a good number of companies (c)/ has kept pace
way to communicate other versions of India, but (b)/ with the rally in their stock price. (d)/ No error. (e)
we know we have (c)/ just scratch the surface,” says
Elliot. (d)/ No error. (e) 109) If you are spending, say, Rs.1 lakh per month
102) However, most are in the notified Mysore and is (a)/ due to retire in five years, you may be
Elephant Reserve (MER), (a)/ made up of forests wondering as to whether (b)/ you would be able to
from Bhadra in Chikkamagaluru to (b)/ Nagarahole- generate the same level (c)/ of income during your
Bandipur BRT Wildlife Sanctuary belt, besides retirement to sustain your lifestyle. (d)/ No error. (e)
Bannerghatta, (c)/ spread over an area of near
11,000 sq. km. (d)/ No error. (e) 110) You choose your country, you choose your
spouse, (a)/ you choose your profession, you choose
103) The State government had approached the (b)/ your political masters, and you choose (c)/
Centre for (a)/ waiver of farm loans but even if we where you want live and how. (d)/ No error. (e)
fail to get any support from (b)/ the Centre, we will
ensure that farmers are (c)/ freed in their mounting 111) The Times Higher Education BRICS and
debts, said Capt. (d)/ No error. (e) Emerging Economies Rankings 2015, which gives
(a)/ new insights into the performance and
104) On buying the watch, customers will also get a contribution of universities (b)/ in BRICS and
(a)/ log book with all the flight details of its journey emerging economies, demonstrates a stronger and
(b)/ around the world, a great story to tell (c)/ sharp attention (c)/ to issues of quality and
people who compliment them on their watch. (d)/ excellence to be paid by India. (d)/ No error. (e)
No error. (e)
112) The government has initiated the process to
105) As soon as I entered the house I spotted her (a)/ decriminalise attempt to suicide, a move that
father wearing a ‘Harvard’ sweatshirt in (a)/ the 35 (b)/ will ensure that people who are drive to (c)/ kill
degrees temperature as if he (b)/ is some nuclear
Use Code: (AVP10) for 10% India’s No.1 Teacher in Bank Exams for English Language and Editorial 10
additional discount on All Website: www.vishalparihar.com | Follow on Instagram: @vishalthetrainer
Unacademy courses
1000+ Error Detection for All Exams
Last 20 years (Previous Year Papers)
English By Vishal Parihar

themselves do not end up in jail if they don’t 120) Top-ranked South Africa plays its first Test in
succeed. (d)/ No error. (e) five (a)/months when it faces unfamiliar West Indies
(b)/ in the opening contest of its (c)/three-match
113) Public health experts have cautioned that at a series in Pretoria on Wednesday. (d)/ No error. (e)
time when (a)/ there are seasonal short showers,
residents (b)/ have to careful about preventing the 121) Following the hostage related incident in
breeding of (c)/ mosquitoes in their homes or Sydney on Monday, (a)/ the Board of Control for
compounds. (d)/ No error. (e) Cricket in India (BCCI) has decided to (b)/send its
chief of anticorruption and security (c)/ Ravi Savani
114) A couple of years ago, the epidemic that swept to Australia to overseen the security arrangements
through (a)/ Tamil Nadu was fuelled by a set of for the Indian team. (d)/ No error. (e)
circumstances that seemed unique: power cuts (b)/
had forced residents in rural areas to store (c)/ water 122) FGB, UAE’s largest bank by net profit, plan to
in large pots and cisterns at weeks. (d)/ No error. (e) expand its (a)/operations in India in line with its
global expansion (b)/strategy to support clients with
115) The sentence emerged after set of 19 cross border business (c)/and focusing on key trade
pictographs from (a)/ a cave in Hampi were and finance flows. (d)/ No error. (e)
deciphered using (b)/ root morphemes of Gondi 123) BlackBerry, which had to struggle against (a)/
language, considered (c)/ by many eminent linguists aggressive competition over a year ago, is now (b)/
as a proto Dravidian language. (d)/ No error. (e) refocusing itself by (c)/ targeting the professional
consumer. (d)/ No error. (e)
116) A film by a Pune based director on the daily
struggles of four sisters (a)/ across the backdrop of 124) The dearth of consumer friendly apps on a (a)/
farmer suicides in (b)/ Vidarbha in Maharashtra has BB device has been resolved with the (b)/
made it to the long list in (c)/ the “best picture” integration of Amazon App Store on the (c)/ recent
category of the 87th Academy Awards, popularly launched Passport running the 10.3 update. (d)/ No
called the Oscars. (d)/ No error. (e) error. (e)

117) A specialist of the “agricultural documentary”, 125) As sports management companies perform a
Ms. Bhosale has (a)/ created and directed close to raft (a)/ of functions, including public relations,
50 films – all focus (b)/ on the pitfalls and triumphs finance, sponsorships, training, laws (b)/ and ethics,
of (c)/ men and women who work the soil. (d)/ No it needs (c)/ professionals qualified in these areas.
error. (e) (d)/ No error. (e)

118) In 1994 the Supreme Court not only 126) It’s been over a month since several students
decriminalised (a)/ the attempt to suicide but also (a)/ of G.B. Pant Engineering College on south
(b)/ observed that the ‘right to life’ (c)/ include the Delhi’s Okhla (b)/ have been spending the night (c)/
‘right to die.’ (d)/ No error. (e) in their classrooms as a mark of protest. (d)/ No
error. (e)
119) With digital media emerge as new and one of
the most (a)/ powerful election battlegrounds in the 127) A street food vendor was murdered on Sunday
Lok Sabha 2014 (b)/ elections, the term ‘Election night (a)/ in Outer Delhi’s Mangolpuri after he told
2014’ topped (c)/ Google India’s trending list for the (b)/ a group of drunken men that he would not be
year. (d)/ No error. (e) able (c)/ to serve them Gol gappas. (d)/ No error. (e)

Use Code: (AVP10) for 10% India’s No.1 Teacher in Bank Exams for English Language and Editorial 11
additional discount on All Website: www.vishalparihar.com | Follow on Instagram: @vishalthetrainer
Unacademy courses
1000+ Error Detection for All Exams
Last 20 years (Previous Year Papers)
English By Vishal Parihar

128) Following reports Noida Authority staff fact, Dr. Sharma himself has been controlling the
allegedly (a)/ not reaching office on time, UP Chief affairs of the Akademi. (d)/ No error. (e)”
Minister Yogi Adityanath (b)/ on Monday sought the
employee details of (c)/ both Noida and Greater 136) The IOP Party on Monday released (a)/ its
Noida Authorities. (d)/ No error. (e) manifesto for the civic polls, in which it has promised
(b)/ constitution of a municipal development board
129) A delegation led by the Deputy Chief Minister (c)/ and regularisation of contractual employees.
Mr. Manish Sisodia (a)/ also met with Delhi State (d)/ No error. (e)
Election Commissioner and (b)/ gave him a
memorandum, requesting an inquiry (c)/ with the 137) Describing his photo as “degrading and (a)/ a
EVMs being brought from Rajasthan. (d)/ No error. move to defame him”, Mr. Gupta has demanded (b)/
(e) that the Commission take immediate (c)/ legal
action against the BPS. (d)/ No error. (e)
130) A total of 2,537 candidates will contest
elections (a)/ to the 272 wards in the three 138) Delhi Congress president Ajay Maken said at an
municipal corporations (b)/ of the Capital on April event (a)/ to inaugurate candidates’ election offices
23, (c)/ said the Delhi State Election Commission. on Monday (b)/ that the city had seen
(d)/ No error. (e) “unprecedented” development when (c)/ the party
was in power in the Capital since 15 years. (d)/ No
131) Having been robbed cash and valuables (a)/ error. (e)
worth ₹4.10 lakh in Karol Bagh last week, a
Tanzanian family (b)/ which had come to India on a 139) Over this approval, he claimed, only 50% funds
medical tour (c)/ are struggling to survive in the (a)/ were to be given by the Delhi government, but
Capital. (d)/ No error. (e) (b)/ “due to negligence” of the Kejriwal government,
(c)/ the expansion was delayed. (d)/ No error. (e)
132) The family approached the police and
registered a complaint, (a)/ even as the Korean 140) In an affidavit submitted to the State Election
embassy got in touch (b)/ on the victim’s family and Commission (a)/ along with her nomination, Ms.
asked the police (c)/ to expedite the investigation. Kaushik disclosed that (b)/ she owned two buildings
(d)/ No error. (e) measuring 720 square feet and 553 square feet, (c)/
which are priced at ₹50 lakh and ₹32 lakh,
133) Mr. Gopal in his post said the “liberty to lead respective. (d)/ No error. (e)
(a)/ a peaceful life has snatched from us” and (b)/
that this was “harassment of the highest order” (c)/ 141) Liberalising the end use of airport land (a)/ and
and could not be taken lightly. (d)/ No error. (e) unlock its potential is undoubtedly (b)/ to advance
public interest and (c)/ for better management of
134) Sanjay and Ajay were arrested for allegedly the airport. (d)/ No error. (e)
failing (a)/ to complete the Gurugram project in time
and not having (b)/ valid permissions from 142) While the policy extended the benefits to
competent authorities (c)/ before start the project, existing (a)/ and future airports of AAI, as well as
the police said. (d)/ No error. (e) future airport (b)/ projects under Public Private
Partnership (PPP), (c)/ it exclude the existing
135) The Bench said on Monday, “Though it is airports. (d)/ No error. (e)
pleaded (a)/ by the government that an
Administrator has been appointed, (b)/ it appear 143) It’s been over a month since several student
that the same is nothing but an eye-wash and, (c)/ in (a)/ of G.B. Pant Engineering College in south Delhi’s
Use Code: (AVP10) for 10% India’s No.1 Teacher in Bank Exams for English Language and Editorial 12
additional discount on All Website: www.vishalparihar.com | Follow on Instagram: @vishalthetrainer
Unacademy courses
1000+ Error Detection for All Exams
Last 20 years (Previous Year Papers)
English By Vishal Parihar

Okhla (b)/ have been spending the night (c)/ in their 151) “During the visit to U.P, the team was surprised
classrooms as a mark of protest. (d)/ No error. (e) (a)/ to know that all the addresses gave by Jalim
Singh to his company were fake,” (b)/ said the
144) According to the RTI documents, (a)/ instead of officer, adding that Rajeev had forged the
building the college, a part of the land (b)/ had given documents and (c)/ used the identity of Jalim Singh
to an institute — Indraprastha Institute of just to commit the robbery. (d)/ No error. (e)
Information Technology (IIIT) — that (c)/ will come
up on the public-private partnership model. (d)/ No 152) The victim, Ezaz Ahmed, who work for UNI, also
error. (e) accused (a)/ the police of being indifferent to his
ordeal (b)/ and sluggish in their approach when he
145) Students, meanwhile, complained that (a)/ the first made (c)/ the call to the Police Control Room.
classrooms in the college were not suitable (b)/ for (d)/ No error. (e)
attending lectures since (c)/ they were hostel rooms
and lack space. (d)/ No error. (e) 153) The driver and conductor admitted to (a)/
having “misbehaved”, but said that the argument
146) The Supreme Court was on Monday informed was (b)/ over the driver digress (c)/ from the
(a)/ that the process of arriving at a settlement designated route. (d)/ No error. (e)
between (b)/ Union minister Rajyavardhan Singh
Rathore and Parsvnath Developers over (c)/ 154) A police team rushed all of the injured to (a)/
possession of a flat in Gurgaon was going. (d)/ No Kukreja hospital where Ram Singh was declared
error. (e) brought dead (b)/ while Rahul Singh, Dilip and
Anand Prasad were (c)/ discharged after treatment.
147) In a statement give to the police, Mr. Yadav said (d)/ No error. (e)
that (a)/ they were all sleeping inside the house
when the theft took place and (b)/ were informed of 155) “The registration number of the car was not
the incident around 2.30a.m. when Mr. Yadav’s (c)/ clear, (a)/ therefore, we have sent the footage (b)/
mother Murti woke up and saw that the buffaloes to a forensic lab to get a clear picture (c)/ of the car
were missing. (d)/ No error. (e) registration number,” said Mr. Kumar. (d)/ No error.
(e)
148) Police said Raju himself had a criminal record
(a)/ and he as well as one of the attackers (b)/ have 156) “He stopped a scooter that was coming (a)/ his
several cases registered (c)/ with them at the way and took the victim the hospital (b)/ where she
madhura police station. (d)/ No error. (e) was declared dead (c)/ on arrival,” said the official.
(d)/ No error. (e)
149) Two persons working with a cash management 157) The Enforcement Directorate had registered
(a)/ service provider alleged executed a heist of (a)/ a case against Chauhan after the CBI registered
₹32.50 lakh by fleeing (b)/ with a cash van in outer preliminary enquiry (b)/ in June 2015 against Mr.
Delhi’s Mianwali Nagar (c)/ on March 30, police Singh, his wife Pratibha Singh and (c)/ Chuhan and
revealed on Monday. (d)/ No error. (e) other unknown persons. (d)/ No error. (e)

150) Upon checking the identity records of the 158) A Bench headed by NGT Chairperson Justice
gunman, (a)/ it was found out that he was a resident Swatanter Kumar (a)/ asked these defaulting bodies
of Uttar Pradesh’s Etah district (b)/ and his Arms to reply within two weeks (b)/ as to why they should
license were issued (c)/ by the authorities of the not be asked to pay environment compensation (c)/
same region. (d)/ No error. (e) of ₹5 lakh for failing properly manage and treat
sewage. (d)/ No error. (e)
Use Code: (AVP10) for 10% India’s No.1 Teacher in Bank Exams for English Language and Editorial 13
additional discount on All Website: www.vishalparihar.com | Follow on Instagram: @vishalthetrainer
Unacademy courses
1000+ Error Detection for All Exams
Last 20 years (Previous Year Papers)
English By Vishal Parihar

Wildlife, GNCTD, on degraded forest land, gaon


159) Mr. Adityanath also ordered that 73 employees sabha land (c)/ and others available with the
(a)/ of the Noida Authority, who had not been department,” the Minister said. (d)/ No error. (e)
marking themselves (b)/ present on biometric
machine, be marked (c)/ absent since the 167) Mr. Dave also pointed out that Section 10 of the
corresponding days. (d)/ No error. (e) Delhi (a)/ Preservation of Trees Act, 1994, said that
160) Following Chief Minister Yogi Adityanath’s every person who is granted (b)/ permission beside
directions, the authority has (a)/ prepared a roster this Act to fell any tree is bound (c)/ to plant such
of sick and delayed projects and started a crackdown number and kind of trees in the area. (d)/ No error.
(b)/ with developers who have failed to finish their (e)
housing projects, (c)/ leading to problems for
thousands of home buyers. (d)/ No error. (e) 168) Flowing dresses with fitted jackets, haute tops
(a)/ and short skirts with coats, silk brocade skirts
161) Their major civic woes include (a)/ poor and tops (b)/ with appliqué detailing, and gossamer
drinking water supply, no sewage connection, (b)/ lace with lame (c)/ chiffons were the highlight the
erratic electricity supply and poor construction (c)/ collection. (d)/ No error. (e)
of the project between others. (d)/ No error. (e)
169) India has undergone lot of socio-political (a)/
162) Danish, a home buyer who attended the changes since Emergency but freedom (b)/ of
meeting, (a)/ said they are hassled not just because speech keeps coming (c)/ under threat between
of the delay (b)/ in completion of housing projects time to time. (d)/ No error. (e)
but also civic issues (c)/ faced by those who have got
the possess. (d)/ No error. (e) 170) A old man advises her about (a)/ the
inevitability of the course of each one’s life that (b)/
163) The average level of noise pollution (a)/ continues through every birth and points (c)/ out
generally exceeds permissible limits in seven Indian that she is Puranjana reborn as a woman. (d)/ No
cities, (b)/ which includes Mumbai, Delhi, Kolkata error. (e)
and Chennai, (c)/ Union Environment Minister Anil
Madhav Dave said in the Rajya Sabha on Monday. 171) Ramanuja dealt with conflicts of every (a)/ kind
(d)/ No error. (e) -intellectual, philosophical and administrative – and
(b)/ handled them in unique ways, that (c)/
164) The noise standard for motor vehicles, air proclaimed his leadership qualities. (d)/ No error. (e)
conditioners, (a)/ refrigerators, gensets and certain
types of construction equipment (b)/ are prescribed 172) Scriptures hail the renunciation of all that one
in the Schedules (c)/ of the Environment (a)/ considers as one’s own and offer oneself wholly
(Protection) Rules, 1986, the Minister said. (d)/ No (b)/ to the Lord as the highest spiritual attainment,
error. (e) the only (c)/ means by which a jivatma can attain
salvation. (d)/ No error. (e)
165) Over 15,000 trees were cut (a)/ in the Capital in
the three last financial years (b)/ to undertake 173) Women walk the dry landscape that resembles
development and construction activities, (c)/ said a charred (a)/ rice field in search of work, be it the
Union Environment Minister Anil Madhav Dave on Mahatma Gandhi National Rural (b)/ Employment
Monday. (d)/ No error. (e) Guarantee Act’s promised 100 days of work, (c)/ or
indeed any other work that will feed its family. (d)/
166) “Out of 10 sapling, five are planted and (a)/ No error. (e)
maintained by the Department of Forests (b)/ and
Use Code: (AVP10) for 10% India’s No.1 Teacher in Bank Exams for English Language and Editorial 14
additional discount on All Website: www.vishalparihar.com | Follow on Instagram: @vishalthetrainer
Unacademy courses
1000+ Error Detection for All Exams
Last 20 years (Previous Year Papers)
English By Vishal Parihar

174) As you enter Talainayiru block (a)/ of that the time had come to turn a passive homage to
Nagapattinam district, the backwaters spread out an (c)/ active one of adopting sanitation as a goal.
(b)/ enticing, apparently lush and fertile, (c)/ (d)/ No error. (e)
enveloping many villages in the block. (d)/ No error.
(e) 181) It defined both India’s commitment to the
Teesta water-sharing (a)/ agreement and the
175) Elgar, a replacement in the opening positions Central government’s commitment to working (b)/
for the retired Smith, believes (a)/ the side is at with the West Bengal government to conclude the
peace with the loss of their former captain and agreement (c)/ for which the framework was initial
Jacques Kallis, (b)/ who retired from international in 2011. (d)/ No error. (e)
cricket last season, and is looking (c)/ forward to the
years ahead along new skipper Amla, who debuted 182) Not only will these measures strengthen the
in the role in Sri Lanka. (d)/ No error. (e) bonds with Bangladesh, (a)/ with which India shares
its longest international border (b)/ as well as
176) The event bears close study, as it not only historical bonds, they will help India connect to
teaches us the (a)/ political tools that Gandhiji itself, (c)/ to the benefit especially of the
perfected during his struggle for (b)/ the indigo northeastern States. (d)/ No error. (e)
farmers of Champaran, but also the panchamrut
(five nectars) that (c)/ we as Indians got as a result 183) Before we turn to this, however, it may be (a)/
of the struggle. (d)/ No error. (e) useful to suggest some ways in which we can quickly
assuage at least partially (b)/ the hurt that our
177) A day after Hardik Patel held a huge rally in African students must feel, before turning vigorous
Patidar dominated Morbi (a)/ and Tankara in to building institutions that (c)/ ensure their security
Saurashtra region, he and 34 others were (b)/ and encourage them to feel part of the community
booked by the police in Monday for holding the road in India. (d)/ No error. (e)
show and public rally (c)/ without police approval
and for violating prohibitory orders. (d)/ No error. 184) Despite the attempts by the government to
(e) bring international students (a)/ to India, the
experience of foreign students here (b)/ has not
178) The Lok Sabha on Monday cleared always been a happy one, this being particularly so
amendments to the motor vehicles (a)/ law that will for those (c)/ from Africa, though there could be
substantial increase the penalty for traffic violations, exception. (d)/ No error. (e)
(b)/ allow learner’s driving licences to be issued
online (c)/ and penalise contractors for faulty road 185) State governments must be instructed by the
designs. (d)/ No error. (e) Centre to see (a)/ that African students are assured
of their safety and all educational institutions (b)/
179) From a social practice cannot be held to be an must with immediate effect double-up the attention
integral part of a (a)/ faith and belief or an “essential (c)/ they devote to their personal needs, which
religious practice,” it has to satisfy (b)/ the range from housing to food. (d)/ No error. (e)
overarching constitutional goal of gender equality,
(c)/ gender justice and dignity, the government 186) While these are laudable and ambitious
submitted. (d)/ No error. (e) objectives as they address (a)/major concerns of
mental health care, there (b)/ have been some
180) At a programme organised by the Culture critiques attention drawing to the lack of funds, (c)/
Ministry, he said the (a)/ country had paid homage trained personnel, and insufficient emphasis on
to Mahatma Gandhi though all these years and (b)/ community care. (d)/ No error. (e)
Use Code: (AVP10) for 10% India’s No.1 Teacher in Bank Exams for English Language and Editorial 15
additional discount on All Website: www.vishalparihar.com | Follow on Instagram: @vishalthetrainer
Unacademy courses
1000+ Error Detection for All Exams
Last 20 years (Previous Year Papers)
English By Vishal Parihar

193) Note that it was sensitive about the dent the


187) Besides proximity and enduring relationships ban has caused (a)/ to the economy, the Bench
between actors (a)/ and their directors, and some asked the car companies (b)/ to come up with
nepotism that has stuck, the problem with (b)/ the possible alternatives to (c)/ curb the rising air
National Awards is that there is always the view that pollution in the city. (d)/ No error. (e)
someone more deserving (c)/ was overlooked in the
run up to the awards or in the finale. (d)/ No error. 194) Po’s role-reversal from a student to a teacher
(e) delightfully plays out (a)/ in set pieces that involve a
valley of Pandas training in Kung Fu while doing (b)/
188) The Rajya Sabha was adjourned twice on that one thing they love doing; example: a particular
Monday as Congress (a)/ members rushed to the (c)/ panda disrupts the enemy by hug because that’s
well, alleging that the Enforcement Directorate and what he likes doing the best. (d)/ No error. (e)
the CBI were being used (b) against the Chief
Ministers and former Chief Ministers of the 195) Delhi Police Commissioner Alok Verma’s first
Opposition, while (c)/ no action was took against the meeting (a)/ with journalists was little more than a
Chief Ministers of the BJP. (d)/ No error. (e) photo-op with (b)/ the police chief sidestepping
almost all the issues (c)/ raised by the scribes about
189) The Pakistan move to award the death access to him. (d)/ No error. (e)
sentence to Mr. Jadhav (a)/ has been received with
many disbelief by his former Naval (b)/ colleagues 196) On discussing about the complaints, Standing
and intelligence circles, though most of them were Committee chairperson Radhey Shyam Sharma (a)/
(c)/ confident that he would be back home. (d)/ No ordered officials to “stop attending the meetings of
error. (e) the DDCs” and (b)/ “not to allow any work by an MLA
without (c)/ the nod of the head of the department”.
190) “I will continue to work on our government’s (d)/ No error. (e)
principles of development (a)/ and welfare and will
not allow anything to (b)/ shake the confidence 197) The initiative was launched in line with a
place in me,” Mr. Modi said (c)/ at the end of a detailed action plan (a)/submitted by the Aam
marathon five-hour meeting. (d)/ No error. (e) Aadmi Party (AAP) government to the Supreme
Court after (b)/ it demanded information(c)/ on
191) The Election Commission said on Monday that steps be taken to improve air quality and reduce air
it was not bound (a)/ consult the Union Home pollution in the Capital. (d)/ No error. (e)
Ministry before holding any election, following (b)/
reports that it had ignored the Ministry’s advice 198) Even as the recently declassified documents
against holding by (c)/ elections in the Kashmir relating to the (a)/ disappearance of Netaji Subhas
Valley consider the prevailing security situation. (d)/ Chandra Bose do not take us any closer (b)/ to
No error. (e) finding out whether he indeed survived the August
18, 1945 air crash, (c)/ they throw many unverified
192) The Indian Navy spokesperson was quote as documents that claim he outlived the crash. (d)/ No
saying: (a)/ “In a show of international maritime error. (e)
cooperation against piracy, a boarding party (b)/
from the nearby Chinese Navy ship went on board 199) A day before a total ban on the sale and
the merchant ship, (c)/ while an Indian naval consumption of (a)/ country-made liquor across
helicopter provided air cover for the operation. (d)/ Bihar, vendors and outlets offered a huge discount
No error. (e) to exhaust (b)/ their stocks while guzzlers were seen

Use Code: (AVP10) for 10% India’s No.1 Teacher in Bank Exams for English Language and Editorial 16
additional discount on All Website: www.vishalparihar.com | Follow on Instagram: @vishalthetrainer
Unacademy courses
1000+ Error Detection for All Exams
Last 20 years (Previous Year Papers)
English By Vishal Parihar

returning back (c)/ to the shops to replenish their Odissi, as the dance form was gaining popularity.
stocks at home. (d)/ No error. (e) (d)/ No error. (e)

200) It further says that it is mandatory that athletes 207) Steelmakers in Britain pay some of the highest
(a)/ must have achieved the qualification standard energy costs (a)/ and green taxes in the world, but
in AFI-recognised meets (b)/ during 2015 or 2016; the government maintains that the (b)/
moreover, an AFI technical delegate (c)/ must have fundamental problem facing the industry is the
present at the meet. (d)/ No error. (e) collapse in the (c)/ price of steel, caused on
overcapacity in China. (d)/ No error. (e)
201) Footwork is reaction to the ball but balance (a)/
actually starts earlier, from your stance, (b)/ your 208) Just when everyone were wondering whether
set-up to seeing the ball; that is, how still (c)/ you are (a)/ the Saradha probe was being soft pedalled, the
before the ball is being released.” (d)/ No error. (e) Enforcement Directorate (b)/ filed its first charge
202)Digital influence refers to the role that (a)/ the sheet, four days before (c)/ West Bengal goes to the
digital medium plays in influencing (b)/ purchases, polls. (d)/ No error. (e)
irrespective of if the (c)/ purchase is made online or
offline. (d)/ No error. (e) 209) Even in those case, judges wilfully ignored (a)/
the mandatory sentencing provisions of at least one
203) Vettel and Raikkonen made the most of slow year (b)/ of imprisonment, preferring instead a
start by (a)/ the two Mercedes men in Melbourne to “simple imprisonment (c)/ till the rising of the
take control of the race and lead (b)/ for more than court”. (d)/ No error. (e)
half the distance before they were reeled in —(c)/
partly due to tyre strategy decisions — and 210) Besides that, a financial loss to the tune of
overhauled .(d)/ No error. (e) hundreds of crores (a)/ was incurred by people
whose homes and business establishments were
204) Towards the end his gentle gestures and silence (b)/ damaged in the arson that followed the
indicate (a)/ the resolution of the conflict in a subtle agitation leaving (c)/ the State paralysed for near
manner – there is nothing (b)/superfluous in his two weeks. (d)/ No error. (e)
portrait only intense concentration to bringing to
the fore the calm that (c)/ is achieved with 211) C.S. Santosh is also the face of Indian adventure
reconciliation and to come to terms with the on the international stage (a)/ with participation in
changing times. (d)/ No error. (e) the Dakar Rally — considered the graveyard of
champions (b)/ in more ways than one and official
205) Around the years, I have gotten to know the the toughest off- (c)/ road rally in the world — and
British (a)/ filmmaker Stephen Hopkins, and I am the World Cross Country Championship. (d)/ No
keenly looking forward to watching his “Race”, (b)/ error. (e)
a fictionalised version of Owens at the Berlin
Olympics, (c)/ made with the cooperation of the 212) Applicant companies should submit a common
Jesse Owens Foundation as well as the Owens application form SPICe (INC 32) on MCA portal, (a)/
family. (d)/ No error. (e) and once the data of incorporation are sent to CBDT
by (b)/ MCA, the PAN and TAN are issued
206) It was Y.G Doraiswamy, an art connoisseur immediately (c)/ without any further intervention of
from Chennai, (a)/ who saw his performance and the applicant. (d)/ No error. (e)
suggested that he should move (b) to Chennai to
perform Odissi and teach there to local 213) With three runways operating simultaneously,
Bharatanatyam dancers, (c)/ interested of learning it is easier (a)/ to manage IGIA’s heavy traffic even
Use Code: (AVP10) for 10% India’s No.1 Teacher in Bank Exams for English Language and Editorial 17
additional discount on All Website: www.vishalparihar.com | Follow on Instagram: @vishalthetrainer
Unacademy courses
1000+ Error Detection for All Exams
Last 20 years (Previous Year Papers)
English By Vishal Parihar

during peak hours, but with two only (b)/ runways home the threat to developed countries’ steel
available, we can’t have (c)/ simultaneous and industries (b)/ from a glut caused by over-capacity in
independent arrivals and departures,” said a senior China, which has led to a (c)/ collapse in the global
air traffic control official. (d)/ No error. (e) price of commodity steel used mainly in
construction. (d)/ No error. (e)
214) When confronted with the view that the
stretch (a)/ looked more or less the same after 221) On August 19 last year, when the purported
cleaning, (b)/ a senior government official said that video was filmed, (a)/ he was on duty, but due to
a more visible (c)/ impact would came after regular inadequate sleep and not having taken medicines,
maintenance. (d)/ No error. (e) (b)/ he experienced memory loss and drowsiness,
resulting in a (c)/ situation in which he could not
215) The programme will include “a weekly three- even change his uniform, Salem had said. (d)/ No
hour course (a)/ on the presidential campaign error. (e)
taught in real time (b)/ and the participants will be
on the campaign trail, going from (c)/ town to town, 222) “They were also directed to ensure that break-
attending events, and live with American families.” down teams were (a)/ always ready to keep the fleet
(d)/ No error. (e) on track and that buses plied on (b)/ the extreme left
side of roads; there would be disciplinary action
216) “I really would not have believed that one can along depot (c)/ managers if any of these conditions
plan a (a)/ giant flyover on the narrowest of spaces were found not being met,” the official added. (d)/
like this, (b)/ unless I come here,” said an engineer No error. (e)
of the (c)/ Kolkata Municipal Corporation while
photograph the area. (d)/ No error. (e) 223) “If farmers are confident that they would get
217) The scheme was formulated under directions assured returns (a)/ on their produce it will motivate
of the Supreme Court (a)/ which had acted in a PIL them to go for pulses farming, (b)/ resulting in better
filed by activist and lawyer MC Mehta (b)/ who had production, less dependence in imports and (c)/
demanded shifting of industries operating in most importantly will bring price stabilisation,” he
residential (c)/ and non-conforming areas to the said. (d)/ No error. (e)
outskirts for curbing pollution. (d)/ No error. (e)
224) Rubber deposited on the surface from tyres of
218) When broken down point by point, it is quite landing aircraft (a)/ would be thorough scraped and
apparent (a)/ that either there was no ground for the usual touchdown point, (b)/ which bears the
invoking Article 356 within the parameters (b)/ laid maximum load of landing aircraft, (c)/ would be
down by the Supreme Court or that a situation (c)/ overhauled to reduce chances of damage. (d)/ No
warranting Central intervention had not yet arise. error. (e)
(d)/ No error. (e)
225) But Mr. Modi did raise the possibility of
219) In the judgment that uphold the validity of the individuals within (a)/ the state structure getting
Tenth Schedule (a)/ (the anti-defection law), a friendly with a terrorist, (b)/ lead up to a nuclear
dissenting judge had pointed out that the (b)/ terrorism incident, without (c)/ naming any
Speaker’s “tenure being dependent on the will of particularly country, said Mr. Gill. (d)/ No error. (e)
majority therein, (c)/ likelihood of suspicion of bias
could not be ruled out.” (d)/ No error. (e)

220) The announcement that India’s Tata Steel is Read each of the following sentences to find out if
abandoning western countries (a)/ have hammered there is any grammatical error in it. The error, if
Use Code: (AVP10) for 10% India’s No.1 Teacher in Bank Exams for English Language and Editorial 18
additional discount on All Website: www.vishalparihar.com | Follow on Instagram: @vishalthetrainer
Unacademy courses
1000+ Error Detection for All Exams
Last 20 years (Previous Year Papers)
English By Vishal Parihar

any, will be in one part of the sentence. The considered view, (C)/the pandemic triggered
number (A, B, C or D) of this part is your answer. If economic crisis will be inall possibility be
there is no error in the statement, then mark ‘U’shaped(D).
option (E) as your answer choice. A) A B) D C) C D)
B E) No correction required
226. A moot question is that, if IDs are considered to
be the foremost (A)/ tools for ensuring corporate 232. What’s happening isn’t hard to decipher :the
governance, why have this non-compliant supply shock (A)/has dragging demand down with it,
companies (B)/ been allowed to remain listed and through loss (B)/of jobs, and that is further pulling
traded without (C)/ the safeguard of IDs, in many down (C)/the productive forces in a negative spiral.
cases, for very long periods of time? (D) (D)
A) B B) A C) C D) A) B B) D C) C D)
D E) No correction required A E) No correction required

227. ONGC, one of the largest listed company, citing 233. Three years ago , if State governments had the
lack of powers (A)/in deciding upon the board’s premonition that(A)/ embracing GST would lead
composition including the appointment of them to a situation wherein (B)/they would have to
(B)/independent directors and board’s evaluation, borrow to make good revenue (C)/shortfalls, they
has sought exemption (C)/from the related would have opted out of one tax regime. (D)
regulations on corporate governance. (D) A) A B) C C) D D)
A) A B) C C) B D) B E) No correction required
D E) No correction required
228. It needs to be highlighted that (A)/compliance 234. Just as organizations will be looking at
and disclosures are far superior (B)/at the IPO stage minimizing costs and (A)/growing revenues by
across many aspects, (C)/but falls significantly post increasing their geographic footprint and entered
listing. (D) (B)/new market segments taking advantage of the
A) D B) A C) B D) digital transformation (C)/sweeping the world,
C E) No correction required families will need to operate as a business unit to
229. Coupled with the uncertainty caused by a maximise their its earning capacity(D)
pandemic (A)/ that is still taking a growing toll, more A) A B) B C) C D)
gathering clouds and a global scenario (B)/that is D E) No correction required
equally dismal, the nation has to brace (C)/ himself
for a long grind. (D) 235. While banks almost always (A)/meets the
A) C B) D C) B D) overall target, keepingup with the
A E) No correction required (B)/subtargetswas getting (C)/difficult for banks
230. Over the course of about five months since the with limitedexpertise in certain sectors. (D)
Covid19pandemic (A)/struck, policymakers have A) D B) C C) B D)
choose fiscal (B)/and monetary stimulus particularly A E) No correction required
, in the same(C)/ manner as they did in the
aftermath of global financial crisis of 2008-2009. (D) 236. The GST reform was hailed as a (A)/great
A) A B) B C) C D) experiment in “cooperative federalism” (B)/with all
D E) No correction required the states and UTs with legislatures (C)/joining the
Centre in the reform to harmonise domestic trades.
231. The uncertainty and the risk of a severe health (D)
disaster will (A)/jeopardized the traditional demand A) B B) D C) A D)
management stimulus (B)/interventions, and in our C E) No correction required
Use Code: (AVP10) for 10% India’s No.1 Teacher in Bank Exams for English Language and Editorial 19
additional discount on All Website: www.vishalparihar.com | Follow on Instagram: @vishalthetrainer
Unacademy courses
1000+ Error Detection for All Exams
Last 20 years (Previous Year Papers)
English By Vishal Parihar

2.6% increase for wheat is only half the cost


237. The agency is understood to be (A)/ confronted escalation(D)
the official with the (B)/digital evidence it had A) A B) D C) C D)
extracted from the(C)/ gadgets it seized from the B E) No correction required
accused in the case. (D)
A) C B) A C) D D) 244. The pressure of feed India’s (A)/1.3 billion
B E) No correction required people has led to suboptimal choices, (B)/ like a
water-guzzling rice crop (C)/grown in abundance in
238. The action against non-compliance (A)/, at Punjab that bleeds aquifers dry. (D)
present, lies mainly (B)/in the imposition of fines, A) A B) C C) B D)
through(C)/ there are paltry, on the errant D E) No correction required
companies. (D)
A) C B) B C) D D) 245. The criticism together Gillian Flynn’s
A E) No correction required (A)/portrayal of women has barely subsided (B)/and
now she must (C)/batten down the hatches for
239. While private initiative should (A)/not be another onslaught. (D)
delegitimised, it has had only a (B)/limited impacted A) A B) B C) D D)
on building capabilities (C)/ in India as it has focused C E) No correction required
on those with the ability to pay. (D)
A) D B) A C) B D) 246. Even though Cusack’s character — Dr Kevin
C E) No correction required Christie — is (A)/an American, I’m worried that
Utopia (B)/movie might played into the Trumpsters’
240. Pointing to the incongruence among (A)/India’s (C)/narrative that the Covid-19 virus was
low level of human development(B)/ and its status manufactured in a Chinese laboratory. (D)
as a democracy (C)/evokes the response that this is A) A B) B C) C D)
to see the latter in instrumental terms. (D) D E) No correction required
A) A B) B C) C D)
D E) No correction required 247.Trump has campaigning (A)/against large-scale
use of mail-in ballots for (B)/the upcoming elections,
241. In India, the state’s ritualistic attachment alleging, without any (C)/evidence, that it will be
(A)/with the procedures of democracy (B)/has not used for election fraud by Democrats. (D)
been matched by an (C)/awareness of their implicit A) A B) B C) D D)
goal of a fulfilling life for Indians. (D) C E) No correction required
A) A B) D C) C D)
B E) No correction required 248. There are two kinds of mailing ballot
(A)/systems: universal mail-in voting, when states
242. Dystopia will imagined as a place (A)/where the (B)/send ballots to all eligible voters; (C)/and
people experience great suffering (B)/ as they fend absentee balloting, when a voter was sent a mail on
for themselves (C)/under the watchful eyes of an request. (D)
authoritarian state. (D) A) A B) C C) D D)
A) C B) D C) B D) B E) No correction required
A E) No correction required
249. The absconding foreigners make up 90% of
243. Modi has assured farmers the (A)/support will (A)/foreign nationals who were enlarged on bail
stay by announcing prices against the (B)/winter (B)/pending trial and has fled the country or
crop, though as farmer activists (C)/have noted, the
Use Code: (AVP10) for 10% India’s No.1 Teacher in Bank Exams for English Language and Editorial 20
additional discount on All Website: www.vishalparihar.com | Follow on Instagram: @vishalthetrainer
Unacademy courses
1000+ Error Detection for All Exams
Last 20 years (Previous Year Papers)
English By Vishal Parihar

otherwise made (C)/themselves scarce and not carved a niche for themselves in different walks of
available to face prosecution. (D) life. (D)
A) B B) C C) D D) A) B B) C C) D D)
A E) No correction required A E) No correction required

250. Six months after a man in Odisha’s Kandhamal 256. On the 75th anniversary of the UN’s
(A)/district lodged a complaint from Lok Adalat (B)/ foundation, (A)/not only is great power rivalry rising
after failing to get his Aadhaar card after more than to (B)/Cold War levels, but the world is under
20 (C)/attempts, the hearing in the case is yet to (C)/enormous forces of change, whether
begin. (D) technological or environmental. (D)
A) A B) B C) C D) A) B B) D C) A D)
D E) No correction required C E) No correction required
257. In the aftermath of the “letter bomb” (A)/in the
251. With virus cases on the rose and private Congress that didn’t quite exploded, (B)/there has
diagnostic (A)/centres taking at least 24 hours to been much (C)/angst over the apparent lack of inner
(B)/provide available slots, many people are now party democracy in the party. (D)
lining (C)/up outside mohalla clinics to get A) A B) C C) D D)
themselves tested. (D) B E) No correction required
A) A B) D C) C D)
B E) No correction required 258. The dreary evening of February (A)/23 saw
parts of North-East Delhi (B)/gripped by a communal
252.In a circular that factors in the uncertainties frenzy, (C)/reminiscent of the carnage during the
(A)/surrounding COVID-19, the UGC has (B)/offered days of Partition. (D)
institutions the option in (C)/offline, online or A) A B) D C) C D)
blended modes of instruction. (D) B E) No correction required
A) A B) D C) B D)
C E) No correction required 259. The entire territories of the UTs of (A)/Jammu
and Kashmir and Ladakh have been and is (B)/an
253. India’s move to unlock education coincides integral part of India and would remain so (C)/and
(A)/with a reported reduction in overall daily Pakistan has no locus standi to comment on India’s
(B)/incidence of infection, although (C)/absolute internal matters. (D)
numbers is frighteningly high. (D) A) C B) D C) A D)
A) D B) C C) A D) B E) No correction required
B E) No correction required
260. The Supreme Court on Thursday issued(A)/ a
254. Globally, unpreparedness to handle (A)/the notice to the Centre and the Union Public
pandemic and near collapse to the health (B)/Service Commission (UPSC) on a plea filed by
(B)/systems denuded our (C)/ability to focus on UPSC (C)/aspirants urging the court to postponed
health. (D) the upcoming civil services exam. (D)
A) D B) C C) A D) A) A B) C C) D D)
B E) No correction required B E) No correction required

255. Erudite scholars and alumni from (A)/our 261. Chief Minister Shivraj Singh Chouhan said
University are being acknowledged amongst the (B)/ (A)/that a scheme to provide an interest-free loan
thought leaders of the (C)/country and they has (B)/from Rs 10,000 to rural street (C)/vendors of
Madhya Pradesh has been implemented. (D)
Use Code: (AVP10) for 10% India’s No.1 Teacher in Bank Exams for English Language and Editorial 21
additional discount on All Website: www.vishalparihar.com | Follow on Instagram: @vishalthetrainer
Unacademy courses
1000+ Error Detection for All Exams
Last 20 years (Previous Year Papers)
English By Vishal Parihar

A) A B) B C) C D)
D E) No correction required 268.The schools, on their part, are not taking
chances (A)/and have made mandatory the consent
262. The Islamic scholar Zakir Naik, who has (B)/of guardians, besides putting in place the
(A)/courted controversy for his regressive remarks, (C)/Covid protocol to rule out chances of the
(B)/is being closely monitored (C)/across the infection being contract. (D)
Narendra Modi-led government. (D) A) C B) D C) A D)
A) D B) B C) C D) B E) No correction required
A E) No correction required
269. As the UN has emphasises, getting students(A)/
263.The decision to withdraw tenders for the safely back to the classroom must be a top
(A)/supply of ammunition ranging from (B)/rifles, (B)/priority as we confront a generational
fired grenades (C)/to artillery rounds is sure to (C)/catastrophe that could waste untold human
dampen spirits in the industry. (D) potential. (D)
A) B B) C C) D D) A) A B) B C) C D)
A E) No correction required D E) No correction required

264.After 90 years of safekeeping, a Pakistani 270. Decisions that impact the lives (A)/of billions
(A)/Sufi organisation has transferred 110-years old requires scrutiny and fixing of (B)/accountability, yet
(B)/copies of rare manuscripts of the Guru Granth dealing (C)/with a once-in-a-century pandemic is a
(C)/Sahib to a gurdwara in Sialkot, the media tough call. (D)
reported on Thursday. (D) A) D B) C C) B D)
A) A B) D C) C D) A E) No correction required
B E) No correction required
271. While most question the (A)/severity of the
265. For the beleaguered hospitality(A)/ industry, shutdown, some fail to(B)/ comprehend the urgency
the brisk business (B)/reported by hotels at the to (C)/unlock trade and business activity. (D)
tourist (C)/hotspot of Kasauli will comes as a relief. A) A B) B C) C D)
(D) D E) No correction required
A) A B) D C) C D)
B E) No correction required 272.The only sector in the Indian economy (A)/that
survived the Covid contraction is now being
266. Wherever agriculture is being opened to (B)/forced to suffered needless anxiety, with the
markets, (A)/the big capital have successfully government (C)/refusing to address farmers’
(B)/managed to push out the majority (C)/farming legitimate apprehensions. (D)
population and concentrated its control over food. A) A B) D C) C D)
(D) B E) No correction required
A) B B) C C) D D)
A E) No correction required 273. The Covid-19 pandemic has worryingly been
(A)/relegated to the sidelines in Punjab, (B)/even
267. That Covid-19 has disrupted education in an though the state (C)/fatality rate continues to be
(A)/unprecedented way is illustrated (B)/by the fact among the highest in the country. (D)
that schools reopened (C)/after a gap of six months A) D B) B C) C D)
on Monday to a lukewarm response. (D) A E) No correction required
A) A B) B C) C D)
D E) No correction required
Use Code: (AVP10) for 10% India’s No.1 Teacher in Bank Exams for English Language and Editorial 22
additional discount on All Website: www.vishalparihar.com | Follow on Instagram: @vishalthetrainer
Unacademy courses
1000+ Error Detection for All Exams
Last 20 years (Previous Year Papers)
English By Vishal Parihar

274.IN a show of austerity, Parliament(A)/ has cut „antinational‟ or „anti-democratic‟ and such
the salaries and allowances of (B)/ministers and MPs assertions are an attacked on people‟s commitment
to (C)/meet the exigencies arising from of Covid-19. to protecting constitutional values.
(D) A) Only II B) I & III C) None is correct
A) A B) B C) D D) D) Only I E) No correction required
C E) No correction required
278. I. The video gone viral on social media
275. Truth and accuracy are the prime (A)/casualty platforms on Friday, but caught police‟s attention
when TV anchors and reporters (B)/stoop to new when right-wing activists approached the college
(C)/lows just to grab eyeballs. (D) seeking action against the students on Saturday. II.
A) A B) B C) C D) Production of several commonly used medicines
D E) No correction required may be adversely hit along with smartphones and
solar equipment if the supply disruption caused by
the outbreak of coronavirus in China lasts beyond
two months. III. Quite often, disasters are proof of
Each question contains three sentences I, II and III. our own failings, man-made transgressions of order,
The error, if any, may be in one or more parts of the of throw caution to the winds.
sentence. Find the error and mark the answer from A) II & III B) I & III C) None is correct
options A, B, C and D. If there is no error in the D) I & II E) No correction required
statement, then mark option (E) as your answer
choice. 279. I. The government ordered an inquiry, but
members of the opposition wanted to know why the
276. I. An Air India aircraft was force to get airborne circus company was issue a license to operate in
earlier than planned at Pune airport to avoid such an unsafe place. II. The rumour mills were
colliding with an IAF jeep that was dangerously near abuzz too: word spread that the horses had bolted
the airstrip. II. Credit flow is reviving slowly and towards Majestic, a lion was prowling in the vicinity
steadily and is set to improve on the back of steps and a hippo was bounding the railway colony. III.
took by the government and RBI. III. Even as Gujarat Rescue work was challenging, not only because it
waits with bated breath to welcome the US had to be done in the glare of the headlights of
President with “Kem Chho Trump”, the state police and fire brigade vehicles, but also because of
government has been directed to replace the the location.
Gujarati expression with “Namaste President A) I & III B) Only I C) None is correct
Trump” to help the event find resonance across D) Only II E) No correction required
India.
A) Only I B) I & II C) II & III D) 280. I. Though the place is devoid of large wing
None is correct E) No correction required scavengers, the shrub forest hosts many tiny birds
whose chirping rent the air, especially during
mornings and evenings. II. Every Saturday, the
277. I. Within the bounds of law, liberal democracies temple is thronged by villagers and other devotees
ensure that citizens enjoy the right to expressed – many of what cover long distances – to pray to the
themselves in every conceivable manner, including deity. III. As though to celebrate the eternal love of
the right to protest, and express dissent against the washerman and his wife, the place also plays
prevailing laws. II. What is of utmost relevant today host to many young couples during weekends.
is our ability and commitment to preserve, conserve A) II & III B) I & II C) Only I D)
and build on the rich pluralistic history that we have None is correct E) No correction required
inherited. III. Dissenting voices cannot be labeled
Use Code: (AVP10) for 10% India’s No.1 Teacher in Bank Exams for English Language and Editorial 23
additional discount on All Website: www.vishalparihar.com | Follow on Instagram: @vishalthetrainer
Unacademy courses
1000+ Error Detection for All Exams
Last 20 years (Previous Year Papers)
English By Vishal Parihar

281. I. The justice took treatment for 49 days and Vodafone mulls the amount that it will be paying,
resumed work despite not having fully recovered, Sunil Mittal‟s Airtel has already make clear its
keeping the interest of the institution and the intentions to pay, as well as the schedule. III. The RBI
general public in mind. II. Filled with paper notes said that the credit flow was reviving and is expected
that evolve in the country even before the British to improve further in the coming months on the
Rule, the museum has over 700 artefacts collected back of steps taken by the central bank and the
over a span of 20 years. III. A Hyderabad Rs 10 note government.
which was losted in a shipwreck in 1932 and was A) Only I B) I & II C) II & III D)
salvaged later was the cynosure of all eyes on None is correct E) No correction required
Saturday.
A) II & III B) I & III C) None is correct 285. I. RBI has reduced policy rates five time since
D) Only II E) No correction required February 2018 and there are indications that banks
are passing on the cuts now. II. The RBI has proposed
282. I. Every western country has began a change in its JulyJune accounting year to align from
appreciating yogic sciences and is taking it as a the government‟s financial year of AprilMarch. III.
holistic approach towards health after consulting The RBI would be able to provide better estimates of
right doctors. II. Our doctors need to equip the projected surplus transfers of the government
themselves with latest technology like Artificial for the financial year of budgeting purposes.
Intelligence and machine learning to match A) None is correct B) Only II C) I & III
international standards. III. New technologies like AI D) II & III E) No correction required
for better diagnosis of an ailing patient and robotics
for conducting surgical procedures have to be 286. I. The Indian embassy in Japan said it is making
embraced, along with IT and computer skills. efforts to disembark all the Indians on board the
A) II & III B) I & II C) Only I D) cruise ship moored off the Japanese coast after the
None is correct E) No correction required end of the quarantine period. II. India‟s biggest
Chinatown, has ended up being an unwitting victim
283. I. Industrialization and technological of coronavirus that has already claimed 1367 lives in
advancement have also adversely affected the mainland China. III. Terming it a direct invasion and
environment and led to new lifestyle diseases attack on media freedom, the Madras high court has
emerging new challenges. II. A day after Scindia said quashed criminal defamation proceedings initiated
he would hit the streets if manifesto promises by former CM J Jayalalitha against TOI and a Tamil
weren‟t fulfilled by MP government, his real daily in 2014.
departure from a review meeting further deepened A) Only I B) II & III C) I & II D)
the impression of rift. III. Differences between CM None is correct E) No correction required
Nath and Scindia have been out in the open for
around eight months after Scindia criticised the 287. I. Derailing the print media from publishing
Congress government in MP over crop damage content, which in no way could be termed
survey, farm loan waiver and more. defamatory, by initiating prosecution, is nothing but
A) II & III B) I & III C) Only II D) an attack on the rights enshrined under the
None is correct E) No correction required constitution. II. The case relates to reports on BJP
leader Subramanian Swamy‟s statement accusing
284. I. Embattle telecom provider Vodafone Idea is Jayalalitha of trying to usurp credit for release of
ready to pay the government dues related to the some Indian fisherman from the custody of Sri
Supreme Court‟s AGR judgment, but said that its Lankan authorities. III. AAP has decided to repeat all
ability to remain a going concern hinges on a the ministers because it believes that people voted
leniency in the payment schedule. II. While
Use Code: (AVP10) for 10% India’s No.1 Teacher in Bank Exams for English Language and Editorial 24
additional discount on All Website: www.vishalparihar.com | Follow on Instagram: @vishalthetrainer
Unacademy courses
1000+ Error Detection for All Exams
Last 20 years (Previous Year Papers)
English By Vishal Parihar

it back to power because of the work done by the disproportionate filtering out people in those
government and for continuing the work. categories.
A) Only I B) Only II C) I & III D) A) I & III B) Only I C) None is correct
None is correct E) No correction required D) I & II E) No correction required

288. I. The agency recovered emails of Iqbal Memon 292. I. Though the POSCO act becomes law in 2012,
and analysed allege extortion and drug proceeds changes in infrastructure and judicial staff training
laundered from India to Dubai where he ran a few took a couple of years and the results can be seen
hotels to cover up the dirty money. II. The agency now with testimonies of minors that have ended in
has noted travel details of Dheeraj Wadhwan to convictions. II. There have been a sea change in how
Dubai and London and how he had allegedly helped subtitles are written, because of streaming
the fugitive late drug lord in laundering money to platforms and new markets. III. The growth
London via Dubai through non-banking hawala momentum of the first seven years were
channels. III. It is only through your power of remarkable, but economies have ups and downs and
expressing views and courage of stating contrary there was a downturn in the last three years.
positions that you will make others stop and think. A) II & III B) I & III C) Only II D)
A) I & II B) I & III C) Only II D) None is correct E) No correction required
Only I E) No correction required
293. I. The decision to raise import duties is a major
290. I. Build in 1920 to tame the flood fury of River reversal of the policy of gradually reducing import
Musi, Osman Sagar has served the drinking water duties, which have been in place for 30 years and
needs of a million people in parts of Hyderbad for was followed by several governments. II. The CAG
100 years. II. During his talk on „Hues that make came up with unrealistically large estimates of
India‟ Justice Chandrachud called on audience to revenue losses and public opinion came to the view
include communities that is pushed to the fringe of that not auction spectrum was a big mistake. III.
the mainstream for being minorities – based on their Some critics accuse finance minister Nirmala
language, faith, culture or gender. III. A research Sitharaman of gross fiscal conservation and failure
founded that Walnut consumption by healthy, to use a bi budget stimulus to boost growth.
elderly adults had little effect on cognitive function A) I & II B) I & III C) Only I D)
over two years, but it had greater effect on elderly None is correct E) No correction required
adults who had lower baseline neuropsychological
test scores. 294. I. The MEA‟s management and functioning
A) Only III B) I & III C) None is correct structure remains hidebound and out of sync with
D) II & III E) No correction required modern practices and demands of India‟s foreign
policy, which is turning more complex and diverse.
291. I. Walnuts contain omega-3 fatty acids and II. He was a well-behaved boy, drawing a personality
polyphenols, which has been found to counteract sharply divergent from the one his Facebook friends
oxidative stress and inflammation, both of which are saw: of undisguised hatred towards one community
drivers of cognitive decline. II. When it comes to how and her violent intentions. III. The Election
algorithms and artificial intelligence can enable bias Commission has found Kejriwal in violation of a
in the job hiring process, the biggest issue isn‟t even provision that prohibits ministers and other
with the employers themselves. III. While algorithms authorities from making any promise of
may speed up the process of narrowing the pool of construction roads, provision of drinking water
job candidates, they are often not great at finding facilities etc.
the most qualified ones, and instead, end up A) II & III B) Only II C) I & III D)
None is correct E) No correction required
Use Code: (AVP10) for 10% India’s No.1 Teacher in Bank Exams for English Language and Editorial 25
additional discount on All Website: www.vishalparihar.com | Follow on Instagram: @vishalthetrainer
Unacademy courses
1000+ Error Detection for All Exams
Last 20 years (Previous Year Papers)
English By Vishal Parihar

spirit of its manifesto commitment of “with all,


295. I. Deploring the use of chemicals to ripe fruits, development of all, and trust of all”. II. Balance
Delhi high court equated it to poisoning the between the right to protect, and ensuring that they
consumer and called for a launch of criminal don‟t cross the line to illegality is a balanced difficult
prosecution against such culprits. II. A Delhi court to maintain. III. Urns containing the ashes of
issued notice to Tihar Jail authorities asking them to deceased members of Hindu families in Pakistan are
respond to a plea of the death row convicts seeking piling up in temples and crematoriums there as the
stay on their execute slated for February 1. III. The community awaited restoration of train and bus
Mumbai Police withdrew protection extended to links with India to perform the final ritual.
nine builders who allegedly faced threats from the A) II & III B) Only II C) I & III D)
underworld, saying most of the dreaded gangsters None is correct E) No correction required
were lodged in jails.
A) I & II B) None is correct C) Only I 299. I. Vehicle owners will now receive messages
D) II & III E) No correction required from the transport department if their vehicle don‟t
have third party insurance or the policy has expired.
296. I. Crude prices are in the comfort zone and II. Sending a strong message down the line to lower
product prices are expected to face deflationary courts, a five judge constitution bench of the
pressure as coronavirus casts a shadow over trade. Supreme Court has ruled that anticipatory bail can
II. The deliberations pointed to the strong possibility be granted until the completion of trial. III. Being the
of CAA turning out to be the flashpoint which may largest economy in the subcontinent, India would
mar the Budget session, accusing the government of arguably attract workers from neighbouring
focusing merely on passage of its bills and not countries with lagging economies.
addressing the nationwide protests. III. The National A) I & II B) I & III C) None is correct
Democratic Front of Bodoland gave into arms D) Only I E) No correction required
marking an end to a 34-year armed struggle for a
separate Bodoland state. 300. I. Before a presidential candidate get their
A) Only III B) I & III C) None is correct name on the ballot for election day, they must
D) Only II E) No correction required survive a nomination process, which whittle down
the firld of candidates to one for each party. II.
297. I. The Supreme Court refused to stop BSP MP, Sceptics argue that every census undercounts the
which is accused of rape and is in custody, from illegals since Bangladeshis, fearful of detention or
taking oath as Member of Parliament eight months deportation, lies about their birthplace. III. Within
after he got elected as a candidate of SP-MSP hours of Delhi high court refusing permission for the
coalition. II. While Beijing was yet to approve India‟s execution of the four Nirbhaya case convict
request to operate two aircraft to fly back Indian separately, the Centre challenged it in the Supreme
nationals, the Indian side was hoping evacuate will Court.
finally begin January 31st evening. III. India‟s envoy A) I & III B) Only II C) None is correct
to Brussels has met all the groups who have D) II & III E) No correction required
sponsored the drafts for Brexit in the past couple of
days to explain the Indian perspective. 301. I. Maharashtra CM Thackrey said there was no
A) I & III B) Only I C) I & II D) need to fear the CAA, but asserted his government
None is correct E) No correction required will not allowed the proposed NRC to be
implemented as it would “impact people of all
298. I. Outgoing British high commissioner Dominic religions”. II. The House committee suggested
Asquith expressed hope the Indian government will keeping an option for compensating the surrogate
continue to address the CAA-related issues in the mother beyond medical expenses and insurance
Use Code: (AVP10) for 10% India’s No.1 Teacher in Bank Exams for English Language and Editorial 26
additional discount on All Website: www.vishalparihar.com | Follow on Instagram: @vishalthetrainer
Unacademy courses
1000+ Error Detection for All Exams
Last 20 years (Previous Year Papers)
English By Vishal Parihar

coverage that includes taking care of her nutritional abnormalities, the Supreme Court denied the right
food requirements. III. Cabinet‟s decision to to abort a foetus detected with Down Syndrome at
approve MTP (Amendment) Bill, 2020, will truly 22 weeks. III. Frozen nitrogen covers part of Pluto's
address the needs of gender justice through the surface in the shape of a heart and during the day, a
prism of reproductive rights, providing a solution thin layer of nitrogen ice warms and turns into
which women in our country has sought for decades. vapour.
A) None is correct B) I & III C) Only II A) II & III B) I & II C) None is correct
D) I & II E) No correction required D) Only I E) No correction required

302. I. The government is likely to push for 305. I. As air whips close to the surface, it transports
legislation on curbing child pornography after a heat, grains of ice and haze particles to create dark
parliamentary ad hoc committee reported to the wind streaks and plains across the north and north-
House the “serious menace” of child porn. II. In the western regions. II. Russian priests should refrained
Indian context, anticipatory bail was found from the practice of blessing nuclear weapons and
necessary because of incessant targeting of political other weapons of mass destruction that can inflict
rivals and the tendency of police to used its power indiscriminate loss of life. III. Fear of undertrial
of arrest even in cases which require no custodial prisoners absconding, especially those with no one
interrogation. III. Every year the Annual Status of to vouch for their credentials, hinders a more liberal
Education Report brings the sad news that less than bail policy.
half the students in Class V can read a paragraph or A) Only III B) Only II C) I & III D) I
do an arithmetic sum from a Class II text. & II E) No correction required
A) Only II B) I & III C) None is correct
D) Only I E) No correction required 306. I. The government is looking to introduce
prefilled forms with your capital gains data on
303. I. Educationists fear that a ban will cut off Indian mutual fund and share sales in time for you to file
children from the learn revolution happening in the the income tax returns by the end of July. II. Atalji‟s
world, especially digital learning, and disconnect NDA government made serious efforts to ensuring
them from job opportunities in the knowledge safer abortions, by eliminating abortion by
economy. II. The interesting thing is that one who untrained persons and in unhygienic conditions,
really becomes youthful does not become old, thus reducing maternal morbidity under the aegis of
because one who knows the secret of being a youth the MTP. III. The Union Cabinet cleared
does not need to become old. III. Demographic and amendments to the Banking Regulation Act to
economic trends in the two countries suggest that strengthen the regulation of cooperative banks after
India‟s economic attraction for Bangladeshi the massive fraud at PMC Bank, which were
migrants have diminished, especially in Assam which unnoticed and exposed several gaps in the legal
is among the most poorest and least economically framework.
dynamic of Indian states. A) I & III B) None is correct C) Only I
A) I & III B) Only II C) None is correct D) Only II E) No correction required
D) I & II E) No correction required
307. I. Progressive companies are becoming aware
304. I. Illustrating the heavy traffic congestion on of the need to just provide health insurance, but
Indian roads, a report released recently has ranked ensure that employees remain healthy. II.
three Indian cities in the top five among 416 cities Underlining the effectiveness of mediation to
across 57 countries, to having the worst traffic address commercial disputes, Chief Justice S A
gridlocks. II. Despite a plethora of judgments Bobde said it is time to have a comprehensive law
allowing termination of pregnancy in case of foetal on making pre-litigation mediation mandatory. III.
Use Code: (AVP10) for 10% India’s No.1 Teacher in Bank Exams for English Language and Editorial 27
additional discount on All Website: www.vishalparihar.com | Follow on Instagram: @vishalthetrainer
Unacademy courses
1000+ Error Detection for All Exams
Last 20 years (Previous Year Papers)
English By Vishal Parihar

The bill allows abortion up to 24 weeks of any better than we treat animals bred for meat, fur
gestational age for vulnerable categories of women and milk.
and there is no limit of gestational age in case of A) II & III B) Only I C) I & III
pregnancies with substantial foetal abnormalities, D) None is correct E) No correction
diagnosed by a medical board. required
A) I & II B) I & III C) Only I
D) None is correct E) No correction 311. I. Humans live longer and stay healthy and
required active for much longer due to better nutrition and
the ability to combat disease. II. Don‟t be friend with
308. I. Under the tent, occupied mostly by farmers people who are complaining and negative in their
from Punjab during the day, the organizers went mindset because if you are friendly with them, you
around requesting people not to interact with the also get into that mode. III. Doesn‟t matter if it‟s a
media and refrain from political suggestions. II. The fast food joint or an upscale diner, a cup of coffee or
bill seeks to strengthen provisions for protecting the a round of beer, indulging in eating and drinking is a
dignity and privacy of women who seek the refugee favourite pastime in many.
of law when confronted with such a life altering A) I & II B) II & III C) None is
decision. III. The government believes that the correct D) Only I E) No correction required
slowdown of the economy is due to cyclical factors
and the upturn will happen if they do more of the 312. I. India called off Sri Lanka to fulfill the
same – scrounge for money. aspirations of the island nation‟s Tamil minority by
A) Only III B) Only II C) I & III taking forward the process of reconciliation and
D) None is correct E) No correction implementing a constitutional provision aimed at
required the devolution of powers. II. As a Delhi went to the
polls, polling locations in most unauthorized
309. I. In its bid to open up the railways for private colonies in the city were abuzz with conversations
investment, the railway ministry has prepared an suggesting how development and welfare schemes
ambitious plan to allow private players to run 500 focused in such colonies. III. It would be useful to
passenger trains, manage 750 stations and even buy remind oneself that the rights which citizens cherish
rolling stock from private players. II. Recognising the deeply is fundamental – it is not the restrictions that
burgeoning numbers of undertrial prisoners in jails, are fundamental.
initiatives to free those not accused of heinous A) I & II B) Only II C) I & III
offences on personal bail need to be stepped down. D) None is correct E) No
III. Intelligence can perhaps been described in correction required
functional terms, as the ability to learn new skills
and solve various types of problems. 313. I. The impact on the stock of unaccounted
A) Only I B) II & III C) I & II money will be known only after a few 34. I. Once a
D) None is correct E) No correction machine achieves superhuman competence in any
required given domain, that competence can be rapidly
replicate multiple times. II. Spirituality is that which
310. I. Once a machine achieves superhuman enhances the sense of belongingness among people,
competence in any given domain, that competence so there is that natural tendency to be honest and to
can be rapidly replicate multiple times. II. Spirituality care for each other. III. There is no apparent reason
is that which enhances the sense of belongingness a higher intelligence will treat us any better than we
among people, so there is that natural tendency to treat animals bred for meat, fur and milk.
be honest and to care for each other. III. There is no
apparent reason a higher intelligence will treat us
Use Code: (AVP10) for 10% India’s No.1 Teacher in Bank Exams for English Language and Editorial 28
additional discount on All Website: www.vishalparihar.com | Follow on Instagram: @vishalthetrainer
Unacademy courses
1000+ Error Detection for All Exams
Last 20 years (Previous Year Papers)
English By Vishal Parihar

A) II & III B) Only I C) I & III be somewhat miffed that their battery of
D) None is correct E) No correction economists was not consulted by the PMO, the
required Finance Ministry or the RBI while planning
demonetisation. II. Three weeks after PM Modi‟s
314. I. Humans live longer and stay healthy and announcement about withdrawal of Rs 1000 and Rs
active for much longer due to better nutrition and 500 currency notes from circulation, the impact
the ability to combat disease. II. Don‟t be friend with continues to be felt by the man on the street. III.
people who are complaining and negative in their Smokers can effectively turn back time in their lungs
mindset because if you are friendly with them, you by kicking the habit, with healthy cells emerging to
also get into that mode. III. Doesn‟t matter if it‟s a replace some of their tobacco damaged and cancer-
fast food joint or an upscale diner, a cup of coffee or prone ones.
a round of beer, indulging in eating and drinking is a A) Only I B) II & III C) I & III
favourite pastime in many. D) None is correct E) No correction
A) I & II B) II & III C) None is correct required
D) Only I E) No correction required
318. I. The bravado of instruction to BJP leaders that
315. I. India called off Sri Lanka to fulfill the they should file expenditure statements for the
aspirations of the island nation‟s Tamil minority by period Nov 8 to Dec 31, 2016, camouflage the
taking forward the process of reconciliation and question of whether there was selective advance
implementing a constitutional provision aimed at leakage of information of demonetisation. II.
the devolution of powers. II. As a Delhi went to the Promises made during elections are to be backed up
polls, polling locations in most unauthorized by performance, and results of good governance
colonies in the city were abuzz with conversations should gradually became visible to the public. III.
suggesting how development and welfare schemes Scientists have developed a technique to help
focused in such colonies. III. It would be useful to prevent robots from overheating using an
remind oneself that the rights which citizens cherish innovative liquid cooling system that takes
deeply is fundamental – it is not the restrictions that inspiration from the human body.
are fundamental. A) Only I B) I & II C) None is correct
A) I & II B) Only II C) I & III D) D) II & III E) No correction
None is correct E) No correction required required

316. I. The impact on the stock of unaccounted 319. I. The perception that state assembly elections
money will be known only after a few months, when from time distract the centre from good governance
the holders of unaccounted money will have to treat is anathema to the concept of democracy and
the withdrawn notes as waste paper. II. The federalism and is disrespectful to the electorate. II.
Supreme Court directed that Cinema halls over the It‟s the Supreme Court which has often protected
country must play the National Anthem before the and uphold the rights and liberties of the individual
screening of a film and people should stand up as a and the minority against attempts by the state to
mark of respect. III. To listen to someone without encroach on them, often in the name of the
reacting means we accept him and do not create majority‟s mandate. III. The technique allows
even a single judgmental thought in our mind. machines to “sweat” off cooling liquid stored around
A) II & III B) I & II C) None is correct actuators, the component responsible for moving
D) Only II E) No correction required and controlling a mechanism or system.
A) II & III B) I & III C) Only I D)
317. I. The various economic think tanks which Only II E) No correction required
depend on government funds to operate are said to
Use Code: (AVP10) for 10% India’s No.1 Teacher in Bank Exams for English Language and Editorial 29
additional discount on All Website: www.vishalparihar.com | Follow on Instagram: @vishalthetrainer
Unacademy courses
1000+ Error Detection for All Exams
Last 20 years (Previous Year Papers)
English By Vishal Parihar

320. I. Even as the country mourns the young 323. I. Article 25 is an article of faith in the
soldiers who laid down their lives in the line of duty, Constitution, incorporated in recognition of the
citizens need to push their elected leaders to principle that the real test of a true democracy is the
demand answers, for there is no other way to ensure ability of even an insignificant minority to find its
more young lives are not lost in vain. II. If we can identity under the country‟s constitution. II.
work out where healthy cells normally lived and Assurance that rights are secure tends to diminish
what makes them expand when someone stops fear and jealousy of strong government, and by
smoking, perhaps we have opportunities to make make us feel safe to live under it makes for its better
them even more effectively at repair. III. While sport support. III. A latest study found anxiety about
is riddled with many ills, from corruption to unfair appearing “rude” or “ungrateful” was stopping
practices, administrators worldwide need to women from requesting more money – with around
acknowledge child sex abuse as the gravest. a quarter of woman saying they were fearful that
A) I & II B) Only II C) I & III D) contesting pay could endanger benefits like
None is correct E) No correction required maternity leave or flexible working.
A) II & III B) None is correct C) I & III
321. I. NABARD, Cooperatives and RRBs need to take D) Only I E) No correction required
the lead in or organising nation-wide
training/demonstration camps for farmers to 324. I. The aggression expansion of Chinese
familiarize them with digital banking. II. In past smartphone-makers in India has helped the country
weeks, as violence both on the LoC and inside dethrone the US to become the second largest
Kashmit has escalated, New Delhi appeared to be smartphone market in the world. II. The Centre
flailing, uncertain of what its next steps ought to be. offered to exiting Air India and Air India Express by
III. Many people care greatly about their reputation selling its entire stake, instead of the 76% it had
and how they will be judged by others, and a offered two years ago, apart from the 50% it owns
concern about appearing honest may outweigh our in groundhandling joint venture AI-SATS. III. The
desire to actually be honest, even in situations coronavirus outbreak has stoked a waves of anti-
where it will cost us money to lie. China sentiment around the globe, from shops
A) Only III B) I & III C) Only I D) barring entry to Chinese tourists, online vitriol
None is correct E) No correction required mocking the country‟s exotic mean trade and
surprise health checks on foreign workers.
322. I. The great American tragedy happened when A) II & III B) None is correct C) Only II
US bombs, rained on Baghdad, glowing like D) I & II E) No correction required
maliciously fireflies on TV screens, CNN bringing
humanity‟s first televised war, palpable excitement 325. I. The Enforcement Directorate is probing a
ruffling its correspondents‟ immaculate scarves. II. series of financial transactions involving the PFI as
The Supreme Court order for playing national part of a widening probe into the alleged role of the
anthem in cinema halls has touched on an old outfit in fanning protests against the CAA. II. With
debate on whether forcing someone to sing the the European Parliament due to discuss a resolution
anthem infringes fundamental rights. III. When sharply critical of the CAA, India is ramping up
people obtain extremely favourable outcomes, they diplomacy in Brussels to counter the move by
anticipate other people‟s suspicious reactions, and hundreds of MEPs. III. Donald Trump‟s defence
prefer lying and appearing honest over telling the team made the audacious argument that anything
truth, and appearing as selfish liars. the US president does to get re-elected is legal if he
A) II & III B) Only II C) None is correct believes it is in public interest.
D) I & II E) No correction required A) Only I B) I & III C) None is correct
D) II & III E) No correction required
Use Code: (AVP10) for 10% India’s No.1 Teacher in Bank Exams for English Language and Editorial 30
additional discount on All Website: www.vishalparihar.com | Follow on Instagram: @vishalthetrainer
Unacademy courses
1000+ Error Detection for All Exams
Last 20 years (Previous Year Papers)
English By Vishal Parihar

A) Only I B) I & II C) Only II D)


326. I. In general, those who are religious are more II & III E) No correction required
inclined to believe in end of the world theories,
citing unspecified versions of „judgment day‟ that 329. I. While businesses has been complaining of
have no scientific basis. II. It is in terms of modest problems with the platforms for several months, it is
lethality and human ability to react quick, some of only now that the government has begun
the dynamic with regards to spread of coronavirus acknowledging the concerns. II. While Unilever has
may be different with the increasing migration and initiated a strategic review of its global tea business,
mobility of people and products, which explains the Industrial analysts does not expect the Indian
massive quarantines underway in China. III. Chinese subsidiary Hindustan Unilever to evaluate selling its
authorities have said the virus emerged from a tea business in India. III. The Narendra Modi
market selling illegally traded wildlife, giving rise to government battling an acute economic slowdown,
widespread social media mocking of China‟s has forcing to announce a series of steps to
demand for exotic delicacies and ingredients for accelerate activity.
traditional medicine. A) II & III B) None is correct A) II & III B) None is correct C) Only I
C) Only I D) I & II E) No D) I & II E) No correction required.
correction required

327. I. While Bajaj had already taken a back seat


when it came to taking active business decisions Read each of the following sentences to find out if
years ago, this would be the first instant when he there is any grammatical error in it. The error, if
officially gets into a nonexecutive position II. Both any, will be in one part of the sentence. The
Bengal or Delhi fancied their chances of collecting number (A, B, C or D) of this part is your answer. If
three points, but in the end it was the weather which there is no error in the statement, then mark
had the final say as the teams settled for a point option (E) as your answer choice.
each from their drawn Ranji Trophy Elite „A‟ match
at the Eden Gardens. III. US authorities announced 330. The initial trends has shown (A)/that AAP is set
the discovery of the longest smuggling tunnel ever to repeat for (B)/the third time with Chief
founded on the southwest border, stretching more (C)/Minister Arvind Kejriwal heading the
than three quarters of a mile from now to industrial government. (D)
site in Tijuana, Mexico, to the San Diego area. A) B B) A C) C D)
A) I & III B) Only II C) None is correct D E) No correction required
D) I & II E) No correction required
331. The World Health Organization convened
328. I. Unlike in India, where tea drinking is well outside experts to fast-track promising tests,(A)/
entrenched, in the developed markets, black tea is drugs and vaccines to helping slow the outbreak of
seeing a decline as consumers are said to be moving a new (B)/virus that emerged in China that has killed
to other alternatives. II. The iconic “Taj Mahal” tea more (C)/ than 1,000 people and spread to two
under Brooke Bond, which is marketed as a brand dozen other countries. (D)
closely linked to the heritage culture, while Lipton A) A B) C C) B D)
focus to emerging consumer tastes such as green D E) No correction required
tea. III. The finance ministry has summoned senior
Infosys executives to find out why the technology 332. One of the four death row convict in (A)/ the
major has be unable to fix glitches in the GST Nirbhaya gang rape and murder case, Vinay
Network system, which is causing hardship to lakhs Sharma,(B)/ approached the Supreme Court on
of taxpayers. Tuesday challenging (C)/ the rejection of his mercy
Use Code: (AVP10) for 10% India’s No.1 Teacher in Bank Exams for English Language and Editorial 31
additional discount on All Website: www.vishalparihar.com | Follow on Instagram: @vishalthetrainer
Unacademy courses
1000+ Error Detection for All Exams
Last 20 years (Previous Year Papers)
English By Vishal Parihar

petition by the President. (D) A) D B) 339. The trademark owners should be given (A)/ the
A C) B D) C E) option to register with an e-commerce platform (B)/
No correction required so that wherever a trademark product is uploaded
for sale on the (C)/ platform, it should notified the
333. The BBC did help at least (A)/ four generations respective trademark owner. (D)
of Indians to (B)/ meet their goals, improve their (C)/ A) D B) C C) B D)
live, and find their place in the world. (D) A E) No correction required
A) C B) D C) B D)
A E) No correction required 340. Unlike the problem of (A)/ physical piracy and
counterfeiting, (B)/ the piracy of content (C)/ is easy
334. Online copyright theft is a criminal and damaging. (D)
(A)/enterprise which also affect consumers (B)/, and A) B B) D C) A D)
is susceptible to (C)/ malware, identity theft and C E) No correction required
ransomware. (D)
A) A B) B C) C D) 341. With the ruling BJP issuing a whip to their
D E) No correction required members in both (A)/ houses of Parliament, there is
a huge buzz on social (B)/ media that a bill on the
335. Former IAF chief B S Dhanoa warn (A)/that the controversial (C)/ Uniform Civil Code (UCC) will be
Indian government could hit (B)/cross-border introduced on Tuesday. (D)
terrorists harder if they launched a (C)/strike similar A) C B) A C) D D)
to the one in Pulwama on February 14, 2019. (D) B E) No correction required
A) A B) D C) C D)
B E) No correction required 342. A statement from the White House said that
the (A)/ trip will ‗further strengthen the United
336. Millions of people across the (A)/ globe come States-India strategic (B)/ partnership and highlight
together to bring about positive (B)/ changes online, the strong and (C)/ enduring bonds at the American
raise awareness (C)/ and took part in events and and Indian people‘. (D) A) C B) B
activities. (D) C) D D) A E) No
A) B B) D C) C D) correction required
A E) No correction required
343. The woman alleged that Chopra
337. The fact is that the state of J&K would not (A)/ commercialized the entire issue (A)/ of Kashmiri
have come into existence had it not been for (B)/ the Pandit exodus of the 1990s and did not portrayed
formidable combination of diplomacy and valour (B)/ the actual suffering of the community including
shown (C)/ by the Dogras under Maharaja Gulab the genocide, (C)/ mass rapes and mass murders
Singh (1792- 1858). (D) committed by Islamic radical groups in his film. (D)
A) A B) C C) D D) A) D B) A C) B D)
B E) No correction required C E) No correction required

338. While technology offers several solutions to 344. Chennai is the second-worst record of vehicular
(A)/ authenticates the original product, the same pollution in India (A)/ after Delhi, generating 3,200
(B)/ technological tools, particularly artificial (C)/ tonnes of carbon dioxide (B)/ (CO2) per day, a
intelligence, help create clones. (D) primary greenhouse gas, (C)/ according to Centre for
A) A B) B C) C D) Science and Environment (CSE). (D)
D E) No correction required A) A B) B C) C D)
D E) No correction required
Use Code: (AVP10) for 10% India’s No.1 Teacher in Bank Exams for English Language and Editorial 32
additional discount on All Website: www.vishalparihar.com | Follow on Instagram: @vishalthetrainer
Unacademy courses
1000+ Error Detection for All Exams
Last 20 years (Previous Year Papers)
English By Vishal Parihar

350. The scheme was conceptualize with an aim to


345. As the Centre announced that singer Adnan identify the most (A)/ promising startups via a
Sami, who was born in (A)/ Pakistan but later took competitive format, (B)/ reward them and create an
Indian citizenship, has been awarding (B)/ Padma assured path for success by (C)/ eliminating hurdles
Shri this year, Union Aviation Minister Hardeep and challenges typically faced by most founders. (D)
Singh Puri said that he hoped (C)/ protestors at A) A B) B C) D D)
Shaheen Bagh were "listening". (D) C E) No correction required
A) A B) D C) C D)
B E) No correction required 351. A single page in a normal text book will (A)/
amounts to 4 pages in Braille script (B)/ and the
346. A leader from Pakistan Prime Minister Imran advanced machine is fed (C)/ with Baden Software
Khan's party, (A)/ who put up banners with a slogan to print the books. (D)
offensive to the minority (B)/ Hindus in the country, A) A B) C C) D D)
have apologised (C)/ after he came under fire from B E) No correction required
netizens as well as the party. (D)
A) C B) D C) B D) 352. The Press, functioning under Directorate for
A E) No correction required the Welfare of Disabled (A)/ and Senior Citizens,
prints and distributes Braille books (B)/ on general
347. Despite of that the fires from the CAA and NRC knowledge, music, (C)/ Kannada and English
are still burning (A)/ across the country, the BJP is grammar apart of a calendar. (D) A) B B) C C) D D) A
choosing to become (B)/ more combative by E) No correction required
pushing through another contentious (C)/ issue
which is a sensitive issue for the minority 353. On the overall coal scenario in the country, (A)/
community. (D) the production this year will exceeding that of last
A) A B) C C) B D) year, (B)/ though there was some impact due to
D E) No correction required rains, (C)/ by the end of the current fiscal it will be
covered up. (D)
348. A three-judge bench, headed by Justice R A) A B) B C) C D)
Banumathi, comprises justices (A)/ Ashok Bhushan D E) No correction required
and A S Bopanna, said the pendency of the appeal
filed by the (B)/ Centre and the Delhi government 354. Chief Minister Uddhav Thackeray has pledging
before it would not be an impediment (D)/ for the to the (A)/ public that the State government (B)/ will
trial court in issuing fresh date for execution of the prove the crime of the accused as soon (C)/ as
convicts. (D) possible and the accused will be hanged soon. (D)
A) A B) B C) D D) A) A B) D C) C D)
C E) No correction required B E) No correction required

349. Amid the lockdown and travel bans triggered 355. The Opposition Congress demanding that the
(A)/ by the 2019 novel corona virus outbreak in Union Government either (B)/ file a review petition
China, (B)/ fears have been expressed about their in the apex court or amend the (C)/ Constitution to
impact on the (C)/ health of the global economy as make reservation a fundamental right. (D)
a whole. (D) A) A B) D C) B D)
A) A B) B C) C D) C E) No correction required
D E) No correction required

Use Code: (AVP10) for 10% India’s No.1 Teacher in Bank Exams for English Language and Editorial 33
additional discount on All Website: www.vishalparihar.com | Follow on Instagram: @vishalthetrainer
Unacademy courses
1000+ Error Detection for All Exams
Last 20 years (Previous Year Papers)
English By Vishal Parihar

356. This is a constitutional matter that (A)/ should went (C)/out of their way to make me comfortable.
have heard by the full (B)/ Constitution Bench and (D)
not just a (C)/ two-judge Bench of the apex court. (D) A) C B) D C) A D)
A) D B) C C) A D) B E) No correction required
B E) No correction required
363. There is an urgent need to galvanise the (A)/
357. While the quotas have been politicised over the concerned departments into action before the price
years, (A)/ the apex court has merely tried to make (B)/ for inaction became too much to bear and (C)/
them practical (B)/, choosing to separated takes the form of a public health emergency. (D)
educational quotas from (C)/ jobs, where A) A B) C C) D D)
efficiencies depend on merit and performance. (D) B E) No correction required
A) D B) C C) A D)
B E) No correction required 364. The ―Health at your Doorstep‖ initiative of the
Government will (A)/ go a long way in restoring (B)/
358. A Mumbai court had on February 5 rejects the faith of the people and help it (C)/ understand the
anticipatory (A)/ bail application filed by Chudawala, importance of health parameters. (D)
who was recently booked (B)/ in a sedition case for A) A B) B C) C D)
allegedly raising "anti-national (C)/ slogans" in D E) No correction required
support of JNU student Sharjeel Imam. (D)
A) B B) C C) D D) 365. Once a corporation know its value — which can
A E) No correction required be (A)/ obtained through a reputed valuation service
— it (B)/ can negotiate with the buyer and can reject
359. The 24-year-old woman lecturer succumbed to the (C)/ deal, if the offer made is less than what is
(A)/ injuries a week after he was set (B)/ ablaze by a required. (D)
jilted lover near the Hinganghat area in (C)/ A) D B) B C) C D)
Maharashtra's Wardha district, on February 3. (D) A E) No correction required
A) B B) D C) A D)
C E) No correction required 366. Many corporations measure economic value
creation for (A)/ the company as a whole,
360. Once Hanumanji has decided that the dal fry, periodically, using different (B)/ technique to
(A)/ which was destined for you is ready, (B)/ he will measure performance against planned (C)/ targets
pour it to onto a small stainless steel bowl – filled and suitably reward managers on achieving the
(C)/ to the brim and a bit more. (D) same. (D)
A) A B) C C) D D) A) B B) C C) D D)
B E) No correction required A E) No correction required

361. Both DDT and new income (A)/ tax proposals 367. India has asked Vietnam to open its
will not (B)/ have any adverse impact (C)/ from the pharmaceutical (A)/ market for the Indian generic
mutual fund industry (D) drugs and sought better market (B)/ access for
A) A B) D C) C D) several other products during (C)/ the delegation
B E) No correction required level talks between India or Vietnam. (D)
A) A B) D C) C D)
362. Being a voracious reader, she sat down for B E) No correction required
studying (A)/ whenever she got time but it was her
family (B)/ which helped her stay focused as they 368. Liquor baron Vijay Mallya arrived at the Royal
Courts (A)/ of Justice in London of (B)/ his appeal
Use Code: (AVP10) for 10% India’s No.1 Teacher in Bank Exams for English Language and Editorial 34
additional discount on All Website: www.vishalparihar.com | Follow on Instagram: @vishalthetrainer
Unacademy courses
1000+ Error Detection for All Exams
Last 20 years (Previous Year Papers)
English By Vishal Parihar

against being extradited to India to face (C)/ fraud A) D B) C C) B D)


and money laundering charges amounting to Rs A E) No correction required
9,000 crores. (D)
A) A B) D C) C D) 374. Market benchmark Sensex rallied over 300
B E) No correction required points (A)/ in opening session on Wednesday driven
by (B)/ gains in HDFC twins, RIL, ICICI Bank and HUL
369. In any election campaign, the media (A)/ plays ahead (C)/ of the release of inflation and factory
an important role in dissemination (B)/ of output data. (D)
information and, thus, enabled the (C)/ people to A) A B) B C) C D)
make a well-informed choice. (D) D E) No correction required
A) B B) C C) D D)
A E) No correction required 375. The BJP on Monday staunchly defended its
decision to conferring (A)/ Padma award to singer
370. While we do not have concrete data on Adnan Sami by claiming (B)/ that Sonia Gandhi too
complications and deaths (A)/ caused by should be subject of (C)/ discussion if everyone has
malfunctioning equipment, it is not (B)/ hard to to be judged by his or her parents‘ deeds. (D)
imagine a scenario where treatment are prescribed A) A B) D C) C D)
based (C)/ on faulty readings, leading to catastrophic B E) No correction required
consequences. (D)
A) A B) B C) C D) 376. Prime Minister Narendra Modi hit out at the
D E) No correction required Congress in (A)/ Lok Sabha over its opposition to the
CAA and asked if (B)/ first premier Jawaharlal Nehru
371. At present, the country imports over 80 per could be describe as "communal" (C)/ for seeking
cent of its (A)/ medical devices and the Government citizenship for religious minorities from Pakistan. (D)
have made its (B)/ intention clear to make India one A) D B) B C) C D)
of the global (C)/ hubs for medical devices, both in A E) No correction required
manufacturing and distribution/ (D)
A) C B) D C) A D) 377. The verdict of putting Hafiz Saeed in jail for 7
B E) No correction required years came (A)/ four days before the Financial
Action Task Force‘s (FATF) plenary meeting in Paris
372. Ahead of US President Donald Trump‘s maiden to (B)/ assess action took by Pakistan to (C)/ counter
visit from India, the (A)/ two countries are eyeing to terror financing and money laundering. (D)
finalise a raft of mega defence deals (B)/ including A) A B) B C) D D)
procurement of a batch of military helicopters by C E) No correction required
(C)/ Indian Navy from American defence major
Lockheed Martin at a cost of USD 2.6 billion. (D) 378. The Hummer, which was first marketed in 1992
A) A B) B C) C D) and then pulled out (A)/ in the market in 2008
D E) No correction required following questions over (B)/ its viability after the
economic slowdown, is making a (C)/ comeback,
373. The requirement comes as governments albeit with some 'thunderous' modifications. (D)
around the world (A)/ are trying to hold social media A) A B) B C) C D)
companies more accountable for (B)/ the content D E) No correction required
that circulates on their platforms, weather its (C)/
fake news, child porn, racist invective or terrorism-
related content. (D) Read each sentence to find out whether there is
any grammatical error or idiomatic error in it. The
Use Code: (AVP10) for 10% India’s No.1 Teacher in Bank Exams for English Language and Editorial 35
additional discount on All Website: www.vishalparihar.com | Follow on Instagram: @vishalthetrainer
Unacademy courses
1000+ Error Detection for All Exams
Last 20 years (Previous Year Papers)
English By Vishal Parihar

error if any, will be in one part of the sentence. The 388. The evening breeze1)/ won’t carrying the
number of that part is the answer. If there is “No poetry 2)/ of peace beyond 3)/ the school building .
error” the answer is ‘5’.(Ignore errors of 4)/ No error 5).
punctuation if any.)
389. We are bring in the idea that 1)/ European
379. The couple’s work in 1)/ upgrading rural rehabilitation focussed 2)/ on a multi-disciplinary
technicians 2)/ has set a benchmarking 3)/ for future approach 3)/ towards chronic pain. 4)/ No error 5).
generations. 4)/ No errors 5)
390. If parents are able to 1)/ get their children into
380. It has taking almost 1)/ a year for India 2)/ to let schools 2)/ that are far away, the 3)/ next challenge
its pessimism 3)/ translate into fewer jobs. 4)/ No is transportation. 4)/ No error 5).
errors 5)
391. The perception of others 1)/ particularly family
381. The city needs an airport 1)/ that can efficiently members 2)/ changed when he 3)/ qualified on a
manage 2)/ a constantly flow of 3)/ passengers and government job. 4)/ No error 5).
flights. 4)/ No errors 5)
392. Every house should 1)/ have the device as it 2)/
382. This group of 1)/ rural achievers is very 2)/ protects people from a 3)/ common household
different than the 3)/ ones in the past. 4)/ No error disaster . 4)/ No error 5).
5).
393. Automated baggage handling systems are 1)/
383. The government has announced 1)/ plans to ensuring that on the time passengers 2)/ are out of
creating 2)/ one million new 3)/training places. 4)/ the plane their 3)/ baggage is already waiting for
No error 5). them . 4)/ No error 5).

384. The argument assumes that 1)/early detection Read each sentence to find out whether there is any
of the disease 2)/will lead to an immediate drop in grammatical mistake error in it. The error if any, will
3)/ the mortality rating from this disease. 4)/ No be in one part of the sentence. Mark the number of
error 5). that part with error as your answer.If there is”No
error”, mark 5.
385. The two most important numbers 1)/ which the
mandarins of an 2)/ economy have to watch 3)/ are 394. 1) There cannot be any situation where / 2)
inflation and unemployment. 4)/ No error 5). somebody makes money in an asset / 3) located in
India and does not pay tax / 4)either to India or to
386.Wintnessed the young soldier’s ability 1)/ to the country of his origin. / 5) No error .
repeatedly hit bull’s eye at 2)/ arms
training,instructors pushed him 3)/ to participate in 395. 1) India has entered a downward spiral / 2)
the Army marksmanship competition. 4)/ No error Where the organized productive / 3) and law abide
5). sectors are subject to / 4) savage amounts of
multiple taxes. / 5) No error.
387. It is all well known that 1)/ women are generally
in favour of 2)/ light topics like jokes and expressions 396. 1) The bank may have followed / 2) an
3)/ that causing laughter all around . 4)/ No error 5). aggressive monetary tightening policy / 3) but its
stated aim of / 4) curbing inflation have not been
achieved. / 5) No error.

Use Code: (AVP10) for 10% India’s No.1 Teacher in Bank Exams for English Language and Editorial 36
additional discount on All Website: www.vishalparihar.com | Follow on Instagram: @vishalthetrainer
Unacademy courses
1000+ Error Detection for All Exams
Last 20 years (Previous Year Papers)
English By Vishal Parihar

397. 1) Equal opportunities for advancement / 2)


across the length and breadth / 3) of an organization 405. The power tariff had already/been increased
will / 4) keep many problems away . / 5) No error. twice in/ the last 15 months and the Electricity
Board had also / 1) 2) 3) leived additional monthly
398. 1) A customosed data science degree / 2) is yet charges to consumers. / No errors. 4) 5)
to become / 3) a standard programme / 4) to India’s
premier educational institutes. / 5) No error. 406. Despite of curfew / in some areas, minor /
communal incidents were reported/ from different
Read each sentence to find out whether there is any areas of the 1) 2) 3) 4) walled city. / No error 5)
grammatical error in it. The error, if any, will be in
one part of the sentence. The number of that part is 407. This comes / at a time/ when fund allocation/is
the answer. If there is no error, the answer is 5), ie been doubled. / No error 1) 2) 3) 4) 5)
‘No error’.(Ignore errors of punctuation if any.)
408. As the prison will get / an official telephone
399. These companies have been asked 1)/ to facility soon, the prisoners/won’t have to make calls
furnish their financial details 2)/ and information in discreet 1) 2) 3) manner/ through smuggled
about 3)/ its board members. 4)/ No error 5). mobile phones . / No error 4) 5)

400. The scheme which will be launched 1)/ during 409. The area was plunged into / darkness mid a
the next two years 2)/ require an additional wave of / cheering and shouting / slogans like ‘Save
investment 3)/ of one hundred crore . 4)/ No error The Earth’. / 1) 2) 3) 4) No error 5)
5).
410. The poll contestants approached / the
401. Road developers unable 1)/ to complete their commission / complaining that the hoardings /
projects 2)/ on time will not be 3)/ awarded new violated the code of 1) 2) 3) conduct / and influenced
ones . 4)/ No error 5). public perception. / No error 4) 5)

402. We have taken on 1)/ the responsibility of 2)/ 411. The country has / adequate laws but problems
arranging the required training 3)/and supervise the / arise/ when these are not / implemented in letter
new staff. 4)/ No error 5). and spirit. / 1) 2) 3) 4) No error 5)

403. The government has signed 1)/ a memorandum 412. The Management feels that / the employees of
of understanding with 2)/ the company to set up 3)/ the organization are / non – productive, and do not
a plant in the state . 4)/ No error 5). want / to 1) 2) 3) work hard. / No error 4) 5)

Read each sentence to find out whether there is any 413. As far the issue of land encroachment / in
grammatical error or idiomatic error in it. The error villages is concerned, people will / have to make a
if any, will be in one part of the sentence. The start from their 1) 2) 3) villages by / sensitizing and
number of that part is the answer. If there is “No educating the villagers about this issue . / No error
error” the answer is 5) . (Ignore errors of 4) 5)
punctuation if any.)
Read each sentence to find out whether there is any
404. The Government has asked individuals/ with grammatical error or idiomatic error in it. The error
income of over Rs 10 lakhs to / electronic file tax if any, will be in one part of the sentence. The
returns for 1) 2) 3) the year 2011-12, / something number of that part is the answer. If there is ‘No
which was optional till last year./ No error 4) 5)
Use Code: (AVP10) for 10% India’s No.1 Teacher in Bank Exams for English Language and Editorial 37
additional discount on All Website: www.vishalparihar.com | Follow on Instagram: @vishalthetrainer
Unacademy courses
1000+ Error Detection for All Exams
Last 20 years (Previous Year Papers)
English By Vishal Parihar

error’ the answer is 5. (Ignore errors of punctuation


if any.) 424. 1) Farmers who grow the crop/ 2) are the worse
hit,as the investment / 3) they make on a crop is / 4)
414. 1) The transit system’s underground / 2) huge as compared to the profit. / 5) No error.
tunnels and stations will / 3) be constructed / 4) next
heritage structures. / 5) No error. 425. 1) The government is made it / 2) mandatory
for pharmacies to send / 3) copies of medical bills
415. 1) Residents have been planting / 2) the along with / 4) prescriptions of drugs and details of
ornamental trees outside / 3) their homes and in patients. / 5) No error.
lawns to/ 4) add beauty and give their place a grand
look. / 5) No error. 426. 1) While these schools have/ 2) a strong
traditional hold, the reason for / 3) forming a
416. 1) A diamond jeweller’s peon/ 2) tipped off a consortium in India is to encourage / 4) cultural
gang / 3) about the gold / 4) in his employer’s vault. diversity for their institutes. / 5) No error.
/ 5) No error.
427. 1) The Government soon will make / 2) it
417. 1) He said that the performance of the Indian mandatory for all private hospitals / 3) to notify
team / 2) was satisfactory at the international meet infectious cases/ 4) as soon as they are detected. /
/ 3) and that they learnt a lot from / 4) watching top 5) No error.
seeded players through action at the grand slam. /
5) No error. 428. 1) School children, who so far / 2) have had
rationed access to / 3) the few playgrounds on the
418. 1) Watching the exponential / 2) talent of world city must be / 4) happy by the recent move. / 5) No
tennis / 3) was the best things/ 4) to happen to him. error.
/ 5) No error.

419. 1) The meeting took place a day after/ 2) the Read each sentence to find out whether there is
agency held a meeting with project contractors/ 3) any grammatical error or idiomatic error in it. The
to evaluate steps that being taken to / 4) ensure that error if any, will be in one part of the sentence. The
the buildings were not affected. / 5) No error. number of that part is the answer. If there is “No
error” the answer is 5. (Ignore errors of
420. 1) The director explained/ 2) the theme and / punctuation if any.)
3) the concept of / 4) the international folk festival.
/ 5) No error. 429. Despite the merger, 1)/ the airline has reported
2)/ sizeable losses and is unable 3) / to pay their
421. 1) The fight among/ 2) rival candidates between employees 4) / No Error 5)
the medium / 3) of catchy slogans/ 4) has started. /
5) No error. 430. We have recently began 1) / the process of
recruiting 2)/ insurance agents for 3)/ our North –
422. 1) Acting on a tip-off / 2) the anti-robbery squad Eastern branches 4) / No Error 5)
led / 3) by inspectors laid / 4) a trap for the robbers.
/ 5) No error. 431. The company’s decision to issue 1) / bonus
shares clearly indicates 2)/ the management
423. 1) The process of / 2) revising figures of damage confident 3)/ about the future 4) / No Error 5)
/ 3) to get additional compensation / 4) has began. /
5) No error.
Use Code: (AVP10) for 10% India’s No.1 Teacher in Bank Exams for English Language and Editorial 38
additional discount on All Website: www.vishalparihar.com | Follow on Instagram: @vishalthetrainer
Unacademy courses
1000+ Error Detection for All Exams
Last 20 years (Previous Year Papers)
English By Vishal Parihar

432. Inspite that organizations are aware 1) / of the 441. His comments came after 1) / the research
importance of IT 2)/ they often do not know 3) / how group said that its 2)/ consumer confidence index
to delay it effectively 4) / No Error 5) were 3) / slumped to its lowest level 4)/ No Error (5)

433. Keeping in mind that India 1) / is one of the 442. If all goes well 1)/ the examination scheduled
world’s fastest 2)/ growing economies many for next month 2) / is all set to be completely free 3)
international 3)/companies are invested in India 4) / / from annoying power cuts and disruptions 4)/ No
No Error 5) Error (5)

434. Although interlinking of rivers will 1)/ help 443. There are just too few trains 1)/ for the ever –
tackle drought, the government 2)/ has not paid grow 2)/ number of passengers 3) / in the city 4)/ No
much 3) / attention to this project 4) / No Error 5) Error (5)

435. The rise in prices of food items is 1) / likely the 444. The buzz at the party was 1) / that a famous 2)/
cause of inflation 2) / to exceed eight per cent 3)/ film star and politician would 3) / probable drop by
during the next quarter 4) / No Error 5) for a while. 4) / No Error (5)

436. Many people believe that 1) / the 445. The Opposition disrupted proceedings 1)/ in
nationalization of banks in 2) / the 1960s have both Houses of Parliament 2)/ for the second
protected India 3) / from the current financial crisis consecutive day 3)/ above the plight of farmers in
4)/ No Error 5) the country 4)/ No Error (5)

437. RBI is in favour 1)/ of amending currently 2)/ 446. In response to growing crisis 1)/ the agency is
policies governing operations of 3)/ foreign banks in urgently asking for 2) / more contributions, to make
India 4). / No Error 5) up for 3)/ its sharp decline in purchasing power 4)/
No Error (5)
438. These recently upgraded branch offices 1)/
have been fully computerized 2)/ and are connected 447. The tennis player easy through/ the opening set
to headquarters 3) / to ensure proper monitoring 4)/ before her opponent / railed to take the final two
No Error 5) sets/ for the biggest victory of her young career. /
No Error (5)
Read each sentence to find out whether there is
any grammatical error or idiomatic error in it. The 448. Aggression in some teenage boys 1)/ may be
error if any, will be in one part of the sentence. The linkage to overly 2)/large glands in their brains, 3)/ a
number of that part is the answer. If there is “No new study has found. 4)/ No Error (5)
error” the answer is 5. (Ignore errors of
punctuation if any.) Read each sentence to find out whether there is
any grammatical error or idiomatic error in it. The
439. The President has denied 1)/ that the economy error if any, will be in one part of the sentence. The
is in recession 2)/ or was go into one 3)/ despite a number of that part is the answer. If there is “No
spate of downcast reports 4)/ No Error (5) error” the answer is 5. (Ignore errors of
punctuation if any.)
440. The angry at being 1)/ left out of the bonanza
2)/ is palpable among 3)/employees of the 449. The biggest 1)/ health burden that India 2)/ is
organization 4)/ No Error (5) set to face in the coming years 3) / is tackle cancer
4) / No Error 5)
Use Code: (AVP10) for 10% India’s No.1 Teacher in Bank Exams for English Language and Editorial 39
additional discount on All Website: www.vishalparihar.com | Follow on Instagram: @vishalthetrainer
Unacademy courses
1000+ Error Detection for All Exams
Last 20 years (Previous Year Papers)
English By Vishal Parihar

Read each sentence to find out whether there is


450. Illegal sand mining in the peripheral areas 1) / any grammatical error in it. The error if any, will be
of the district continues to remain a big problem for in one part of the sentence. The number of that
2)/ forest –range officiers as yet another another part is the answer. If there is “No error” the answer
instance illegal 3)/ mining was reported yesterday 4) is 5. (Ignore errors of punctuation if any.)
/ No Error 5)
459. Our country can contribute 1)/ to the
451. The policemen, who 1)/ was deployed heavily mitigation of global warning 2)/ by protecting
2)/ in the area, did nothing to 3)/ dissuade the forests 3)/ and starting at afforestation programme
protesters 4) / No Error 5) 4)/No error 5)

452. Festivals are prime occasions 1)/ for splurging 460. The task for ensuring employment 1)/ for the
on presents and owing to improved economic labour force has been 2)/ a persistent concern
situation,2) / the youths is gung–ho 3)/ about throughout 3) / India’s post – independence
breaking all previous records. 4)/ No Error. 5) development4)/ No error 5)

453. It is important to recruit personnel at 1)/ 461. Psychologists have been documenting / the
different levels in the organization so that 2)/ the emotional and physical / effect of / negative political
ensuing human resource gap is bridged 3)/at least advertisements. No error 5)
for the critical operations 4) / No Error 5)
462. India’s efforts that are 1)/ aimed at controlling
454. Banks are on the verge 1)/ of facing a 2)/ infectious diseases is likely 3)/ to show only
formidable challenge 2)/ of losting over fifty per cent partial success4)/ No Error 5)
of 3)/ their employees due to retirement. 4)/ No
Error 5) 463. The magazine industry 1) / in India 2)/ has been
going from 3)/ confusing times 4)/No error 5)
455. Not only has the commerce ministry fixed /
extraordinarily high minimum prices for onion 464. The fast pace of progress on 1) / basic
exports / but also made licences mandatory / for education and literacy 2)/ is consistent with an
every consignment. / No Error 5) increase 3)/ in demand for education 4)/ No error 5)

456. A new study found that while weight loss 1)/ via 465. Technology is 1)/ transforming the way 2)/ films
surgery may improve knee pain, in obese patients are screen 3)/ in the theatres 4)/ No error 5)
2)/ there mat be permanent damage to the knee 3)/
from being severe overweight. 4)/ No Error 5) 466. One needs to set goals 1) / at differing stages
2)/ of one’s career and 3)/ monitor achievements
457. In order to streamline 1)/ the movement of and accomplishments 4)/ No error 5)
vehicles during 2)/ the festival, traffic police have 3)/
chalked out diversion plans 4) / No Error 5) 467. Non – communicable disease 1) / often require
2) / long – term and expensive interventions which
458. With a view to avoid another caste conflict,1) / 3) / poor people cannot afford 4)/No error 5)
the administration has deployed additional police
force 2)/ on the village, while the administration as 468. A leader is a group member which 1)/ exerts
well as police officials 3)/ are monitoring the profound influence 2)/ on the behaviour and
situation.4) / No Error 5) attitudes 3)/ of other members of the group 4)/No
error 5)
Use Code: (AVP10) for 10% India’s No.1 Teacher in Bank Exams for English Language and Editorial 40
additional discount on All Website: www.vishalparihar.com | Follow on Instagram: @vishalthetrainer
Unacademy courses
1000+ Error Detection for All Exams
Last 20 years (Previous Year Papers)
English By Vishal Parihar

Read each sentence to find out whether there is 477. The movement, which aims to 1)/ raise
any grammatical error or idiomatic error in it. The awareness about climate change, hopes 2)/ to bring
error if any, will be in one part of the sentence. The people together to think about 3)/ what they can do
number of that part is the answer. If there is “No for reduce harmful pollution. 4)/ No error 5).
error” the answer is 5. (Ignore errors of
punctuation if any.) 478. Workers battling to restore 1)/ power supply to
the damaged reactors 2)/ have successful in 3)/
469. The actor, who plays 1)/ a detective in the reaching up to the second reactor . 4)/ No error 5).
film,alleged to have pushed the director 2)/ when he
asked him to get into his costume 3)/ as it was Read each sentence to find out whether there is
getting late . 4)/ No error 5). any grammatical error in it or a wrong word has
been used. The error if any, will be in one part of
470. Manish was advised an operation 1)/ more than the sentence which has been numbered 1), 2), 3) or
thiry years back, but 2)/ due to his financial 4). The number of that part is the answer. If there
conditions he 3)/ could not undergoes the is “No error” the answer is 5), ie ‘No error’. (Ignore
procedure . 4)/ No error 5). errors of punctuation if any.)

471. India’s largest corporate house may be 1)/ force 479. The letter of offer 1)/ is being dispatched to 2)/
to review the selection parameters after 2)/ the share holders of the company 3)/ whom were
struggling to shortlist potential candidates 3)/ for allotted shares. 4)/ No error 5)
the past eight months . 4)/ No error 5).
480. The government policy does not 1)/ necessary
472. This questions force us 1)/ to face the have 2)/ to be 3)/ dictated by global policy changes.
unpleasant truth that 2)/ the polices of social justice 4)/ No error 5)
have 3)/ reached a dead end. 4)/ No error 5).
481. Nothing is perfect 1)/ and we might realize 2)/
473. Many of the young people 1)/ studying abroad that even when 3)/ we went wrong, the
agreed that 2)/ returning home was always 3)/ an consequences was not as bad. 4)/ No error 5)
attractive option. 4)/ No error 5).
482. When the astronomers first begin 1) / studying
474. The government needs to provide 1)/ specific the motion of galaxies,2) / they had no idea they had
regulations that create an environment 2)/ where 3)/ embarked upon a great discovery. 4) / No error
trained scientific personnel enjoyed space and 5)
freedom 3)/ to design new products benefiting
society. 4)/ No error 5). 483. The child 1) / was to 2)/ small to 3)/ reach the
switch. 4)/ No error 5)
475. Though the book is not yet available on India,
1)/ reviews published in this country suggest 2)/ that 484. The rate of metabolism of 1) / a body is
it is a complex study of 3)/ an extremely complex comparatively lowest when 2)/ it is at rest and is 3)/
personality . 4)/ No error 5). thus optimum for examination 4) / No error 5)

476. Conventionally speaking, 1)/ men are supposed 485. The opposition leader tried 1)/ to bolster his
to earn while 2)/ women are supposed to sit at position 2)/ with the voters by pressing
home, 3)/ do housework and bringing up the 3)/corruption charges against rivals 4)/ No error 5)
children . 4)/ No error 5).
Use Code: (AVP10) for 10% India’s No.1 Teacher in Bank Exams for English Language and Editorial 41
additional discount on All Website: www.vishalparihar.com | Follow on Instagram: @vishalthetrainer
Unacademy courses
1000+ Error Detection for All Exams
Last 20 years (Previous Year Papers)
English By Vishal Parihar

486. The recently imposed dress code 1)/ in the 496. Though he has promoted to 1) / the bank’s
university has enraged 2)/ the students who will be board as a director 2)/ he continues to carry out 3)/
going 3)/ on strike since tomorrow 4) / No error 5) all his current responsibilities. 4)/ No error 5)

487. Ever since he took over 1)/ as the chief minister 497. He had telephoned yesterday to 1)/ ask how
of the state, 2)/ rate for unemployment 3)/ has much of the youth 20/ who attend our classes would
drastically increased 4) / No error 5) be 3)/ interested in working for a textile company.
4)/ No error 5)
488. Although a brilliant writer,1) / an lying 2)/
pessimism prevails in 3)/ all her novels. 4)/ No Error 498. We were forced into react 1)/ as no
5) organization can 2)/ afford to adhere to 3)/ these
outdated regulation. 4)/ No error 5)
489. Changed social setting 1) / demands the schools
to teach 2)/ moral and social values among 3) /with 499. The government is working 1)/ out a new
the academic skills. 4)/ No error 5) system to compensate 2)/ those to sell 3)/ products
below the market price. 4) No error 5)
Read each sentence to find out whether there is
any grammatical mistake/error in it. The error if Read each sentence to find out whether there is
any, will be in one part with error as your answer. any grammatical mistake/error in it. The error, if
If there is “No error”, mark 5. any, will be in one part of the sentence. Mark the
number of the part with error as your answer. IF
490. We had extensively discussions 1)/ with the there is ‘No error’, mark 5).
participants and 2)/ obtained their feedback 3)/
regarding our new services. 4) / No error 5) 500. We are yet starting 1)/offering this facility to 2)
out customers as we are 3)/awaiting approval from
491. The success of the 1) / government sponsor job the Board 4)/No error 5)
guarantee programme 2)/ has resulted in a 3)/
drastic drop in poverty 4) / No error 5) 501. The Chairman of all large 1)/public sector banks
met with 2) senior RBI officials to give its
492. The Board’s decision has provided 1)/ 3)/suggestions about implementing the new policy
employees with the opportunity 2)/ to acquire upto 4)/No error 5)
100 3)/ shares by the company 4) / No error 5)
502. They have not fully considered 1)/the impact
493. Their failure to inspect 1)/ our factories is a 2)/ that relaxing 2)/these guidelines is likely 3)/the
clear indications that our 3)/ licence will not be transfer of funds 4)/No error 5)
renewed. 4)/ No error 5)
503. Had this notification1)/the impact that relaxing
494. Arranging such a large amount 1) / of funds now 2)/these guidelines is likely 3)/to have with
will be a problem why 2)/ banks are usually not open economy.4)/No error 5)
3) / so early in the morning. 4)/ No error 5)
504. There are many insurance 1)/disputes
495. If the manufacturing sectors continues / to nowadays because of 2)/most people do not fully
grow at the same rate for / the next few months,I 3)/understand the terms and conditions of their
think it / has a high growth rate this year. / No error policies.4)/No error5)
5)

Use Code: (AVP10) for 10% India’s No.1 Teacher in Bank Exams for English Language and Editorial 42
additional discount on All Website: www.vishalparihar.com | Follow on Instagram: @vishalthetrainer
Unacademy courses
1000+ Error Detection for All Exams
Last 20 years (Previous Year Papers)
English By Vishal Parihar

Read each sentence to find out whether there is 514.While providing such facilities 1) / online makes
any grammatical error or idioma tie error in it. The it convenient 2)/ and easily accessible for customers
number of that part is the answer. If there is no 3)/ we face several Challenges.4) / No error 5)
error, the answer is 5). (Ignore errors of
punctuations, if any.) Read each sentence to find out whether there is
any grammatical error in it. The error if any will be
505. Some genuine issues exist 1)/with the newly in one part of the sentence the number of that part
adopted 2)/ System and needs to 3)/ be examined will be the answer . If there is ‘No error’, mark(5) as
seriously. 4)/No error 5) the answer.(Ignore errors of punctuation, if any)

506. Whether or not to confront 1)/ them about 515.Angered over the he delay in giving
their role 2)/ in the matter is a decision 30/ which is compensation; 1)/ factory workers shouted 2)
yet to take. 4)/ No error 5) /slogans against the president 3)/ when he reaches
the office, 4)/No error 5)
507. The government is still in the 1) /process of
finalized new policy 2)/ guidelines for the allocation 516.The cascading effect of economic slowdown 1)/
of land 3)/ to private sector organizations. 4)/ No has brought a much unnerving gloom 2)/ to the real
error 5) estate industry last year 3)/ but the industry is
looking up this year 4)/ No error 5)
508.According to government estimates / at least
four millions tones of sugar / will have to be 517.A recycling plant in close proximity to 1)/ the
imported / this year because of a poor monsoon. / residential area can pose 2)/ serious threats from
No error 5) residents 3)/ by leaving behind persistent pollutants.
4)/ No error 5)
509.In our experience people usually 1)/ value
things that they have to 2)/pay off more than those 518.The government has the obligation 1)/ to
that 3)/ they receive free of cost. 4)/ No error 5) provide basic infrastructure facilities 2)/ to
regulating the process of 3)/urbanization in the
510.At present China is the 1)/ world’s leader country 4)./No error 5)
manufacture 2)/ of environment –friendly products
3)/ such as electric cars and bicycles. 4)/No error 5) 519.Bharatpur is transforming into 10/ India’s most
fastest growing bird sanctuary 2)/ attracting
511.Over percent from us 1)/ feel that if we had thousands of rare migratory birds 3) / from Europe
taken 2)/ Some corrective measures eighty 3) / the and Siberia 4)/No error 5)
crisis could have been averted 4)/ No error 5)
520.Plagued by huge losses, 1)/full service airlines
512.The Manager of that city branch 1)/ cannot are struggling 2)/ to cope with the competition 3)/
handle it with the help of 2)/ only two personal as from low cost airlines 4) /No error. 5)
3) /business has Increased substantially. 4)/ No error
5) 521.Volunteers of an NGO1) / interacted with school
students 2) /to spread awareness about 3)/
513. With the literary rates in this 1)/ region as low Environment related issues 4)/ No error 5)
as ten percent 2)/we need to encourage 3) / local
people to build schools. 4)/ No error 5) 522.The Union health minister said that 1)/ there
was an acute shortage of 2)/ health personnel in

Use Code: (AVP10) for 10% India’s No.1 Teacher in Bank Exams for English Language and Editorial 43
additional discount on All Website: www.vishalparihar.com | Follow on Instagram: @vishalthetrainer
Unacademy courses
1000+ Error Detection for All Exams
Last 20 years (Previous Year Papers)
English By Vishal Parihar

rural areas 3)/ who needed to be addressed urgently 531. Everyone is keen in knowing 1)/ the forecast for
4)/ No error 5) the monsoon 2)/ this year as it has been the 3)/
major cause of inflation in Asia. 4)/No error 5)
523.Even though most of India’s youth 1)/ lives in
villages, many people considers 2)/ the ones 532. Building biogas plants will help to reduce 1)/
roaming in dazzling multiplexes 3)/ as their true greenhouse gas emissions by reducing 2)/ the
reflection. 4)/No error 5) consuming of con2ventional fuels 3)/ such as
firewood and kerosene 4)/No error 5)
524. The RBI has proposed to introduce 1)/ polymer
notes after taking into considering 2)/ the cost and 533. Since some banks may take longer 1)/ to
Longevity 3)/associated with their manufacturing achieve these targets, RBI 2)/ has considered to
4)/ No error 5) revise these guidelines 3)/on a case to case basis.
4)/No error 5)
Read each sentence to find out whether there is
any grammatical mistake/ error in it. The error, if 534. After joining the Hong Kong based 10/ bank in
any, will be in one of the sentence. Mark the 1990,he has 2) / head various departments 3)/
number of that part with error as your answer. If Including corporate and investment banking. 4) / No
there is no error, mark5). error 5)

525. While major economies set aside 1)/about 3 Read each sentence to find out whether there is
percent of their GDP 2)/ in research and any grammatical error or idiomatic error in it. The
Development, India 3)/ spends less than 1 percent error, if any, will be in one part of the sentence. The
on this 4)/ No error 5) number of that part is the answer. If there is no
error, the answer is (5).(Ignore errors of
526. Owing to the poor quality of 1)/ the equipment punctuation, if any)
producing by Chinees firms 2)/ many Foreign
companies have placed 3)/ orders with Indian firms. 535. Sugar – sweetened drinks does not 1)/Pose any
4)/No error 5) particular health risk, and /2) are not a unique risk
factor /3) for obesity or heart disease. 4) No error 5)
527. We except a hike 1)/ in the cash reserve ratio
2)/ which is the portion of deposits 3)/ that Banks 536. Airline managements should note 1)/that the
keep with RBI. 4)/ No error 5) ultimate passenger – unfriendliness 2)/ are not a
unique risk factor / 3) for obesity or heart disease.
528. Unless these differences 1)/ will be resolved 4)/No error 5)
soon 2) / there will be an adverse 3)/ effect on
Foreign investment. 4)/No error 5) 537. Celebrating its ten long years 1)/in the industry,
a private entertainment channel 2)/announce a
529. We are confident that the steps 1) / we have series of 3)/ programmes at a press
taken to attract talented people 2)/ and build a Solid conference.4)/No error 5)
organization 3)/ will ensure we remain profitably 4)/
No error 5) 538. The award ceremony ended 1)/on a note of
good cheer 2)/with audiences responding warmly
530. At a time which most firms 1)/ were finding it 3)/to its line-up of films. 4)/No error 5)
difficult 2)/ to raise money we succeeded in
3)/raising the necessary funds 1)/ No error 5) 539. The actress was ordered for 1)/wears an
alcohol- monitoring bracket and 2)/ submits to
Use Code: (AVP10) for 10% India’s No.1 Teacher in Bank Exams for English Language and Editorial 44
additional discount on All Website: www.vishalparihar.com | Follow on Instagram: @vishalthetrainer
Unacademy courses
1000+ Error Detection for All Exams
Last 20 years (Previous Year Papers)
English By Vishal Parihar

random weekly drug testing after 3)/she failed to 548. It is important not 1)/ to direct people too 2)/
appear to for a court date last week. 4)/No error 5) much and to let them 3)/ find their own way 4)/ No
error 5)
540. Coaches have the advantage of 1)/draw on
their, personal experiences 2)/and providing their 549. Product planners, in the production units 1)/
players 3)/with unique inputs. 4)/No error 5) work intimately with programme 2)/Managers to
defining new ideas for products 3) /or product
5411. The actor loves to think, 1)/never enjoys stick features 4)/ No error. 5)
to one kind of role 2)/and finds it difficult to 3)/fulfill
everyone’s expectations. 4)/No error 5) 550. I will always remember 1)/the advice from my
supervisors as 2)/ I work on phones : Smile. If
542. A major computer security firm urged 1)/the doesn’t matter that they can’t see you; 3)/ you’ll
social networking site to set up 2)/an early – sound friendly and happy 4)/No error. 5)
warming system after hundreds of users were
(3)/hit by a new wave of virus attacks 4)/No error 5) 551. To our knowledge 1)/no one foresaw exactly
how fast and 2)/ how big the PC market Had grown
543. The finding may help doctors 1)/give more in the 3)/ years to come 4)/ No error 5)
personalized care to patients 2) / and modify the
amount of powerful drugs 3)/administered over 552. Results would have teen much different 1)/ had
their parents. 4)/No error 5) he projected himself into the minds 2)/ of the
masses of ordinary people and 3)/ ask himself these
544. The actress made a rare appearance 1)/at the question 4)/ No error 5)
party and was 2)/overheard talking in 3)/her next big
project. 4)/No error/5) 553. It had all begun 1)/ in a brightly lit room as 2)/ I
dreamily ran my fingers over 3)/ the surfaced of a
In each of the following sentences, the error, if globe 4)/ No error 5)
there is one will be in the part. The part which
contains error is the answer. If there is no error, the 554. On writing this book,1)/ we cannot over
answer is 5). The error if any will be of grammar or/ emphasize the unique 2)/ opportunity we have to
and usage. (Ignore error of punctuation, if any). probe broadly 3)/ and freely into the organization
4)/No error. 5)
545. Further trial have added weight on these
findings and an expert panel concluded 1)/There
was no evidence to recommend beta carotene 555. He has taken care to 1)/ compliance with the
supplements 2)/ for the general Population and norms 2)/ so he expects the proposal 3)/ to be
strong evidence 3)/ to recommend hat smokers approved without delay 4)/No error 5
avoid it 4)/ No error 5)
556. Under the terms of the new deal 1)/ the
546. Had I known 1)/ it earlier I 2)/ would had made channel can broadcast 2)/ the next cricket
3)/ a different plan 4)/ No error 5) Tournament to be 3)/ played among India and
Australia 4) / No error 5
547. If you decide to straighten 1)/ out a lot of things
in your life all at once,2)/ it is probable that you will 557.Our equipment gets damages 1) / very often in
3)/achieve nothing 4)/No error. 5) summer 2)/ because there are 3)/ frequent power
cuts 4)/ No error 5

Use Code: (AVP10) for 10% India’s No.1 Teacher in Bank Exams for English Language and Editorial 45
additional discount on All Website: www.vishalparihar.com | Follow on Instagram: @vishalthetrainer
Unacademy courses
1000+ Error Detection for All Exams
Last 20 years (Previous Year Papers)
English By Vishal Parihar

558.We have received many/ of the letters from 567. Her talk was 1)/ judged by many 2)/ as one of
customers/ asking us to extend / the deadline to the most important talks 3)/ given in the seminar 4)/
repay their loans / No error. 5 No error 5

559.Since I had lived there 1)/ for many years the 568.The boy who was guilty with 1)/ having stolen
villagers 2)/ were very comfortable talked 3)/ to me the cell phone 2)/ came out 3)/ with the truth 4)/
about their problems 4)/No error 5 No error 5

560.We have been under 1)/ a lot of pressure to 2)/ 569. She is the teacher 1)/ whom I know 2)/has
open fifty new stores 3)/ by the ending of the year helped my son 3)/ in the examination. 4)/ No error 5
4) /No error 5
570. You cannot demand 1)/ all your articles 2)/
561. The government has 1)/launched many unless you do not 3)/ give advance notice 4)/ No
creative schemes 2)/ to make banking services 3)/ Error 5
available to everyone 4)/No error 5
571. We insisted, but neither 1)/ Jona nor her
562.The company is in debt 1)/and has been unable assistant 2)/ are ready to 3)/ work on Sunday 4)/ No
2)/ to pay their employees salaries 3)/ for the past error 5
six months 4)/ No error 5
572. There is just not enough1) / time in my unit to
563.This is turned out to be1) / one of most sit round 2)/talking about how we feel 3)/ About
successful projects 2)/ and we have made quite 3)/ each other 4)/ No error 5
a large profit from it 4)/ No error 5
573. The act will be passed 1)/ with a comfortable
564.A non-banking financial company is a majority if all 2)/ the partisans of 3)/ her group
1)/financial institution similarly to a bank 2)/ but it Favours it. 4)/ No error 5
cannot issue 3)/ cheque books to customers 4)/ No
error 5 574. Hardly had I 1)/ entered the airport 2)/ than I
met 3)/ my cousin Tilak 4)/ No error 5
Read each sentence to find out. Whether there is
any grammatical error in it. The error, if any, will be 575. An anti-terrorism cell is opened 1)/ by the town
in one part of the sentence. The number of that police 2)/ three months ago 3)/ as a Precautionary
part is the answer. If there is no error the answer is measure 4)/ No error 5
5) (Ignore error of punctuation, if any)
576. The artist who was giving 1)/ detailed
565.Selling peanuts 1)/ on the road is 2)/ the only description of 2)/the sculptures was 3)/ speaking
means of earning 3)/ in respect of most of the fluently French. 4)/ No error 5
people 4) / No error 5
577. Martin would attempt 1)/ to open the umbrella
2)/ when her spectacles slipped off 3)/ and fell down
566. If you had read (1)/ the relevant literature 4)/ No error 5
carefully (2)/ you would have answered (3)/ most of
the questions correctly. (4)/ No error (5). 578. If you had not / reached so quickly,1) / we might
well 2)/ have had 3)/a disaster 4)/ No error 5

Use Code: (AVP10) for 10% India’s No.1 Teacher in Bank Exams for English Language and Editorial 46
additional discount on All Website: www.vishalparihar.com | Follow on Instagram: @vishalthetrainer
Unacademy courses
1000+ Error Detection for All Exams
Last 20 years (Previous Year Papers)
English By Vishal Parihar

579. Ancient artifacts are 1)/ a part of global


heritage 2)/ and should not be 3)/ sold to the highest 591. Ever since he took over 1)/ as the chief minister
bidder 4)/ No error 5 of the state 2)/rate for unemployment 3)/has
drastically increased 4)/No error 5
580. Most people like to 1) / rest after a day’s hard
work 2)/ but he seemed to have 3)/ an inexhaustive 592. Although the brilliant writer1)/an underlying
supply of energy 4)/No error 5 2)/ pessimism prevent in 3)/ all her novels 4)/No
error 5
581. None of the student 1)/ in the class 2)/ scored
below the 3)/given cut-off marks 4)/No error 5 593. Changed social setting / demands the schools
to teach /moral and social values / among with the
582. To be a king and 1)/ wear a crown are 2)/ more academic skills. No error. 5
glamorous to 3)/ see than to bear. 4)/ No error 5

583. Ashok is among the 1) / few people in the world 594. The president has denied 1)/ that the economy
2) / which did not blindly follow 3)/ the path of is in recession 2)/or was go into one 3)/ despite a
others 4)/ No error 5 Spate of downcast reports 4)/ No error 5

584. In emerging economies, 1)/the private credit 595. The angry at being 1)/ left out the bonanza 2)/
market 2)/ remains highly segmented and thus is palpable among 3)/ employees of the organization
3)/weaken power of monetary policy 4)/No error 5 4)/No error 5

585. The recent election campaign 1)/ has been one 596. His comments came after 1)/ the research
of 2)/ the most noisiest campaigns 3)/ in the last group said that its 2)/ consumer confidence index
decade.4) / No error 5 were 3)/ slumped to its lowest level 4)/No error 5

586. Wholesome strategic planning 1)/ was the 597. If all goes well 1)/ the examination scheduled
focus as 2)/ the firm manage through a difficult for next month 2)/ is all set to be completely free 3)/
Period 3)/ a couple of years ago 4)/ No error 5 free annoying power cuts and disruptions 4)/ No
error 5
587.Inspite of the best governmental efforts, 1)/
emission of greenhouse gases 2)/ and noxious 598. There are just too few trains 1)/ for the ever –
chemicals 3)/ remain a cause of worry 4)/ No error grow 2)/ number of passengers 3)/ in the city 4) / No
5 error 5

588. The rate of metabolism of 1)/ a body is 599. The buzz at the party was 1)/ that a famous 2)/
comparatively lowest when 2)/ it is at rest and it 3)/ film star and politician, would 3)/ probable drop by
thus optimum for examination 4)/ No error 5 for a while 4) / No error. 5

589. The opposition leader tried 1)/ to bolster his 600. The opposition disrupted proceedings 1)/ in
position 2)/ with the voters by pressing 3)/ both Houses of parliament 2)/ for the second
corruption charges against rivals 4)/No error 5 consecutive day 3)/ above the plight of farmers in
the country 4)/ No error 5
590. The recently imposed dress code 1)/ in the
university 2)/has enraged 3)/ the students who will 601. In response to the growing crisis 1)/ the agency
be 4)/ no error 5 is urgently asking for 2)/ more contributions, to
Use Code: (AVP10) for 10% India’s No.1 Teacher in Bank Exams for English Language and Editorial 47
additional discount on All Website: www.vishalparihar.com | Follow on Instagram: @vishalthetrainer
Unacademy courses
1000+ Error Detection for All Exams
Last 20 years (Previous Year Papers)
English By Vishal Parihar

make up for 3)/ its sharp decline in purchasing 612. I have being working 1) / in that organization
power 4)/No error 5 for five year 2)/ but now I work 3)/ for a different
company.4) / No error 5
602. The tennis player easy through 1)/ the openings
set before her opponent 2)/ rallied to take the final 613. There is no argument against 1)/ the fact that
two sets 3)/ for the biggest victory of her young 2)/ humans are the most intelligent 3)/ of all other
career 4)/No error 5 Species. 4)/ No error.

603. Aggression in some teenage boys 1)/ may be


linkage to overly 2)/ large glands in their brains,3)/ a 614. We have recently began 1)/ the process of
new study has found 4)/ No error 5 recruiting 2)/ insurance agents for3) / our North –
Eastern branches. 4)/ No error. 5

604. The merchant counted1) / the number of pearls 615. Inspite that organizations are aware 1)/ of the
2)/ to make sure that 3)/none of them were missing importance of IT 2)/ they often do not know 3)/ how
4)/ No error. 5 to deploy it effectively. No error 5

605. When deep sea diving 1)/ one should always 616. Although interlinking of rivers will 1)/ help
take care 2)/ that oxygen cylinder is 3)/ tied to the tackle drought, the government 2)/ has not paid
back tightly 4)/No error. 5 much3) / attention to this project. 4)/ No error. 5

606. As the salary 1)/ did not match 2)/ his 617. Keeping in mind that India 1)/ is one of the
expectations, he did not 3)/ accept the job 4)/ No world’s fastest 2)/ growing economies many
error 5 international 3)/ companies are invested in India.4)
/ No error 5
607. The reason behind his success 1)/ in the recent
past 2)/ is due to hard work 3)/ and presence of 618. Despite the merger 1) / the airline has reported
Mind.4) / No error 5 2)/ sizeable losses and is unable 3)/ to pay their
employees. 4)/ No error 5
608. No sooner did the students 1) / seen the
principal approach 2)/ than they ran 3) / from the 619. The company’s decision to issue 1)/ bonus
play Ground.4) / No error. 5 shares clearly indicates 2)/ the management’s
confident 3)/ about the future. 4)/ No error. 5
609. My elder sister and I am 1)/ interested in
painting 2) / and therefore have joined 3)/ the 620. RBI is in favour 1) / of amending currently 2)/
coaching classes 4)/ No error. 5 policies governing operations of 3)/ foreign banks in
India.4) / No error 5
610. Only after a lot of persuasion,1) / the illiterate
villager allowed 2)/ him self’s girl child 3)/ to study 621. These recently upgraded branch offices 1)/
in the school 4)./ No error. 5 have been fully computerized 2)/ and are connected
to headquarters 3) / to ensure proper monitoring.4)
611. The need for alternate1) / sources of energy are / No error 5
2) / essential to conserve 3) / the environment. 4)/
No error 5 626. The rise in prices of food items is 1)/ likely the
cause of inflation 2)/ to exceed eight percent 3)/
during the next quarter 4) / No error. 5
Use Code: (AVP10) for 10% India’s No.1 Teacher in Bank Exams for English Language and Editorial 48
additional discount on All Website: www.vishalparihar.com | Follow on Instagram: @vishalthetrainer
Unacademy courses
1000+ Error Detection for All Exams
Last 20 years (Previous Year Papers)
English By Vishal Parihar

627. Many people believe that 1) / the 638. You have not know it (1)/ but this engine is (2)/
nationalization of banks in 2)/ the 1960s have claimed to have twice (3)/ as powerful as the
protected India 3)/ from the current financial previous one. (4)/ No error (5)
crisis.4) / No error 5
639. Nothing ever becomes real (1)/ till it is
628. Sheela had to travel all the way back 1)/ when experienced. (2)/ Even a proverb is no proverb to
she remembered that 2)/ she had forgot to take 3) you (3)/ till your life has illustrated with it. (4)/ No
/important documents with her 4)/ No error 5 error (5).

629. The party is holding a meeting 1)/ in order to 640. I remember my childhood days (1)/ when I was
begin the 2)/ painful and difficult process 3)/ of used to go (2)/ to the farm with my father (3)/ and
rethink its political strategy 4) / No error. 5 help him in his work. (4)/ No error (5).

630. The only bad thing about 1)/ these flowers are 641. I missed the last train (1)/ which I usually catch
the 2)/ pollen which causes 3)/ allergy in many (2)/ and have to stay at the station (3)/ on my way
people.4) / No error. 5 back home yesterday. (4)/ No error (5).

631. No body in the bus 1)/ offered to give their seat 642. Sureshbabu, who is living (1)/ in this town since
2)/ to the elderly lady who had to 3)/ travel a long 1955, (2)/ is a well – known scholar of history (3)/
distance. 4)/ No error. 5 and a distinguished musician. (4)/ No error (5).

632. This is the only organization 1)/ which could 643. If you had read (1)/ the relevant literature
somehow continue2) / to making profits3) / in the carefully (2)/ you would have answered (3)/ most of
face of recession.4) / No error 5 the questions correctly. (4)/ No error (5).

633. Many more would have died 1)/ if they hadn’t 644. The house where the dead man was found (1)/
received 2) / the timely helping from 3) / the relief is being guarded by police (2)/ to prevent it from
team. 4)/ No error 5 being entered (3)/ and the evidence interfered with
(4)/ No error (5).
634. A man along with 1)/ his wife and two children
2)/ were rescued with the help 3) / of an army Read each sentence to find out if there is any error
helicopter.3) / No error. 5 in it. The error, if any, will be in one part of the
sentence. The number of that part is the answer. If
635. In the past two decades 1)/ preventive there is no error the answer is 5). (Ignore errors in
measures 2)/have drastically reduced the 3)/ punctuation, if any)
occurrence of polio in children. 4)/ No error 5
645. They are scheduled 1)/a meeting today to
636. Neither the ministers 1)/ nor the intelligence 2)/accept the resignation 3)/of the Director 4) No
sources 2)/ was aware of the coup 3)/ being planned error 5)
in the country 4)/ No error 5
646. The launch of 1)/the new products was
637. Certain amounts of stress 1) / at work is natural 2)/delayed which resulted 3)/in heavily losses 4)/ No
but 2)/ it is important that one learns 3)/ to keep it error. 5)
in the right proportion. 4)/ No error. 5

Use Code: (AVP10) for 10% India’s No.1 Teacher in Bank Exams for English Language and Editorial 49
additional discount on All Website: www.vishalparihar.com | Follow on Instagram: @vishalthetrainer
Unacademy courses
1000+ Error Detection for All Exams
Last 20 years (Previous Year Papers)
English By Vishal Parihar

647. The chairman has promised 1)/to sanction 659. To know what people react to (1)/ an
2)/the necessary funds to 3)/modernize the factory unforeseen situation (2)/ is very important (3)/ to
4)/ No error 5) decide our course of action. (4)/ No error (5)

648. The company has decided 1)/ to invest in 2)/ 660. Some people like (1)/ to do some special (2)/
one crore rupees 3)/ in new technology 4)/ No error work even though (3)/ it lacks resources. (4)/ No
5) error (5)

649. The success of the 1)/merger of the two 661. His qualities include(1)/ the ability to take (2)/
2)/banks is dependence 3)/on various factors 4)/No right and quickly decisions (3)/ on important
error. 5) policies. (4)/ No error. (5)

650. The meeting taken place 1)/when you were 662. A group of youngsters (1)/ has lend financial
2)/at a conference 3)/in Delhi last month 4)/ No assistance (2)/ to the flood – affected (3)/ people of
error 5) the villages. (4)/ No error (5)

651. Most companies neglect 1)/to spend none 2)/of 663. We must take (1)/ regular exercises (2)/ to keep
their resources 3)/on training older employees 4)/ (3)/ our self fit. (4)/ No error (5)
No error. 5)
664. If you want to (1)/ ensure prompt service (2)/
652. Even today there are 1)/ several million people please have contact us (3)/ as soon as possible. (4)/
2)/who do not have 3)/ access of banking services No error (5)
4)/ No error. 5)
Read each sentence to find out whether there is
653. We are still 1)/waiting for the 2)/committee any grammatical error in it. The error if any will be
decision about 3)/the pension plan 4)/ No error 5) in one part of the sentence, the number of that part
will be the answer. If there is no error, mark 5) as
654. Ramesh doubts when 1)/ he will be ready 2)/ the answer. (Ignore errors of punctuation, if any.)
for the presentation 3)/ in time 4)/ No error. 5)
665. A large number of workers 1)/ of the factory
recommends 2)/ Rohit as the union leader 3)/ for
655. Our customers are always visit (1)/ our the next two years. 4)/ No error 5)
branches to (2)/ discuss their problems (3)/ with our
staff. (4)/ No error (5) 666. The public believed 1)/ that the national leader
2)/ was one of the 3)/ greatest leaders of every time.
656. Although he was expected (1)/ to work quite 4)/ No error 5)
hard (2)/ he could not do so (3)/ owing to his
prolonged illness. (4)/ No error. (5) 667. A recent study suggested 1)/ that tuberculosis
was 2)/ more widely spread 3)/ disease than
657. Just before the demonstration started (1)/ the malaria. 4)/ No error 5)
police arrive at the site (2)/ as if they had (3)/
received information. (4). No error (5) 668. The teachers could not find 1)/ which one of the
students 2)/ were responsible of 3)/ breaking the
658. Habit of (1)/ regular saving (2)/ helps us (3)/ window pane. 4)/ No error 5)
during bad times. (4)/ No error (5)

Use Code: (AVP10) for 10% India’s No.1 Teacher in Bank Exams for English Language and Editorial 50
additional discount on All Website: www.vishalparihar.com | Follow on Instagram: @vishalthetrainer
Unacademy courses
1000+ Error Detection for All Exams
Last 20 years (Previous Year Papers)
English By Vishal Parihar

669. She was taken with surprise 1)/ when she was family planning (3)/ have been eroded considerably.
announced 2)/ the winner of the 3)/ national level (4)/ No error (5).
competition. 4)/ No error 5)
680. I live on a top floor 1)/ of an old house and 2)/
670. 1) There cannot be any situation where /2) when the wind blows 3)/ all the windows rattle. 4)/
somebody makes money in asset /3) located in India No error 5)
and does not pay tax /4) either to India or to the
country of his origin./5) No error 681. Had he not reached in time 1)/ he would have
missed 2) a golden opportunity which comes 3)/
671. 1) India has entered a downward spiral / 2) once in a while. 4)/ No error 5)
where the organized, productive / 3) and law abide
sectors are subject to / 4) savage amounts of 682. The ministry of Labour conducts its basic survey
multiple taxes. / 5) No error 1)/ on trade unions every year 2)/ gathering
information with all 3) labour unions as of June 30.
672. 1) The bank may have followed / 2) as 4)/ No error 5)
aggressive monetary tightening policy / 3) but its
stated aim of / 4) curbing inflation have not been 683. Stories have always be used 1)/ as a powerful
achieved. / 5) No error tool for 2)/ communicating vital information from 3)
one generation to another. 4)/ No error 5)
673. 1) Equal opportunities for advancement / 2)
across the length and breadth / 3) of an organization 684. We all are indebted to him 1)/ for holding not
will / 4) keep many problems away. /5) No error. only high moral values 2) but also rendering
dedicated services 3)/ for the betterment of human
674. 1) A customized data science degree / 2) is yet race. 4) No error 5)
to become / 3) a standard programme / 4) to India’s
premier educational institutes. / 5) No error 685. We are trying to 1)/ find out the root cause 2)/
of this recurring problem 3)/ since the last four days
675. This laboratory of Physics is (1)/ not only 4)/ No error 5)
equipped with (2)/ all state – of – the – art
instruments (3)/ but also with outstanding 686. I told him bluntly 1)/ that he is a lazy fellow 2)/
physicists. (4)/ No error (5) and that he had done 3)/ his work very badly 4)/ No
error 5)
676. No method of making (1)/ other people agree
to (2)/ your view-point is (3)/ as effective as this 687. Mumbai is full of tall buildings 1)/ which is at
method. (4)/ No error (5) once its 2)/ strength as well as weakness 3)/
depending upon how one look at it. 4)/ No error 5)
677. I was pretty sure that (1)/ he would support my
views (2)/ for changing the age-old (3)/ and static 688. I am fully aware that 1) my boss is one of those
structure of our organization. (4)/ No error (5) 2)/ who is totally dedicated 3)/ to work and proper
discipline. 4)/ No error 5)
678. I did not like his (1)/ comments on my paper
(2)/ but I had no alternative (3)/ as I had agreed to 689. The director of our company does not believe
keep quiet. (4)/ No error that 2)/ we are working sincerely 3)/ and with
interest for all these years. 4)/ No error 5)
679. The report is candid in admitting (1)/ that the
investment by the government (2)/ in health and
Use Code: (AVP10) for 10% India’s No.1 Teacher in Bank Exams for English Language and Editorial 51
additional discount on All Website: www.vishalparihar.com | Follow on Instagram: @vishalthetrainer
Unacademy courses
1000+ Error Detection for All Exams
Last 20 years (Previous Year Papers)
English By Vishal Parihar

In each question, a sentence is given with a part of 1) Only A 2) Only B 3)


it printed in bold type. That part may contain a Either A or B 4) No correction required
grammatical or idiomatic error. Each sentence is 5) NOT
followed by two parts denoted by (A) and (B). Find
out which part ‘A only’, ‘B only’ or ‘Either A or B’ Directions: Read each sentence any to find
can correct the error, if any in the sentence and out whether there is any grammatical
mark your answer accordingly. If the sentence error or idiomatic error in it. The error, if
contains no error, mark (4) i.e. ‘No correction any, will be in one part of the sentence. If
required’ as your answer. However, if the sentence there is no error, the answer is No error.
has an error, but none of the two parts 1 or 2 can (Ignore errors of punctuation, if any)
correct it, mark (5) i.e. ‘None of these’ as the
answer. Q695. The judge asked the man (a)/if the bag
he had lost (b)/contain five thousand rupees.
690. The revenue deficit was low in the revised
(c)/The man replied that it did. (d)/No error
estimates than the budget estimates.
(e).
(A) has lower (B) was lower
1) Only A 2) Only B 3)
Either A or B 4) No correction required Q696. I trust you will (a)/show forbearance to
5) NOT me (b)/a few minutes more (c)/so that I can
finish this work. (d)/ No error (e).
691. He has been stayed in the same house for the
last ten years. Q697. The ground outside the village,
(A) has been stayed (B) was staying (a)/abounding with frogs and snakes, (b)/the
1) Only A 2) Only B 3) enemies of mankind, (c)/is soft and marshy.
Either A or B 4) No correction required (d)/No error (e).
5) NOT
Q698. We are all short-sighted (a)/and very
692. One of the major motives of levying any tax is often see but one side of the matter. (b)/Our
to increase government revenue. views are not extended (c)/ to all that has a
(A) main motives behind (B) major connection with it. (d)/No error (e).
aims of
1) Only A 2) Only B 3) Q699. Just laws are no restraint with (a)/the
Either A or B 4) No correction required freedom of the good, (b)/for the good man
5) NOT desires nothing (c)/ which a just law interfere
with. (d)/No error (e).
693. Had you been told us earlier, we would have
helped you.
(A) had you (B) If you had
1) Only A 2) Only B 3) Directions : In each of the questions given
Either A or B 4) No correction required below, there is a statement which is
5) NOT correct grammatically. Mark that option
as your answer. All of the options can also
694. We had no other option without to admit him. be correct, In that case, mark option (e) as
(A) but to (B) except your answer.

Q700.
Use Code: (AVP10) for 10% India’s No.1 Teacher in Bank Exams for English Language and Editorial 52
additional discount on All Website: www.vishalparihar.com | Follow on Instagram: @vishalthetrainer
Unacademy courses
1000+ Error Detection for All Exams
Last 20 years (Previous Year Papers)
English By Vishal Parihar

(a) I could clearly seen the site since it was (b)Good advertising can make people buy
dark. your products even they sucks.
(b) We are all atheists about most of the gods (c)Making people laugh is trickable.
that humanity has ever believed in. (d)The delegation of any power, to any
(c) Seat reservation in colleges is a case of person, is never absolute.
haste action. (e)All are correct.
(d) He is usual tense but today he appears
rather restless.
(e) All are correct. Directions : In the following questions
read each of the following four sentences
to find out whether there is any
Q701. grammatical mistake/error in it. Choose
(a)To a large extent, many cooperative the sentence which has a grammatical
societies need advice and guidance. error, as your answer. If all the given
(b)The investigation was confine to sentences are grammatically correct,
manufacturing firms in the area. choose (e) i.e., “None of these” as an
(c)They quickly come back with pitchers answer.
laden with water.
(d)This is another kind of signing behavior. Q704. (a) Assam Finance Minister Himanta
(e)All are correct Biswa Sarma had in 2017 said that a law
would be introduced to ensure parents are
Q702. cared for.
(a)India national pastime is cricket. (b) Justice S.A. Bobde explained that the
(b)Christopher Reeve confirm a faith in life mediation, if undertaken, would be
that will not be easily forgotten. confidential and court-monitored and would
(c)Forecasting the weather has always been a last eight weeks.
difficult business. (c) As he is usually busy, he has to travel by a
(d)He himself didn’t have many hope of car everywhere.
recovery. (d) Mr. Shah said he had decided to launch the
(e)All are correct campaign from Ghazipur as it was the land of
Suheldev and Abdul Hamid.
Q703. (e) None of these
(a)We seldom examined the basic character
of its anatomy. Q705. (a) The Nagaland Assembly has passed
(b)The process continued for millions of year. a resolution rejecting the contentious
(c)The decline in write skills can be stopped. Citizenship (Amendment) Bill.
(d) Financial services is up against tight (b) Gauri was shot dead in broad daylight in
liquidity and falling markets. front of her Bengaluru residence.
(e)All are correct. (c) In the buffer zone, the fire destroyed
nearly 50 acres in Bokkapuram and Singara.
Q704. (d) It is appearing that the two countries are
(a)Learning is an important for both people plotting against the regional super power.
and organisations. (e) None of these

Use Code: (AVP10) for 10% India’s No.1 Teacher in Bank Exams for English Language and Editorial 53
additional discount on All Website: www.vishalparihar.com | Follow on Instagram: @vishalthetrainer
Unacademy courses
1000+ Error Detection for All Exams
Last 20 years (Previous Year Papers)
English By Vishal Parihar

Q706. (a) Armed men looted ₹2.5 lakh from a correct, mark ‘option (e)’ i.e., “all are
petrol pump in Gomati district. correct” as your answer choice.
(b) A predicted 70% decline in summer rains
by 2050 would devastate Indian agriculture. Q709. (a) The natural gas sector saw growth
(c) The police held a foreign national after he slowing in March to 1.4% from 3.8% in
breached the high security gates of the Israeli February.
Embassy. (b) House sales have collapsed to their lowest
(d) He was much more surprised than me to level since Paul Keating was prime minister,
see him at the restaurant. dragging down property values across the
(e) None of these country
(c) The rate of property turnover had
collapsed to less than 3 per cent, a
Q707. (a) The country has over 120 million development that was a bad sign for
hectares suffering from some form of renovations and consumption.
degradation. (d) Farmer’s institutes are held throughout
(b) He is almost quite competent to deal with the South by teachers of the school.
this situation and lead the country to its (e) All are correct
ultimate goal.
(c) Teams were expected to be disruptive in Q710. (a) Despite some good rainfall last
their outlook and challenge the status quo. month, outlooks are still not indicating
(d) Security forces continued to patrol widespread drought relief in the short term.
sensitive areas to check potential (b) Ruszczyk's brother Jason and his wife
troublemakers. started sobbing and embraced each other.
(e) None of these (c) Major political parties have turned a blind
eye to their campaigners violating the spirit of
the model code of conduct.
Q708. (a) The police verified his credentials (d) Observers of Indian elections are likely to
before initiating any legal action agree that rarely has campaigning been as
(b) The two missiles were tested for different fractious and malicious as the current one.
altitudes and conditions. (e) All are correct
(c)The survey shows that 85% of Indian
corporate houses are hiring gig workers
(d) The company's last-mile delivery model,
was born out of the efforts of a four-member Q711. (a) Circular Economy is a way to map
cross-functional team. and manage all material resources through
(e) None of these their life cycle so that nothing is wasted.
(b) The number of millionaires at the top of
India’s economy is growing while millions are
Directions : In each of the following scrambling for employment to earn adequate
question, four sentences are given. incomes.
Identify and mark the sentence that (c) One little girl had lesser presents than the
contains a grammatical or a contextual rest, and Helen insisted on sharing her gifts
error. If all the given sentences are with her.

Use Code: (AVP10) for 10% India’s No.1 Teacher in Bank Exams for English Language and Editorial 54
additional discount on All Website: www.vishalparihar.com | Follow on Instagram: @vishalthetrainer
Unacademy courses
1000+ Error Detection for All Exams
Last 20 years (Previous Year Papers)
English By Vishal Parihar

(d) The model code of conduct presumes that Q716. A representative of the (a)/ Reserve
political parties will largely adhere to its Bank will provide students an (b)/ insight
underlying spirit. into the (c)/ economic future of our country.
(e) All are correct (d)/ No error. (e)
Ans: (b) When provide (Verb) means give
someone something, then with (Prep.) is used
Q712. (a) By packaging the shampoo in small with it.
sachets, the company was able to increase its
sales. Q717. As one of the leader (a)/ insurance
(b) Unless people who work become the companies in (b)/ India they offer (c)/
owners of the enterprises, they will not be comprehensive financial services. (d)/No
making any profits, nor will they earn wealth. error. (e)
(c) There are similarities in the structures of
the circular material economy and the Q718. There is a rumour that (a)/ this
circular financial economy. multinational company will (b)/ set up its
(d) He hadn't been able to sleep in two days regional headquarters (c)/ in India in short.
but fell fast into a deep, peace slumber. (d)/ No error. (e)
(e) All are correct
Q719. Despite taking steps to (a)/ encourage
Q713. (a) Those who have wealth will make foreign investment (b)/ there has been any
more wealth by investing it in more (c)/ substantial improvement in our
enterprises to make more profits. economy. (d)/ No error. (e)
(b) The cell-walls of plants renders the entry
of solid material into the organism Q720. We had made every effort (a)/ to
impossible. ensure that a (b)/ com promise is reached
(c) In the financial economy, financial capital and (c)/ that deal was signed. (d)/ No error.
is generated out of the production system and (e)
it accumulates in the financial sector.
(d) The size of the financial sector in all Q721. India‘s travel and tourism (a)/ industry
economies has grown greatly in the past is poised (b)/ on growth at seven (c)/ per cent
thirty years. this year alone. (d)/ No error. (e)
(e) All are correct
Q722. The Ministry was considered (a)/
Q714. The two candidates share (a)/ a several proposals for the (b)/ development of
reputation for (b)/ competency as well as (c)/ small and medium (c)/ enterprises during
for good communication skills. (d)/ No error. Budget discussions. (d)/ No error. (e)
(e)
Q723. The amount of foreign (a)/ direct
Q715. His main qualification (a)/ on the job is investment in (b)/ the country in 2008 is (c)/
(b)/ his extensive experience (c)/ in foreign doubled
branches. (d)/ No error. (e) that received in 1997. (d)/No error.(e)

Use Code: (AVP10) for 10% India’s No.1 Teacher in Bank Exams for English Language and Editorial 55
additional discount on All Website: www.vishalparihar.com | Follow on Instagram: @vishalthetrainer
Unacademy courses
1000+ Error Detection for All Exams
Last 20 years (Previous Year Papers)
English By Vishal Parihar

Q724. The details of the scheme (a)/ will be Directions : In each of the questions given
made clearly (b)/ to the public by (c)/ the below a part of the sentence is given in
end of the financial year. (d)/ No error. (e) bold. It is then followed by three options.
Find the alternative that can replace the
Q725. Government officials have been given bold part to make the sentence
working (a)/ overtime to answer queries grammatically and contextually correct. If
raised (b)/ by banks on numerous of none of the alternatives is correct and the
(c)/issues pertaining to the loan waiver. (d)/ sentence is correct as it is then select
No error. (e) option (e), ‘no correction required’ as your
choice.
Q726. We have sought (a)/ clearance in (b)/
additional expenditure incurred (c)/ during Q732.
the project. (d)/ No error. (e) He had bought a dozen of copies of his
favourite music album, which was released
Q727. The Reserve Bank of India‘s decision last week and has already sold over 500
(a)/ to waive ATM charges (b)/ have put copies.
banks (c)/ in a difficult position. (d)/ No (i) a dozen copies
error. (e) (ii) dozens of copies
(iii)a dozens of copies

Q728. The government is planning to (a)/ (a)Only (i)


sanction grants to pharmaceutical companies (b)Only (ii)
(b)/ for inventing new treatments for (c)/ (c)Only (iii)
diseases about malaria and tuberculosis. (d)/ (d)Both (i) and (ii)
No error. (e) (e) No correction required

Q729. The Finance Minister‘s view (a)/ is that Q733. Venezuela has sent its oil minister,
the scheme will ensure (b)/ that millions of Manuel Quevedo, to India to convince refiners
farm ers will be (c)/ lifted out of poverty. (d)/ to double their oil purchases.
No error. (e) (i) will have sent its oil minister
(ii) had been sending its oil minister
Q730. The survey conducted reveals (a)/ that (iii) have sent its oil ministers
there is a lack (b)/ of adequate healthcare
facilities (c)/ even in urban areas. (d)/ No (a) Only (i)
error. (e) (b) Only (ii)
Ans: (e) No error. (c) Only (iii)
(d) All of the above
(e) No correction required

Q731. Wheat productions in the region (a)/


will fall drastically (b)/ unless we do anything
(c)/ to stop climate change. (d)/ No error. (e) Q734. You would have qualified the
examination by now if you would not have
wasted your time searching for the job.
Use Code: (AVP10) for 10% India’s No.1 Teacher in Bank Exams for English Language and Editorial 56
additional discount on All Website: www.vishalparihar.com | Follow on Instagram: @vishalthetrainer
Unacademy courses
1000+ Error Detection for All Exams
Last 20 years (Previous Year Papers)
English By Vishal Parihar

(i) if you have not wasted correct and does not require any
(ii) if you would had not replacement, choose option (e), i.e. “No
(iii) if you had not wasted replacement required” as your answer.

(a) Only (iii) 737.


(b) Only (i) Supreme Court had correctly deduced
(c) Only (ii) whether the spate of lynchings and
(d) Both (i) and (iii) emboldening of mobs is a law and order issue
(e) No correction required needing administrative redress.
(a) Supreme Court had correctly deducing
that
Q735. He might win the race as he is (b) Supreme Court has correctly deduced
comparatively faster than his competitors. whether
(i) as he is comparatively fast than (c) Supreme Court has been correctly
(ii) as he is faster than deduced that
(iii) as he is fast than (d) Supreme Court has correctly deduced that
(e) No replacement required
(a) Both (ii) and (iii)
(b) Both (i) and (ii)
(c) Both (i) and (iii)
(d) All the above Q738.
(e) No correction required Electric power is the way to go for the
transport sector, gave the need to curbing
greenhouse gas emissions and air pollution.
Q736. You must either inform the police else (a) to give the need to curbing the
be prepared to suffer any loss. (b) to give the need to curb
(i) else get prepared (c) given the need to curb
(ii) or else be prepared (d) gave the need to curb
(iii) or be prepared (e) No replacement required

(a) Only (i) Q739.


(b) Only (ii) This Independence Day, when Prime Minister
(c) Both (i) and (iii) Narendra Modi announced India’s manned
(d) Only (iii) space mission by 2022, among the
(e) No correction required surprising were Indian Space Research
Organisation (Isro) scientists.
Directions : In the questions given below, (a) among those surprised were
there is a sentence in which one part is (b) between the surprised were
given in bold. The part given in bold may (c) amongst the surprised are
or may not be grammatically correct. (d) with those surprising are
Choose the best alternative among the (e) No replacement required
four given which can replace the part in
bold to make the sentence grammatically
correct. If the part given in bold is already Q740.
Use Code: (AVP10) for 10% India’s No.1 Teacher in Bank Exams for English Language and Editorial 57
additional discount on All Website: www.vishalparihar.com | Follow on Instagram: @vishalthetrainer
Unacademy courses
1000+ Error Detection for All Exams
Last 20 years (Previous Year Papers)
English By Vishal Parihar

It was the curiosity to know of the reason company to set up (c)/ a plant in the state.
for behind the fall of an apple which made (d)/ No error (e).
Isaac Newton invents the Laws of Motion.
(a) of knowing the reason for Q746. Owing the new policy (a)/ we feel that
(b) of knowing the reason the targets (b)/ set for this year (c)/ may not
(c) to know of the reason be achieved. (d)/ No error (e)
(d) for knowing of the reason
(e) No replacement required Q747. Since the lack of (a)/ man power we
cannot (b)/ conduct the survey (c)/ in rural
Q741. It is not only the change but ‘the rate of areas. (d)/ No error (e)
change’ also that is increasing as we are
moving forward in the journey of human Q748. Palm oil is very beneficial (a)/ and is
evolution. use to (b)/ make products ranging (c)/ from
(a) we have been moving forward on soap to biodiesel. (d)/ No error (e)
(b) we have moved forward to
(c) we were moving forward to Q749. There are only (a)/ a few company (b)/
(d) we had moved forward in which can handle (c)/ projects of
(e) No replacement required this magnitude. (d)/ No error (e)

Q750. The data shows that (a)/ the


Directions: Read each sentence any to find unemployment rate has (b)/ raised to 6.1 per
out whether there is any grammatical cent (c)/ the highest in five years. (d)/ No
error or idiomatic error in it. The error, if error (e)
any, will be in one part of the sentence. If
there is no error, the answer is No error. Q751. The government has (a)/ launched
(Ignore errors of punctuation, if any) many creative schemes (b)/ to make
banking services (c)/ available to everyone
(d)/. No error (e).
Q742. The scheme which will be launched
(a)/during the next two years (b)/ require an Q752. We have received many (a)/ of the
additional investment (c)/ of one hundred letters from customers (b)/ asking us to
crores. (d)/ No error (e). extend (c)/ the deadline to repay their loans
(d)/. No error (e).
Q743. Road developers unable (a)/ to
complete their projects (b)/ on time will not Q753. Under the terms of the new deal (a)/
be (c)/ awarded new ones. (d)/ No error (e) the channel can broad cast (b)/ the next
cricket tournament to be (c)/ played among
Q744. We have taken on (a)/ the India and Australia (d)/. No error (e).
responsibility of (b)/ arranging the required
training (c)/ and supervise the new Q754. This is turned out to be (a)/ one of our
staff.(d)/No error (e) most successful pro jects (b)/ and we have
made quite (c)/ a large profit from it (d)/. No
Q745. The government has signed (a)/ a error (e).
memorandum of understanding with (b)/ the
Use Code: (AVP10) for 10% India’s No.1 Teacher in Bank Exams for English Language and Editorial 58
additional discount on All Website: www.vishalparihar.com | Follow on Instagram: @vishalthetrainer
Unacademy courses
1000+ Error Detection for All Exams
Last 20 years (Previous Year Papers)
English By Vishal Parihar

Q755. A non-banking financial company is a Q764. Most people like to (a)/rest after a day
(a)/ financial institution similarly to a bank ‘s hard work (b)/ but he seemed to have
(b)/ but it cannot issue (c)/ cheque books to (c)/an inexhaustive supply of energy. (d)/No
customers (d)/. No error (e). error. (e)

Q756. He has taken care to (a)/ compliance


with the norms (b)/ so he expects the Q765. Ancient artefacts are (a)/ a part of
proposal (c)/ to be approved without delay global heritage (b)/ and should not be (b)/
(d)/. No error (e). sold even to the highest bidder. (d)/ No
error.(e)
Q757. Since I had lived there (a)/ for many
years the villagers (b)/ were very Q766. In emerging economies, (a)/ the
comfortable talked (c)/ to me about their private credit market (b)/ remains highly
problems (d)/No error (e). segmented and thus (c)/ weaken the power of
monetary policy. (d)/ No error (e)
Q758. Our equipment gets dam age (a)/ very
often in summer (b)/ because there are (c)/ Q767. The recent election campaign (a)/has
frequent power cuts (d)/. No error (e). been one of (b)/ the most noisiest campaigns
(c)/ in the last decade. (d)/ No error (e)
Q759. We have been under (a)/ a lot of
pressure to (b)/ open fifty new stores (c)/ by Q768. Wholesome strategic planning (a)/ was
the ending of the year (d)/. No error (e). Ans: the focus as (b)/ the firm manage through a
(d) by the ending of the year difficult period (c)/ a couple of years ago. (d)/
No error (e)
Q760. The company is in debt (a)/ and has Ans: (c) The sentence shows past time.
been unable (b)/ to pay their employees‘ Therefore, use Simple Past here i.e. the firm
salaries (c)/ for the past six months (d)/. No managed through a difficult period is the right
error (e). usage.

Q761. To be a king and (a)/ wear a crown are Q769. Inspite of the best govern mental
(b)/ more glamorous to (c)/ see than to bear. efforts, (a)/ emission of greenhouse gases
(d)/No error (e). (b)/ and noxious chemicals (c)/ remain a
cause of worry. (d)/ No error (e)
Q762. None of the student (a)/ in the class
(b)/ scored below the (c)/ given cut-off Q770. The rate of metabolism of (a)/ a body is
marks. comparatively lowest when (b)/ it is at rest
(d)/ No error (e). and is (c)/ thus optimum for examination.
(d)/ No error (e)
Q763. Ashok is among the (a)/ few people in
the world (b)/ which did Q771. The opposition leader tried (a)/ to
not blindly follow (c)/ the path of others.(4)/ bolster his position (b)/ with the voters by
No error. (e) pressing (c)/ corruption charges against
rivals. (d)/ No error (e)

Use Code: (AVP10) for 10% India’s No.1 Teacher in Bank Exams for English Language and Editorial 59
additional discount on All Website: www.vishalparihar.com | Follow on Instagram: @vishalthetrainer
Unacademy courses
1000+ Error Detection for All Exams
Last 20 years (Previous Year Papers)
English By Vishal Parihar

Q772. The recently imposed dress code (a)/ not (c)/ confessed their crime. (d)/ No error
in the university has enraged (b)/ the (e)
students who will be going (c)/ on strike since
tomorrow. (d)/ No error (e) Q782. As soon as the CEO (a)/ entered the
office, the (b)/ union leaders approached him
Q773. Ever since he took over (a)/ as the chief (c)/ and report the matter to him. (d)/ No
minister of the state, (b)/ rate for error (e)
unemployment (c)/ has drastically increased.
(d)/ No error (e) Q783. Instead of being a clear winner (a)/ he
was not given (b)/ the best employee (c)/ of
Q774. Although a brilliant writer, (a)/ an the year award. (d)/ No error (e)
lying (b)/ pessimism pre vails in (c)/ all her
novels. (d)/ No error (e) Q784. Having been found (a)/ guilty of the
theft, (b)/ Sunny was sentenced (c)/ to five
Q775. Changed social setting (a)/ demands year‘s imprisonment. (d)/ No error (e)
the schools to teach (b)/ moral and social
values (c)/ among with the academic skills. Q785. Instead of laying off (a)/ the workers,
(d)/ No error (e) the company (b)/ decided to ask them (c)/ to
avail voluntary retirement. (d)/ No error (e)

Q776. I may go to the (a)/ swimming class Q786. Some genuine issues exist (a)/ with the
tomorrow (b)/ if I have recovered (c)/ from newly adopted (b)/ system and needs to (c)/
the cold. (d)/ No error (e) be examined seriously. (d)/ No error (e)

Q777. The prime minister announced (a)/ Q787. Whether or not to confront (a)/ them
that the taxes will be (b)/ increasing from the about their role (b)/ in the matter is a
(c)/ beginning of next year. (d)/ No error (e) decision (c)/ which is yet to take. (d)/ No
error (e)
Q778. He is the most (a)/ intelligent and also
(b)/ the very talent ed (c)/student of the Q788. The government is still in the (a)/
college. (d)/ No error (e) process of finalised new policy (b)/
guidelines for the allocation of land (c)/ to
private sector
Q779. She immediately quit (a)/ the job in organisations. (d)/ No error (e)
which (b)/ neither the skill nor (c)/
knowledge were required. (d)/ No error (e) Q789. According to government estimates
(a)/ at least four million tonnes of sugar (b)/
Q780. The meteorological department (a)/ will have to be imported (c)/ this year
predicted that the (b)/ rains and because of a poor monsoon.
thunderstorm may (c)/ continue throughout (d)/ No error (e)
today. (d)/ No error (e)
Q790. In our experience people usually (a)/
Q781. The cruel king believed (a)/ in value things that they have to (b)/
punishing the prisoners (b)/ until they did
Use Code: (AVP10) for 10% India’s No.1 Teacher in Bank Exams for English Language and Editorial 60
additional discount on All Website: www.vishalparihar.com | Follow on Instagram: @vishalthetrainer
Unacademy courses
1000+ Error Detection for All Exams
Last 20 years (Previous Year Papers)
English By Vishal Parihar

pay off more than those that (c)/ they receive


free of cost. (d)/ No error (e) Q799. Unless it is accepted to both the parties,
an (a)/ arbitrator would be of no (b)/ use to
settle this
Q791. At present China is the (a)/ world‘s dispute. (c)/ No Error (d)
leader manufacturer (b)/ of environment
friendly products (c)/ such as electric cars Q800. Although the manager was keen on
and bi cycles. (d)/ No error (e) getting the work (a)/ done through Sudhir
yesterday, (b)/ he
Q792. Over eighty per cent from us (a)/ feel tries to avoid it. (c)/ No Error (d)
that if we had taken (b)/ some corrective
measures earlier (c)/ the crisis could have
Q801. The various consequences of (a)/ the
been averted. (d)/ No error (e)
decision taken by the (b)/ finance minister was
not
Q793. The manager of that city branch (a)/
foreseen by the bureaucrats. (c)/ No Error (d)
cannot handle it with the help of (b)/ only two
personnel as (c)/ business has increased
substantially. (d)/ No error (e) Q802. I never considered him to be a person
who would (a)/ go back on his promise (b)/ and
then do not
Q794. You must either (a)/ be regular with your even apologies. (c)/ No Error (d)
studies (b)/ and study for longer period before
the Q803. Having finished at school (a)/ Raghu
examination. (c)/ No Error (d) thought of going to Bombay in (b)/ search of
some job. (c)/
Q795. The new taxation rates (a)/ announced by No Error (d)
the government (b)/ are bound to effect the
export 804. More persons than one (a)/ has been
sector. (c)/ No Error (d) involved (b)/ in this quarrel. (c)/ No Error (d)

Q796. These days, job opportunity are not as 805. I met the two girls, (a)/ whom I believe, (b)/
better (a)/ as they used to be (b)/ in the early are twin-sisters. (c)/ No Error (d)
70’s. (c)/ No
Error (d) 806. He entered (a)/ into his (b)/ new house last
Sunday. (c)/ No Error (d)
Q797. When viewed with his point of view, the
(a)/ entire episode assumes (b)/ a different 807. Even many people (a)/ carry an infection
colour (b)/ without showing its symptoms. (c)/ No
together. (c)/ No Error (d) Error (d)

Q798. On many occasions (a)/ we did helped the 808. They will be coming (a)/ to attend the
poor (b)/ people by way of giving them food and farewell party, (b)/ isn’t it? (c)/ No Error (d)
clothes to put on. (c)/ No Error (d)
Use Code: (AVP10) for 10% India’s No.1 Teacher in Bank Exams for English Language and Editorial 61
additional discount on All Website: www.vishalparihar.com | Follow on Instagram: @vishalthetrainer
Unacademy courses
1000+ Error Detection for All Exams
Last 20 years (Previous Year Papers)
English By Vishal Parihar

809. When shall we (a)/ arrive (b)/ to our 821. It is high time (a)/ he stood on his own (b)/
destination? (c)/ No Error (d) two legs. (c)/ No Error (d)

810. Based on the newspaper reports, (a)/ we 822. Knowledge of (a)/ at least two languages
can conclude that (b)/ many accidents caused (b)/ are required to pass the examination. (c)/
by No Error(d)
reckless driving. (c)/ No Error (d)
821. When I arrived (a)/ I found them both
811. Females (a)/ are not appointed (b)/ in our sitting at the table (b)/ waiting on me. (c)/ No
college. (c)/ No Error (d) Error (d)

812. The officer (a)/ is angry on the clerk (b)/ for 822. I saw him (a)/ a couple of times (b)/ since
not attending to the work. (c)/ No Error (d) May. (c)/ No Error (d)

813. No sooner (a)/ I had spoken, (b)/ than he 823. Tea (a)/ which I am drinking (b)/ is hot. (c)/
left. (c)/No Error (d) No Error (d)

814. Computer education (a)/ in universities 824. Although the police officer sympathized
and colleges today (b)/ leaves much to be with the poor (a)/ he refused to (b)/ take an
desired (c)/ action against
No Error (d) the rich man. (c)/ No Error (d)

815. You will be persecuted (a)/ for bringing 825. We were looking forward (a)/ to hear news
seeds (b)/ into Australia. (c)/ No Error (d) (b)/ about the missing fishermen. (c)/ No Error
(d)
816. You must either tell me (a)/ the whole or at
least (b)/ the first half of it. (c)/ No Error (d) 826. The actress (a)/ was shocked (b)/ by the
news of her dog’s death. (c)/ No Error (d)
817. Our new neighbours (a)/ had been living in
Arizona (b)/ since ten years before moving to 827. One of the questions (a)/ he asked me was
their present house. (c)/ No Error (d) (b)/ “Who did you travel with”? (c)/ No Error (d)

818. The patient was (a)/ accompanied (b)/ with 828. I know (a)/ a doctor you are (b)/ referring
his friend. (c)/ No Error (d) to. (c)/ No Error (d)

819. If I was you (a)/ I would not attend (b)/ the 829. The introduction of job oriented curse (a)/
function. (c)/ No Error (d) in the self-financing colleges (b)/ attract many
students.
820. He did (a)/ nothing else (b)/ than play. (c)/ (c)/ No Error (d)
No Error (d)

Use Code: (AVP10) for 10% India’s No.1 Teacher in Bank Exams for English Language and Editorial 62
additional discount on All Website: www.vishalparihar.com | Follow on Instagram: @vishalthetrainer
Unacademy courses
1000+ Error Detection for All Exams
Last 20 years (Previous Year Papers)
English By Vishal Parihar

830. It is better (a)/ to keep one’s head in the 842. Your success in the IAS examinations
face of danger than (b)/ losing one’s courage. depends not only on what (a)/ papers you have
(c)/ No selected (b)/
Error (d) but on how you have written them. (c)/ No Error
(d)
831. The sort story (a)/ should not exceed (b)/
more than two hundreds words. (c)/ No Error 843. Heavy rain (a)/ prevented us (b)/ to go to
(d) the cinema. (c)/ No Error (d)

832. A city dweller finds it difficult (a)/ to pass 844. If the majority of the individuals in a State
away the time (b)/ in a village. (c)/ No Error (d) (a)/ prosper (b)/ the State itself would prosper.
(c)/
833. Visitors (a)/ were not permitted (b)/ No Error (d)
entering the park after dark. (c)/ No Error (d)
845. If motorists do not observe the traffic
834. The fifth and final act (a)/ of Macbeth regulations, (a)/ they will be stopped, ticketed
contain (b)/ the sleep walking scene. (c)/ No and (b)/ have
Error (d) to be pay a fine. (c)/ No Error (d)

835. One of the terrorists (a)/ of the Kashmir 846. He asked (a)/ supposing if he fails (b)/ what
valley (b)/ are shot dead. (c)/ No Error (d) he would do. (c)/ No Error (d)

836. Ten kilometers (a)/ is (b)/ a long distance 847. The people of ancient India (a)/ were more
to walk. (c)/ No Error (d) civilized (b)/ than ancient Europe. (c)/ No Error
(d)
837. To die with honour (a)/ is better than (b)/
live with dishonour. (c)/ No Error (d) 848. The list of the names (a)/ of the tax
defaulters (b)/ were published in the
838. It is I (a)/ who is to blame (b)/ for this bad newspaper. (c)/ No
situation. (c)/ No Error (d) Error (d)

839. Gowri told me (a)/ his name after (b)/ he 849. One of my firmest belief is (a)/ that nothing
left. (c)/ No Error (d) can (b)/ substitute hard work. (c)/ No Error (d)

840. John would have told (a)/ you the truth (b)/ 850. People who are not belonging to (a)/ any
if you had asked him. (c)/ No Error (d) nationalist party (b)/ want to divide the country.
(c)/ No Error (d)
841. My sister (a)/ read (b)/ by pages after pages
of the Bible. (c)/ No Error (d) 851. Neither the principal nor the students (a)/
takes interest in the (b)/ general welfare of the
students. (c)/ No Error (d)

Use Code: (AVP10) for 10% India’s No.1 Teacher in Bank Exams for English Language and Editorial 63
additional discount on All Website: www.vishalparihar.com | Follow on Instagram: @vishalthetrainer
Unacademy courses
1000+ Error Detection for All Exams
Last 20 years (Previous Year Papers)
English By Vishal Parihar

852. We have received a message (a)/ from the (c)/ No Error (d)
Minister for Education (b)/ only yesterday. (c)/
No Error (d) 864. I have (a)/ seen her only once (b)/ but I’m
liking her a lot. (c)/ No Error (d)
853. He is writing novels (a)/ ever since (b)/ he
became a graduate. (c)/ No Error (d) 865. I told him (a)/ that we enjoyed very much
(b)/ at the party. (c)/ No Error (d)
854. I asked him (a)/ why did he call (b)/ me a
fool. (c)/ No Error (d) 866. John is working very hardly (a)/ as the
examinations (b)/ are fast approaching. (c)/ No
855. I was surprised (a)/ when he told me (b)/ Error (d)
how much does he study every day. (c)/ No 867. He ate (a)/ nothing (b)/ since yesterday.
Error (d) (c)/ No Error (d)

856. The Government had decided (a)/ to 868. An experimental vaccine (a)/ has brought
decrease the prices (b)/ for rice and wheat. (c)/ (b)/ glimmer of hope for malarial researcher.
No Error (d) (c)/ No
Error (d)
857. Two hours have passed (a)/ since (b)/ he
had fallen asleep. (c)/ No Error (d) 869. After making me wait for two organizing
hours (a)/ the great man called men in (b)/ and
858. Having broken down (a)/ the driver sent asked
the car (b)/ to the garage. (c)/ No Error (d) me what do I want. (c)/ No Error (d)

859. He is one of those writers (a)/ who has won 870. The ebb and flow of the tides (a)/ are (b)/
acclaim (b)/ the world over. (c)/ No Error (d) now understood. (c)/ No Error (d)

860. The mason will not (a)/ do the work (b)/ 871. The green paint of the wall (a)/ provides a
except give the order. (c)/ No Error (d) suitable contrast (b)/ with the yellow doors. (c)/
No Error (d)
861. When students are ill (a)/ they find that
they have a lot of work (b)/ to catch up when 872. He was (a)/ bent upon (b)/ to murder him.
they return. (c)/ No Error (d)
(c)/ No Error (d)
873. You will (a)/ see the match tomorrow (b)/
862. Scarcely did I reach the airport, (a)/ isn’t it? (c)/ No Error (d)
nervous and tense, than the plane took off (b)/
leaving me 874. I made him (a)/ apologize (b)/ for his
stranded in an alien place. (c)/ No Error (d) rudeness. (c)/ No Error (d)

863. The power to (a)/ distinguish between 875. We can decide it (a)/ only after (b)/ farther
differences (b)/ is the basis of science and art. investigation. (c)/ No Error (d)
Use Code: (AVP10) for 10% India’s No.1 Teacher in Bank Exams for English Language and Editorial 64
additional discount on All Website: www.vishalparihar.com | Follow on Instagram: @vishalthetrainer
Unacademy courses
1000+ Error Detection for All Exams
Last 20 years (Previous Year Papers)
English By Vishal Parihar

888. Everyone of the films (a)/ you suggested


876. I am hearing that (a)/ the meeting is (b)/ (b)/ are not worth seeing. (c)/ No Error (d)
likely to be postpone. (c)/ No Error (d)
889. The Secretary and the Principal of the
878. Even today (a)/ it is incredulous to think college (a)/ are attending (b)/ the district
(b)/ that men have walked on the moon. (c)/ No Development
Error (d) council Meeting at the Collectorate. (c)/ No
Error (d)
879. The firemen wore (a)/ inflammable
clothing (b)/ for protection. (c)/ No Error (d) 890. No sooner had the hockey match started
(a)/ when it began (b)/ to rain. (c)/ No Error (d)
880. Beside food, (a)/ the pilgrims carried (b)/ 891. The Secretariat (a)/ comprise of (b)/ many
some medicines. (c)/ No Error (d) air conditioned rooms. (c)/ No Error (d)

881. Adult suffering chicken pox (a)/ can 892. It is high time (a)/ he stood on his own (b)/
develop (b)/ all kinds of complications. (c)/ No two legs. (c)/ No Error (d)
Error (d)
893. You should avoid (a)/ to travel (b)/ in the
882. The well-known pianist (a)/ had to practice rush hour. (c)/ No Error (d)
for several hours a day (b)/ even after he rose to
fame. (c)/ No Error (d) 894. There is (a)/ only one of his novels (b)/ that
are interesting. (c)/ No Error (d)
883. The Prime Minister was asked (a)/ to write
a forward (b)/ to the book. (c)/ No Error (d) 895. He denied (a)/ to have (b)/ been there. (c)/
No Error (d)
884. I must complement you (a)/ on your good
manners (b)/ and in your impeccable behaviour. 896. Knowledge of (a)/ at least two languages
(c)/ No Error (d) (b)/ are required to pass the examination. (c)/
No
885. The sweets (a)/ were shared (b)/ between Error (d)
the four girls. (c)/ No Error (d)
897. The members of the Opposition Party in
886. The long distance train (a)/ which met with the Parliament (a)/ shout upon the minister (b)/
an accident (b)/ was carrying some army if he makes a wrong statement (c)/ No Error (d)
personnel.
(c)/ No Error (d) 898. If the worst (a)/ comes worst, (b)/ I will
have to bid good bye to my studies and join my
887. The young men from Japan (a)/ found the family
assent of the mountain (b)/ hard going. (c)/ No business. (c)/ No Error (d)
Error (d)
899. The interim report does not (a)/ analyze
thoroughly the principle causes (b)/ of the
Use Code: (AVP10) for 10% India’s No.1 Teacher in Bank Exams for English Language and Editorial 65
additional discount on All Website: www.vishalparihar.com | Follow on Instagram: @vishalthetrainer
Unacademy courses
1000+ Error Detection for All Exams
Last 20 years (Previous Year Papers)
English By Vishal Parihar

disaster. (c)/ No Error (d)


911. A hot and (a)/ a cold spring (b)/ was found
900. The items I liked most (a)/ were the near each other. (c)/ No Error (d)
rosewood carvings (b)/ and the teak wood
furnitures of 912. All doubts are cleared (a)/ between (b)/
Dutch design. (c)/ No Error (d) you and I. (c)/ No Error (d)

901. A part of the training (a)/ they offered was 913. Either of the roads (a)/ lead (b)/ to the
(b)/ real good. (c)/ No Error (d) park. (c)/ No Error (d)

902. The soil was moist as (a)/ as there was little 914. I went to the temple (a)/ with my parents,
rain (b)/ the day before. (c)/ No Error (d) my aunts (b)/ and my cousin sisters. (c)/ No
Error (d)
903. The manager wanted to know who had
arrived (a)/ early that day (b)/ the cashier or the 915. I have passed (a)/ the examination (b)/ two
accountant. (c)/ No Error (d) years ago. (c)/ No Error (d)

904. They are a politically important family; (a)/ 916. The earth moves (a)/ round the Sun (b)/
one of his sister is a minister (b)/ and another is isn’t it? (c)/ No Error (d)
married with a minister. (c)/ No Error (d)
917. Unless you do not give (a)/ the key of the
905. Supposing that the information (a)/ proves safe (b)/ you will be shot. (c)/ No Error (d)
to be totally reliable, (b)/ should we still have to
recast the plan? (c)/ No Error (d) 918. A milk (a)/ can provide protein (b)/ for a
nutritionally balanced diet. (c)/ No Error (d)
906. Though its gloss can (a)/ fool few unwary
customers, (b)/ it wouldn’t be difficult for the 919. When the football walked onto he field (a)/
clever ones to judge its real worth. (c)/ No Error the crowd burst into applause, (b)/ but some
(d) cheers
were heard too. (c)/ No Error (d)
907. She pretends as if she has (a)/ never in her
life, (b)/ told a lie, isn’t it? (c)/ No Error (d) 920. A lot of form-filling is just red tape, (a)/ and
sometimes the forms are so complicated (b)/
908. If there are billions of stars in the galaxy, that people can’t make head or tail from them.
(a)/ how much are (b)/ suitable for life? (c)/ No (c)/ No Error (d)
Error (d)
921. You are required to explain (a)/ your
909. The value of the dollar (a)/ declines as the conduct within two days (b)/ for the receipt of
rate (b)/ of inflation raises. (c)/ No Error (d) this
letter. (c)/ No Error (d)
910. One of my desires (a)/ are to become (b)/
a doctor. (c)/ No Error (d)
Use Code: (AVP10) for 10% India’s No.1 Teacher in Bank Exams for English Language and Editorial 66
additional discount on All Website: www.vishalparihar.com | Follow on Instagram: @vishalthetrainer
Unacademy courses
1000+ Error Detection for All Exams
Last 20 years (Previous Year Papers)
English By Vishal Parihar

922. I now (a)/ am knowing (b)/ all the facts 933. Being (a)/ a rainy day (b)/ I could not go
about him. (c)/ No Error (d) out. (c)/ No Error (d)

923. We had a lot of difficulty (a)/ to find (b)/ 934. He is (a)/ capable to do this work (b)/
the house. (c)/ No Error (d) within the stipulated period. (c)/ No Error (d)

924. Patience as well as perseverance (a)/ are 935. I’ll send my servant (a)/ but I can’t insure
necessary (b)/ for success. (c)/ No Error (d) (b)/ that he’ll reach there in time. (c)/ No Error
(d)
925. The passer-by told us (a)/ where was the
marriage hall (b)/ and even led us to it. (c)/ No 936. The lawyer asked the complainant (a)/ to
Error (d) put his sign (b)/ on the paper. (c)/ No Error (d)

926. The increase in consumption is directly (a)/ 937. When you have gone through the papers,
proportional to the increase (b)/ in income. (c)/ (a)/ kindly return them (b)/ to us. (c)/ No Error
No Error (d) (d)

927. In Singapore (a)/ my brother-in-law with 938. You could be better off these days (a)/ and
his wife (b)/ were present at the function. (c)/ this enables you (b)/ to have little more fun. (c)/
No No
Error (d) Error (d)

928. Scarcely had (a)/ I arrived than (b)/ the 939. In that wholesale shop, (a)/ they do not sell
train left. (c)/ No Error (d) (b)/ fewer than ten bags of rice. (c)/ No Error (d)

929. The reason why (a)/ he was rejected (b)/ 940. The period (a)/ between 1980 to 1990 (b)/
was because he was too young. (c)/ No Error (d) was very significant in my life. (c)/ No Error (d)

930. Teachers of various schools (a)/ met to 941. He isn’t the fastest bowler (a)/ in the Indian
discuss about (b)/ how to improve the standard team, (b)/ isn’t he? (c)/ No Error (d)
of
English. (c)/ No Error (d) 942. I went into (a)/ the bank and (b)/ some
withdrew money. (c)/ No Error (d)
931. His-tradition bound attitude (a)/ had to be
a constant source of dissatisfaction (b)/ among 943. The strain of all (a)/ the difficulties and
the younger members of the family. (c)/ No vexations and anxieties (b)/ are more than he
Error (d) could beat. (c)/ No Error (d)

932. The two first to arrive (a)/ were the lucky 944. In the background they could hear John
recipients (b)/ of surprise gift. (c)/ No Error (d) laughing and joking loudly. (a)/ John was the

Use Code: (AVP10) for 10% India’s No.1 Teacher in Bank Exams for English Language and Editorial 67
additional discount on All Website: www.vishalparihar.com | Follow on Instagram: @vishalthetrainer
Unacademy courses
1000+ Error Detection for All Exams
Last 20 years (Previous Year Papers)
English By Vishal Parihar

life and soul of any party (b)/ and he was (c)/ No Error (d)
cracking a joke every few minutes. (c)/ No Error
(d) 956. The introduction of tea and coffee (a)/ and
such other beverages (b)/ have not been
945. I was thinking (a)/ if I could do (b)/ anything without some effect. (c)/ No Error (d)
to help. (c)/ No Error (d)
957. In spite of the roadblock (a)/ the guard
946. I meant nothing (a)/ less than (b)/ to allowed us (b)/ enter the restricted area to
compel you to come. (c)/ No Error (d) search for
our friends. (c)/ No Error (d)
947. Females (a)/ are not appointed (b)/ in our
college. (c)/ No Error (d) 958. The newer type of automatic machines
(a)/wash (b)/ the clothes faster. (c)/ No Error (d)
948. He has read four plays (a)/ written by
Shakespeare (b)/ by the end of his vacation. (c)/ 959) In June 2017, an anodyne footnote to the
No Error (d) Bank of Baroda’s (BoB) quarter results
mentioned a (1)/ fine levied by the South
949. The officer (a)/ is angry on the clerk (b)/ for African Reserve Bank (SARB), (2)/
not attending to the work. (c)/ No Error (d) headquartered in Pretoria. (3)/ No error (4)
1) 1
950. Banks were developed to keep people’s
2) 2
money safe (a)/ and to make it available (b)/
3) 3
when
they need it. (c)/ No Error (d) 4) 4

951. Based on the newspaper reports (a)/ we 960) No sooner did I come out of my home to
can conclude that (b)/ many accidents caused go to market (1)/ when it started raining heavily
by (2)/ which drenched me completely. (3)/ No
reckless driving. (c)/ No Error (d) Error (4)
1) 1
952. He walked (a)/ till the (b)/ end of these 2) 2
street. (c)/ No Error (d) 3) 3
4) 4
953. The whole country (a)/ was suffering by
(b)/ a financial crisis. (c)/ No Error (d)
961) Five summons have been (1)/ issued by the
district court (2)/ but he has not turned up yet.
954. I do not know where could he have gone
(a)/ so early (b)/ in the morning. (c)/ No Error (d) (3)/ No Error (4)
1) 1
955. I suggest that (a)/ he goes (b)/ to the doctor 2) 2
as soon as he returns from taking the
examination 3) 3
Use Code: (AVP10) for 10% India’s No.1 Teacher in Bank Exams for English Language and Editorial 68
additional discount on All Website: www.vishalparihar.com | Follow on Instagram: @vishalthetrainer
Unacademy courses
1000+ Error Detection for All Exams
Last 20 years (Previous Year Papers)
English By Vishal Parihar

4) 4 or content that rides over those networks. (3)/


No error (4)
1) 1
962) Acceptance is not about allowing 2) 2
everything to occur or to go on, (1)/ it is 3) 3
neither related to passivity and weakness, (2)/ 4) 4
nor is it about confirmation or mediocrity. (3)/
No error (4)

967) Liquid water cannot exist on the surface of


1) 1
Mars as to low (1)/ atmospheric pressure, which
2) 2
is less than 1% of Earth’s, (2)/ except at the
3) 3
lowest elevations for short periods. (3)/ No
4) 4
error (4)
1) 1
963) She was (1)/ quite amusing (2)/ at that
2) 2
story. (3)/ No error (4)
3) 3
1) 1
4) 4
2) 2
3) 3
968) It was (1)/ difficult decided (2)/ about
4) 4
break up. (3)/ No error (4)
1) 1
2) 2
964) She picked (1)/ on the (2)/ trail soon after.
3) 3
(3)/ No error (4)
4) 4
1) 1
2) 2
969) This year, we were visited by the director
3) 3
of Austrian Culture Forum, (1)/ based on Delhi
4) 4
and together we all wanted (2)/ to initiate an art
festival in South India. (3)/ No error (4)
965) She has (1)/ reduced her (2)/
1) 1
consumption to considerably. (3)/ No error (4)
2) 2
1) 1
3) 3
2) 2
4) 4
3) 3
4) 4
970) I seen (1)/ that it (2)/ is getting dark. (3)/
No error (4)
966) Internet Service Providers (ISPs) should
1) 1
provide us (1)/ with open networks and should
2) 2
not block or discriminate for (2)/ any application
3) 3
Use Code: (AVP10) for 10% India’s No.1 Teacher in Bank Exams for English Language and Editorial 69
additional discount on All Website: www.vishalparihar.com | Follow on Instagram: @vishalthetrainer
Unacademy courses
1000+ Error Detection for All Exams
Last 20 years (Previous Year Papers)
English By Vishal Parihar

4) 4 975) No sooner had she arrive that her (1)/


friends arranged a reception in her (2)/ honour
971) The long-term capital gains on listed in the best resort in the town. (3)/ No Error (4)
securities (1)/ and the dividend tax on equity [SSC CPO Spotting Error 01-july-2017]
mutual funds are the right steps (2)/ to generate 1) 1
resources required for drive demand in rural 2) 2
India. (3)/ No error (4) 3) 3
1) 1 4) 4
2) 2
3) 3 976) During India’s struggle for (1)/ freedom
4) 4 many brilliant students (2)/ gave up their
studies by Bhagat Singh’s call. (3)/ No Error (4)
972) The first and decisive step (1)/ in the 1) 1
expansion of Europe overseas (2)/ was the 2) 2
conquest of Atlantic Ocean. (3)/ No error (4) 3) 3
1) 1 4) 4
2) 2
3) 3 977) Madhuri and I have done my (1)/ work
4) 4 patiently and diligently (2)/ just for our safe and
secure future. (3)/ No Error (4)
973) The word ‘Macabre’ had been gained its 1) 1
significance from its (1)/ use in French as 2) 2
ladanse macabre for the allegorical (2)/ 3) 3
representation of the universal power of 4) 4
death. (3)/ No error (4)
1) 1 978) Sunita is feeling little uncomfortable today
2) 2 (1)/ because she is ill, so she can perform (2)/
3) 3 her work after having some rest for a while. (3)/
4) 4 No Error (4)
1) 1
974) Our professor taught us that (1)/ one 2) 2
should respect the (2)/ religions of others as 3) 3
much as his own. (3)/ No Error (4) [01-july- 4) 4
2017]
1) 1 979) We must (1)/ denounce justice (2)/ and
2) 2 oppression. (3)/ No Error (4)
3) 3 1) 1
4) 4 2) 2
3) 3
Use Code: (AVP10) for 10% India’s No.1 Teacher in Bank Exams for English Language and Editorial 70
additional discount on All Website: www.vishalparihar.com | Follow on Instagram: @vishalthetrainer
Unacademy courses
1000+ Error Detection for All Exams
Last 20 years (Previous Year Papers)
English By Vishal Parihar

4) 4 4) 4

980) I don’t think, Neha is as wiser as (1)/ Sonali, 985) Had she not known (1)/ about it she should
so she is not (2)/ competent for this job. (3)/ No (2)/ have stayed longer. (3)/ No Error (4)
Error (4) 1) 1
1) 1 2) 2
2) 2 3) 3
3) 3 4) 4
4) 4
986) If you will finish your homework I (1)/ will
981) In the Mathematics class, I sit next to (1)/ give you (2)/ an ice-cream. (3)/ No Error (4)
Raj, who is the captain of the basket ball (2)/ 1) 1
team and undoubtedly the best basket ball 2) 2
player in Delhi. (3)/ No Error (4) 3) 3
1) 1 4) 4
2) 2
3) 3 987) Mr. Gupta with his wife (1)/ and younger
4) 4 brother (2)/ were present at the station. (3)/ No
Error (4)
982) He commanded me (1)/ as if he was (2)/ 1) 1
my husband. (3)/ No Error (4) 2) 2
1) 1 3) 3
2) 2 4) 4
3) 3
4) 4 988) Rajni is planning to settle (1)/ in Mumbai
as soon as (2)/ she retires in August the next
983) The English is a very (1)/ popular language year. (3)/ No Error (4)
(2)/ amongst south Indians. (3)/ No Error (4) 1) 1
1) 1 2) 2
2) 2 3) 3
3) 3 4) 4
4) 4
989) I was born (1)/ on 14th July, 1990 (2)/ in
984) This container is full with water (1)/ so I Allahabad. (3)/ No Error (4)
can’t carry it for a (2)/ long distance at one go. 1) 1
(3)/ No Error (4) 2) 2
1) 1 3) 3
2) 2 4) 4
3) 3
Use Code: (AVP10) for 10% India’s No.1 Teacher in Bank Exams for English Language and Editorial 71
additional discount on All Website: www.vishalparihar.com | Follow on Instagram: @vishalthetrainer
Unacademy courses
1000+ Error Detection for All Exams
Last 20 years (Previous Year Papers)
English By Vishal Parihar

990) Not only we saw (1)/ the President but 995) Because copper prices are going down (1)/
also (2)/ the Chief Minister of Delhi. (3)/ No demand for alternative (2)/ products to copper
Error (4) are lessening. (3)/ No Error (4)
1) 1 1) 1
2) 2 2) 2
3) 3 3) 3
4) 4 4) 4

991) Can you imagine (1)/ that she has thirty 996) Her aunt didn’t (1)/ give me (2)/ a minute
(2)/ five pair of shoes. (3)/ No Error (4) of peace. (3)/ No Error (4)
1) 1 1) 1
2) 2 2) 2
3) 3 3) 3
4) 4 4) 4

992) Not only her husband (1)/ but even her 997) Aayushi has come (1)/ here to do a master
father (2)/ also find her selfish. (3)/ No Error (4) (2)/ degree in Social Work. (3)/ No Error (4)
1) 1 1) 1
2) 2 2) 2
3) 3 3) 3
4) 4 4) 4

993) The West Indies team had (1)/ not won 998) There is no money in (1)/ the bank in
any match (2)/ last year. (3)/ No Error (4) Rajesh’s and (2)/ Reena’s joint account. (3)/ No
1) 1 Error (4)
2) 2 1) 1
3) 3 2) 2
4) 4 3) 3
4) 4
994) Hardly had he come out of the (1)/
cinema hall then the bomb exploded (2)/ and
shattered the hall completely. (3)/ No Error (4) 999) The CEO has decided to visit all the (1)/
1) 1 departments of the office tomorrow (2)/
2) 2 evening to review of the situation. (3)/ No Error
3) 3 (4)
4) 4 1) 1
2) 2
3) 3

Use Code: (AVP10) for 10% India’s No.1 Teacher in Bank Exams for English Language and Editorial 72
additional discount on All Website: www.vishalparihar.com | Follow on Instagram: @vishalthetrainer
Unacademy courses
1000+ Error Detection for All Exams
Last 20 years (Previous Year Papers)
English By Vishal Parihar

4) 4 replaced with ‘interference’ as we need a


noun.
1000) Neha is a very good fashion (1)/ designer 6. Answer: A in part c in place of take in it
but her designs are not (2)/ easily accessible should be take down. Take in means to allow
admittance while take down means the
with the public. (3)/ No Error (4)
removal of a website, web page, or file from
1) 1 the Internet,
2) 2 7. Answer: B ‘have been’ should be replaced
3) 3 with ‘have’ as the sentence is in active voice
4) 4 [HAVE + V3 = ACTIVE/ HAVE + BEEN + V3 =
PASSIVE]
1001) The two men were (1)/ quarrelling with 8. Answer: C in part a in place of Biotechit
one another (2)/ claiming the same watch as should be Biotech’s and in part b in place of
shows it should be showed
their own. (3)/ No Error (4) .
9. Answer: E no error
1) 1 10. Answer: D in part d in place of give it
2) 2 should be giving
3) 3 11. Answer: C Explanation- in part c in place
4) 4 of uplift it should be uplifting and in part d in
place of platform it should be platforms
12. Answer: B ‘rule’ should be replaced with
SOLUTION: ‘be ruled’; here ‘the risk’ is the object as the
first part of the sentence is in passive voice.
1. Answer: E All the parts are grammatically [CANNOT +V1 = ACTIVE/ CANNOT+ BE + V3
correct. = PASSIVE]
2. Answer: A Part C is incorrect; here “hold” 13. Answer: E The sentence is correct.
should be replaced with “held” as after “must 14. Answer: C Part B is incorrect; here
be” we should use V3 (“held”). “separate” should be replaced with
3. Answer: C Part B is incorrect; here “separately” as to describe a Verb (“see”) we
“separate” should be replaced with should use an adverb(“separately”). Part C is
“separately” as to describe a Verb (“see”) we incorrect; “and” should be omitted from this
should use an adverb(“separately”). Part C is part as the pair of “not just” is “but/But that”.
incorrect; “and” should be omitted from this 15. Answer: B Fragment I is erroneous as, it
part as the pair of “not just” is “but/But that”. uses there, which is incorrect since while
4. Answer: C Part B is incorrect; here “low” defining democratic barbarism when a
should be replaced with “lower” as after “to” comma is used it is generally followed with
a verb(lower) should be used. Part D is in where, thus, is incorrect. Fragment III is
correct; here “make” should be replaced with incorrect as, it uses incorrect preposition in
“makes” as with a singular subject the judiciary and should get replaced by “of
“suggestion” a V5(makes) should be used. the ….” No other fragment has errors.
5. Answer: C Part B is incorrect, ‘to’ should be Therefore, option B is the correct answer.
omitted. Rule: HELP + V1 (WITHOUT ‘TO’) 16. Answer: D Fragment IV is incorrect as it
Part C is incorrect; ‘interfere’ should be uses inappropriate form of the word
infraction, which according to the context
Use Code: (AVP10) for 10% India’s No.1 Teacher in Bank Exams for English Language and Editorial 73
additional discount on All Website: www.vishalparihar.com | Follow on Instagram: @vishalthetrainer
Unacademy courses
1000+ Error Detection for All Exams
Last 20 years (Previous Year Papers)
English By Vishal Parihar

needs to be in plural form, infractions. No 24. Answer: B More than Most leaders, Boris
other fragments have any error. Therefore, Johnson relies on the people around him, for
option D is the correct answer. his positive qualities are counterbalanced by
17. Answer: C Fragment III is erroneous as, it negative ones Rely on is used in context of a
uses incorrect preposition at the policy and living being; Rely upon is used in context of a
should be replaced with in the policy, to make non living thing For e.g - You can rely on me;
the sentence contextually correct. No other You can rely upon the fact
fragments have an error. Therefore, option C 25. Answer: D Biden is engaged in a grand
is the correct answer. relaunch of his administration after a
18. Answer: D Fragment D is erroneous as, it disaster-prone 11 months since the election.
uses incorrect adverb ‘for’ prior to ‘exit’ and Although both these words present the
should be replaced with ‘to’, in order to make starting time of an action, they cannot be used
the sentence grammatically correct. All other interchangeably. The doctor will be here from
fragments are error free Therefore, option D 10 am tomorrow – Correct The doctor will be
is the correct answer. here since 10 am tomorrow – Incorrect The
19. Answer: D Fragment D is erroneous as, a doctor has been waiting for you since 10 am –
determiner ‘that’ is missing in between words Correct The doctor has been waiting for you
‘established’ and ‘the’, and it should be used from 10 am – Incorrect
to make the sentence contextually correct. All 26. Answer: B As the public, governments and
other fragments are error free. Therefore, investors wake up to climate change, the
option D is the correct answer. clean-energy industry is gaining momentum.
20. Answer: C Fragment C is erroneous, as it Article “THE” before investors is redundant.
uses incorrect preposition ‘they’, to represent Since we already have “THE” before the
a Board. It should be correctly replaced with “PUBLIC” usage of article “THE” again before
‘it’ to match with the context All other investors is unnecessary.
fragments are error free. Therefore, option C 27. Answer: B Mr Biden’s most daunting
is the correct answer. immediate domestic task will surely be
21. Answer: B Fragment B is erroneous as, it dealing with covid-19—despite this week’s
uses incorrect preposition ‘of’ prior to further good news about vaccines. Here Only
‘health’. It should be replaced with ‘for’, to one task i.e to deal with the virus spread. So
express possessiveness. All other fragments plural word “TASKS” is incorrect.
are error free. Therefore, option B is the 28. Answer: B Chhattisgarh Chief Minister
correct answer. 22. Answer: A Fragment A is Bhupesh Baghel will launch two key schemes
erroneous, as it uses incorrect article ‘the’ related to distribution of fortified rice and
prior to ‘massive’ and should be replaced with setting up of Swami Atmanand Government
‘a’ to represent a singular context to the English medium schools on Sunday during
overcrowd. All other fragments are error free. ''Rajyotsav'' statehood celebrations. “Related
Therefore, option A is the correct answer. to” is the correct usage.
23. Answer: E Sentence is correct as it is. As 29. Answer: E In a bid to strengthen capital
oil demand dwindles, they will face a vicious base, the union government has provided Rs
fight for market share which will be won by 670 crore to Regional Rural Banks
the countries with the cheapest and cleanest considering their importance in agriculture
crude. finance during these difficult times. Sentence
is correct as it is.
Use Code: (AVP10) for 10% India’s No.1 Teacher in Bank Exams for English Language and Editorial 74
additional discount on All Website: www.vishalparihar.com | Follow on Instagram: @vishalthetrainer
Unacademy courses
1000+ Error Detection for All Exams
Last 20 years (Previous Year Papers)
English By Vishal Parihar

30. Answer: B The Niti Aayog has released a 35. Answer: A ‘All in India’ is erroneous. Here
draft model Act and rules for states on in is incorrect preposition as it shows
conclusive land titling with an aim to reduce something inside India, but the context of the
litigations and ease the land acquisition sentence wanted to mean all that belong to
process for infrastructure projects. Titling is India. Thus, it should be replaced with of, to
the correct usage there. impart contextual correctness. Therefore,
31. Answer: C Fragment I is erroneous as, it option A is the correct answer.
uses have amassed,it is the incorrect form 36. Answer: B ‘For review’ is erroneous, since
referring a singular noun and should be the word for, is classified under prepositions
replaced with has amassed. Fragment III is when it is used to indicate the use of
incorrect as, it uses signed in incorrect tense, something, the location where an object or
and should be replaced with signs. Fragment person is going to, and to show the duration
II and IV are error-less Therefore, option C is of time. This is not in this case. It should be
the correct answer. replaced with to, can be used as a Preposition
32. Answer: B Fragment I is erroneous as, and as an Adverb. “To” can be considered as a
since most is incorrect to the tone and context preposition if it is used to indicate that a
of the sentence and should be replaced with noun/pronoun is moving towards something
while most. Fragment IV is erroneous as, Therefore, option B is correct answer.
there is the incorrect homonym and should 37. Answer: A Use ‘consider’ in the place of
get replaced with their Fragment II and III are ‘considers’ as “many” is a plural subject and it
error-less Therefore, option B is the correct should take V1.
answer. 38. Answer: A In part C, “which” is incorrect,
33. Answer: A Fragment II is erroneous as, in it should be replaced with “that”. Rule: “The
dominance is incorrect preposition of Same” + That (as relative pronoun). All parts
possession and should be replaced with of except C is correct.
dominance. Fragment IV is erroneous as, their 39. Answer: D In part C, “people” should be
style is incorrect use of pronoun, and should replaced with “person (singular noun)”. In
be replaced with his style. Fragment I and III Part D, “have” should be replaced with “has
are error-less. Therefore, option A is the (singular verb)” Rule: Many A + Singular noun
correct answer. + Singular verb
34. Answer: A Data since years is erroneous. 40. Answer: B In part B, “are” should be
Since is generally used to denote the replaced with “is”. Rule: The number of +
intervening period between (the time Plural noun + Singular verb.
mentioned) and the time under 41. Answer: A In part A, “normal (adjective)”
consideration, typically the present or for the should be replaced with “normally (adverb)”.
reason that; because, which is different with Rule: Adverb is used to describe Adjective and
respect to this context. It should be rightly Verb, it is placed before Adjective,
replaced with from indicating a source of “probusiness” is an adjective, hence,
knowledge or the basis for one's judgement “normally” (adverb) should be placed before
or indicating the point in space at which a it.
journey, motion, or action starts, suits 42. Answer: D Use ‘state’ in the place of
perfectly with the context. Therefore, option ‘states’.
A is the correct answer. 43. Answer: E no error

Use Code: (AVP10) for 10% India’s No.1 Teacher in Bank Exams for English Language and Editorial 75
additional discount on All Website: www.vishalparihar.com | Follow on Instagram: @vishalthetrainer
Unacademy courses
1000+ Error Detection for All Exams
Last 20 years (Previous Year Papers)
English By Vishal Parihar

44. The knowledge so far available about the correct usage is ‘promotion schemes have’
endocrane glands is very limited. 60.Option c, spell doom is the correct phrase
A.knowledge B.available C.endocrane meaning failure or end of something.
D.limited E.All Correct 61.Option b, laden with is the correct usage,
45. Answer: B endocrine– is the correct laden always follows the preposition ‘with’
spelling. 62.Option e, Roll out (officially launch) is
46. Answer: E wrong usage in the sentence, Roll in (receive
47. Answer: C Use ‘to’ – in place of – ‘for’. in large amounts) is the correct usage.
48. Answer: D Use ‘people’ – in place of – 63.Option e, no error. The sentence is entirely
‘peoples’ as “people” is considered as plural. negative so no article will be used before
49. Answer: E The sentence is correct. ‘little’
50. Answer: B expecting is erroneous in 64.Option d, the word ‘other’ is followed by
fragment B. This mentioned word is plural noun’ so correct usage is among other
grammatically incorrect tense and also refers benefits.
the university willing to get results, however 65.Option d, two nouns cannot exist together,
it is incorrect to the context as the context is they have to be separated by apostrophe to
hopeful that the university will deliver show the possession. the correct usage would
results. Thus it should be corrected with be region’s food habits.
expected. Therefore, option B is the correct 66.Option b, alliance takes preposition ‘with’
answer. and not of.
67. Option e , no error.
51.Option c, not only + but also is the correct 68.Option c, after comma always especially is
combination, ‘also’ is missing from c. so after used and not specially.
the corrections in statement c it will be ‘....but 69.Option b, there are two subjects so the
will also contribute...’ verb used should be plural in form , correct
52.Option c, article ‘the’ is missing before usage is ‘ loan are comparatively’
superlative degree, after the corrections it 70.Option c, on the ground is the correct
will be ‘one of the biggest’ 53.Option b, if the phrase meaning in a place where real or
name of the person is mentioned after the practical work is done.
designation, article ‘the’ is not used before 71.Option b, has+v3 structure has to be
designation. so omit ’the’ followed, the correct sentence would be ‘...has
54.Option d, Around the state is wrong usage issued..’
, it should be across the state 72.Option d, ‘on behalf of ‘ is the correct usage
55.Option e, no error and not for.
56.Option c, The pronoun used is incorrect, 73.Option a, The subject ‘auto sales’ is
the subject is singular (government) so singular, so ‘has slumped’ is the correct usage.
singular pronoun its is used. 74.Option e, no error.
57 Option a, Alternate means one after 75.Option d, the sentence is giving a sense of
another, so alternative means more than one past tense, so after the corrections the
choice, so alternative is the correct usage. sentence will be ‘...manpower required’
58.Option b, the subject for witness is India ,
which is singular so singular form of witness 76. c) Replace ‘show’ with ‘shows’
should be witnesses. 59.Option b, a number of 77. b) Replace ‘have’ with ‘have been’
+plural noun +plural verb is used. so the 78. c) Add ‘an’ before ‘Initiative’
Use Code: (AVP10) for 10% India’s No.1 Teacher in Bank Exams for English Language and Editorial 76
additional discount on All Website: www.vishalparihar.com | Follow on Instagram: @vishalthetrainer
Unacademy courses
1000+ Error Detection for All Exams
Last 20 years (Previous Year Papers)
English By Vishal Parihar

79. b) Replace ‘have’ with ‘has’ 121. d) Replace ‘overseen’ with ‘oversee’
80. a) Replace ‘speak’ with ‘speaking’ 122. a) Replace ‘plan’ with ‘plans’
81. d) Replace ‘need’ with ‘needed’ 123. e) No Error
82. d) Replace ‘on’ with ‘buy’ 124. d) Replace ‘recent’ with ‘recently’
83. c) Replace ‘demand’ with ‘demanded’ 125. c) Replace ‘it needs’ by ‘they need’
84. b) Replace ‘sad’ with ‘sadness’ 126. b) Replace ‘on’ by ‘in’
85. a) Place ‘certainly contain’ after ‘will’ 127. e) No Error
86. c) Replace ‘Independent’ with 128. a) Place ‘of’ after ‘reports’
‘Independently’ 129. d) Replace ‘with’ with ‘into’
87. e) No Error 130. e) No error
88. d) Replace ‘were’ with ‘was’ 131. a) Place ‘of’ after ‘robbed’
89. c) Replace ‘in’ with ‘on’ 132. c) Replace ‘on’ with ‘with’
90. b) Replace ‘eventual’ with ‘eventually’ 133. b) Place ‘been’ after ‘has’
91. c) Replace ‘to’ with ‘and’ 134. d) Replace ‘start’ with ‘starting’
92. b) Replace ‘year’ with ‘years’ 135. c) Replace ‘appear’ by ‘appears’
93. d) Replace ‘its’ with ‘their’ 136. e) No Error
94. b) Replace ‘within’ with ‘among’ 137. e) No error
95. c) Replace ‘large’ with ‘largely’ 138. d) Replace ‘since’ with ‘for’
96. c) Replace ‘to’ with ‘for’ 139. a) Replace ‘Over’ with ‘under’
97. b) Replace ‘shift’ with ‘shifted’ 140. d) Replace ‘respective’ with
98. a) Add ‘out’ after ‘carried’ ‘respectively’
99. d) Replace ‘frisk and question’ with 141. b) Replace ‘unlock’ with ‘unlocking’
‘frisked and questioned’ 142. d) Replace ‘exclude’ with ‘excluded’
100. a) Replace ‘is’ with ‘are’ 143. a) Replace ‘student’ with ‘students’
101. d) Replace ‘scratch’ with ‘scratched’ 144. c) Add ‘been’ after ‘had’
102. d) Replace ‘near’ with ‘nearly’ 145. d) Replace ‘lack’ with ‘lacked’
103. d) Replace ‘in’ with ‘of’ 146. d) Add ‘on’ after ‘going’
104. b) Replace ‘its’ with ‘their’ 147. a) Replace ‘give’ with ‘given’
105. c) Replace ‘on’ with ‘in’ 148. d) Replace ‘with’ by ‘against’
106. e) No error 149. b) Replace ‘alleged’ with ‘allegedly’
107. b) Replace ‘aspect’ with ‘aspects’ 150. c) Replace ‘were’ with ‘was’
108. d) Replace ‘has’ with ‘have’ 151. b) Replace ‘gave’ with ‘given’
109. a) Replace ‘is’ with ‘are’ 152. a) Replace ‘work’ with ‘works’
110. d) Add ‘to’ after ‘want’ 153. c) Replace ‘disgress’ with ‘disgressing’
111. c) Replace ‘sharp’ by ‘sharper’ 154. e) No Error
112. c) Replace ‘drive’ by ‘driven’ 155. e) No Error
113. c) Add ‘be’ after ‘to’ 156. b) Add ‘to’ after ‘victim’
114. d) Replace ‘at’ with ‘over’ 157. b) Place ‘a’ after ‘registered’
115. a) Add ‘a’ before ‘set’ 158. d) Place ‘to’ after ‘failing’
116. b) Replace ‘across’ with ‘against’ 159. d) Replace ‘since’ with ‘for’
117. b) Replace ‘focus’ with ‘focussing’ 160. c) Replace ‘with’ by ‘on’
118. d) Replace ‘include’ with ‘includes’ 161. d) Replace ‘between’ with ‘among’
119. a) Replace ‘emerge’ with ‘emerging’ 162. d) Replace ‘possess’ with ‘possession’
120. b) Add ‘an’ after ‘faces’ 163. c) Replace ‘includes’ with ‘include’
Use Code: (AVP10) for 10% India’s No.1 Teacher in Bank Exams for English Language and Editorial 77
additional discount on All Website: www.vishalparihar.com | Follow on Instagram: @vishalthetrainer
Unacademy courses
1000+ Error Detection for All Exams
Last 20 years (Previous Year Papers)
English By Vishal Parihar

164. c) Replace ‘are’ with ‘is’ 209. d) Replace ‘of’ with ‘in’
165. b) Interchange ‘three last’ to ‘last three’ 210. d) Replace ‘on’ with ‘by’
166. a) Replace ‘sapling’ with ‘saplings’ 211. a) Replace ‘were’ with ‘was’
167. c) Replace ‘beside’ with ‘under’ 212. a) Replace ‘case’ with ‘cases’
168. d) Place ‘of’ after ‘highlight’ 213. d) Replace ‘near’ with ‘nearly’
169. d) Replace ‘between’ with ‘from’ 214. c) Replace ‘official’ with ‘officially’
170. a) Add ‘an’ before ‘old’ 215. b) Replace ‘are’ with ‘is’
171. e) No Error 216. b) Interchange ‘two only’ to ‘only two’
172. b) Replace ‘offer’ with ‘offering’ 217. d) Replace ‘came’ with ‘come’
173. d) Replace ‘its’ with ‘their’ 218. d) Replace ‘live’ with ‘living’
174. c) Replace ‘enticing’ with ‘enticingly’ 219. d) Replace ‘photograph’ with
175. d) Replace ‘along’ with ‘under’ ‘photographing’
176. a) Interchange ‘not only’ and ‘teaches us’ 220. b) Replace ‘in’ with ‘upon’
177. c) Replace ‘in’ with ‘on’ 221. d) Replace ‘arise’ with ‘arisen’
178. b) Replace ‘substantial’ with 222. a) Replace ‘uphold’ with ‘upheld’
‘substantially’ 223. b) Replace ‘have’ with ‘has’
179. a) Replace ‘from’ with ‘since’ 224. e) No error
180. b) Replace ‘though’ with ‘through’ 225. c) Replace ‘along’ by ‘against’
181. d) Replace ‘initial’ with ‘initialled’ 226. c) Replace ‘less’ with ‘lesser’
182. e)No error 227. b) Replace ‘thorough’ with ‘thoroughly’
183. c) Replace ‘vigorous’ with ‘vigorously’ 228. c) Replace ‘lead’ with ‘leading’
184. d) Replace ‘exception’ with ‘exceptions’ 229. The subject of this phrase is ‘IDs’ which
185. e) No Error is a plural, which requires a plural
186. c) Interchange ‘attention drawing’ to demonstrative noun to represent itself.
‘drawing attention’ Therefore, ‘this’ is incorrect here. The correct
190. e) No Error option is B. A moot question is that, if IDs are
191. d) Replace ‘took’ with ‘taken’ considered to be the foremost (A)/ tools for
192. b) Replace ‘many’ with ‘much’ ensuring corporate governance, why have
193. c) Replace ‘place’ with ‘placed’ these non-compliant companies (B)/ been
194. d) Replace ‘consider’ with ‘considering’ allowed to remain listed and traded without
195. a) Replace ‘quote’ with ‘quoted’ (C)/ the safeguard of IDs, in many cases, for
196. a) Replace ‘note’ with ‘noting’ very long periods of time? (D)
197. d) Replace ‘hug’ by ‘hugging’
198. e) No error 230. ‘Numeral (one/two/etc) + of’ is always
199. a) Remove ‘about’ after ‘discussing’ followed by a plural noun. Therefore, it
200. d) Replace ‘be’ with ‘being’ should be ‘companies’ in (A). The correct
201. d) Add ‘up’ after ‘throw’ option is A. ONGC, one of the largest listed
202. c) Remove ‘back’ after ‘returning’ companies, citing lack of powers (A)/in
203. d) Add ‘been’ after ‘have’ deciding upon the board’s composition
204. e) No Error including the appointment of independent
205. c) Replace ‘if’ by ‘whether’ directors and board’s evaluation, has sought
206. a)Replace ‘start’ with ‘starts’ exemption from the related regulations on
207. c) Replace ‘bringing’ with ‘bring’ corporate governance.
208. a) Replace ‘around’ with ‘over’
Use Code: (AVP10) for 10% India’s No.1 Teacher in Bank Exams for English Language and Editorial 78
additional discount on All Website: www.vishalparihar.com | Follow on Instagram: @vishalthetrainer
Unacademy courses
1000+ Error Detection for All Exams
Last 20 years (Previous Year Papers)
English By Vishal Parihar

231. The subject of the sentence is jobs, and that is further pulling down the
‘compliance and disclosures’ which makes it a productive forces in a negative spiral. 8. No
plural. Therefore, the verb or helping verb correction required. The correct option is E
related to it should also be plural. ‘Fall’ should Three years ago , if State governments had the
be written here. The correct option is D. It premonition that embracing GST would lead
needs to be highlighted that (A)/compliance them to a situation wherein they would ha ve
and disclosures are far superior at the IPO to borrow to make good revenue shortfalls,
stage across many aspects, but fall they would have opted out of one tax regime.
significantly post listing. Asking States to borrow is precisely what has
232. ‘Nation’ is a non living thing. And it been recommended by the GST Council in its
cannot be followed by a possessive pronoun latest meeting. \
of ‘itself’. The correct option is D Coupled with 236. The sentence is a continuous tense
the uncertainty caused by a pandemic (A)/ sentence, but ‘entered’ here is a past.
that is still taking a growing toll, more Therefore ‘entering’ must be used here. The
gathering clouds and a global scenario that is correct option is B Just as organizations will
equally dismal, the nation has to brace itself be looking at minimizing costs and growing
for a long grind. revenues by increasing their geographic
footprint and entering new market segments
233. ‘Have’ is always followed by the 3rd form taking advantage of the digital
of verb. Therefore, ‘choosen’ must be written. transformation sweeping the world, families
The correct option is B Over the course of will need to operate as a business unit to
about five months since the maximise their its earning capacity
Covid19pandemic (A)/struck, policymakers 237. The subject of the sentence is ‘banks’
have chosen fiscal and monetary stimulus which is a plural. ‘Always’ when used after a
particularly , in the same manner as they did plural uses a verb in its basic form without ‘s’
in the aftermath of global financial crisis of after it. Therefore, ‘meet’ is correct here. The
2008-2009. correct option is B While banks almost always
234. Modals like ‘will/shall/may/might etc’ meet the overall target, keeping up with the
are always followed by a verb in its basic subtargets was getting difficult for banks with
form. Therefore, ‘jeopardize’ must be written. limited expertise in certain sectors
The correct option is B This uncertainty and 238. No correction required. The correct
the risk of a severe health disaster will option is E The GST reform was hailed as a
jeopardize the traditional demand great experiment in “cooperative federalism”
management stimulus interventions, and in with all the states and UTs with legislatures
our considered view, the pandemic triggered joining the Centre in the reform to harmonise
economic crisis will be in all possibility be ‘U’ domestic trades.
shaped. 239. The subject here is active and does the
235. ‘Has’ when used for perfect sentences is action by itself and not on itself. Therefore,
followed by 3 rd form of Verb and when used ‘confronting’ should be used. The correct
for perfect continuous, it is followed by ‘been option is B The agency is understood to be
+ verb (ing)’. ‘Has + Verb (ing)’ is wrong. The confronting the official with the digital
correct option is B What’s happening isn’t evidence it had extracted from the gadgets it
hard to decipher: the supply shock has seized from the accused in the case.
dragged demand down with it, through loss of
Use Code: (AVP10) for 10% India’s No.1 Teacher in Bank Exams for English Language and Editorial 79
additional discount on All Website: www.vishalparihar.com | Follow on Instagram: @vishalthetrainer
Unacademy courses
1000+ Error Detection for All Exams
Last 20 years (Previous Year Papers)
English By Vishal Parihar

240. ‘Though’ should be used here instead of 2.6% increase for wheat is only half the cost
‘through’ because here we require a escalation
connector to show two opposite sentences. 245. ‘Feed’ is used here for a continuous verb.
The correct option is C The action against Therefore, it must be written as ‘feeding’. The
non-compliance, at present, lies mainly in the correct option is A The pressure of feeding
imposition of fines, though there are paltry, India’s 1.3 billion people has led to
on the errant companies. suboptimal choices, like a water-guzzling rice
241. ‘Impact’ is here used for a noun, crop grown in abundance in Punjab that
therefore, ‘impact’ is correct and not bleeds aquifers dry. 20. ‘Subsided’ which is
‘impacted’ because this makes it a verb. The used in part (B) of the sentence uses the
correct option is C While private initiative adverb ‘around’. Therefore, ‘together’ is
should not be de-legitimised, it has had only a wrong here. The correct option is A The
limited impact on building capabilities in criticism around Gillian Flynn’s portrayal of
India as it has focused on those with the women has barely subsided and now she
ability to pay. must batten down the hatches for another
242. ‘Among’ is used for more than two onslaught.
people or things. Here, we are mentioning the 246. Modals like ‘will/shall/may/might etc’
incongruence between only two. Therefore, are always followed by a verb in its basic
‘between’ fits here. The correct option is A form. Therefore, ‘play’ must be written. The
Pointing to the incongruence between India’s correct option is C Even though Cusack’s
low level of human development and its character — Dr Kevin Christie — is an
status as a democracy evokes the response American, I’m worried that Utopia movie
that this is to see the latter in instrumental might play into the Trumpsters’ narrative
terms. that the Covid-19 virus was manufactured in
243. ‘India’ is a singular non living thing. a Chinese laboratory.
Therefore, the possessive adjective to it must 247. ‘Has’ when used for perfect continuous
be ‘its’ and not ‘their’. The correct option is D sentences is followed by ‘been + Verb (ing)’
In India, the state’s ritualistic attachment with ‘Been’ is missed here. The correct option is A
the procedures of democracy has not been Trump has been campaigning against large-
matched by an awareness of its implicit goal scale use of mail-in ballots for the upcoming
of a fulfilling life for Indians. elections, alleging, without any evidence, that
244. The sentence states a universal fact. And it will be used for election fraud by
for a universal fact, we don’t use past form or Democrats.
future form. Therefore, ‘is imagined’ is correct 248. The sentence is a universal fact which
here. The correct option is A Dystopia was cannot be written in past or future forms.
imagined as a place where the people Therefore, ‘is’ must be used in D. The correct
experience great suffering as they fend for option is D There are two kinds of mailing
themselves under the watchful eyes of an ballot systems: universal mail-in voting, when
authoritarian state. 18. Prices are announced states send ballots to all eligible voters; and
‘for’ and not ‘against’. The correct option is B absentee balloting, when a voter is sent a mail
Modi has assured farmers the support will on request.
stay by announcing prices for the winter crop, 249. The subject of this sentence is
though as farmer activists have noted, the ‘foreigners’ which is a plural. Therefore,
‘have’ must be used here. The correct option
Use Code: (AVP10) for 10% India’s No.1 Teacher in Bank Exams for English Language and Editorial 80
additional discount on All Website: www.vishalparihar.com | Follow on Instagram: @vishalthetrainer
Unacademy courses
1000+ Error Detection for All Exams
Last 20 years (Previous Year Papers)
English By Vishal Parihar

is C The absconding foreigners make up 90% Erudite scholars and alumni from our
of foreign nationals who were enlarged on University are being acknowledged amongst
bail pending trial and have fled the country or the thought leaders of the country and they
otherwise made themselves scarce and not have carved a niche for themselves in
available to face prosecution. different walks of life.
250. The sentence shows a complaint lodged 257. No correction required. The correct
in Lok Adalat. Therefore, here we must have option is E On the 75th anniversary of the
‘in Lok Adalat’ and not ‘from’. The correct UN’s foundation, not only is great power
option is B Six months after a man in Odisha’s rivalry rising to Cold War levels, but the world
Kandhamal district lodged a complaint in Lok is under enormous forces of change, whether
Adalat after failing to get his Aadhaar card technological or environmental.
after more than 20 attempts, the hearing in 258. ‘Did’ being already a past helping verb
the case is yet to begin. doesn’t use the Verb in past forms. Therefore,
251. This sentence shows a present situation Verb’s first form must be used after it. The
of rise of cases. Therefore, ‘rise’ must be used correct option is B In the aftermath of the
here and not ‘rose’. The correct option is A “letter bomb” in the Congress that didn’t quite
With virus cases on the rise and private explode, there has been much angst over the
diagnostic centres taking at least 24 hours to apparent lack of inner party democracy in the
provide available slots, many people are now party.
lining up outside mohalla clinics to get 259. No correction required. The correct
themselves tested. option is E The dreary evening of February 23
252. No correction required. The correct saw parts of NorthEast Delhi gripped by a
option is E In a circular that factors in the communal frenzy, reminiscent of the carnage
uncertainties surrounding COVID-19, the UGC during the days of Partition.
has offered institutions the option of offline, 260. The subject of the sentence is ‘territories’
online or blended modes of instruction. which is a plural while at one place in (B) we
253. ‘Numbers’ is a plural noun and therefore, have used ‘is’ to represent the same, which is
must be followed by a plural helping verb. wrong. The correct option is B The entire
Therefore, ‘are’ must be used here. The territories of the UTs of Jammu and Kashmir
correct option is D India’s move to unlock and Ladakh have been and are an integral
education coincides with a reported part of India and would remain so and
reduction in overall daily incidence of Pakistan has no locus standi to comment on
infection, although absolute numbers are India’s internal matters.
frighteningly high. 261. ‘To’ when followed by a Verb makes an
254. The sentence focuses on effects of ‘infinitive verb’ and here, it is always followed
pandemic on health and collapse of health by Verb in its 1st form only. The correct
systems. Therefore, ‘to’ after collapse is option is D The Supreme Court on Thursday
wrong here. The correct option is B Globally, issued a notice to the Centre and the Union
unpreparedness to handle the pandemic and Public Service Commission (UPSC) on a plea
near collapse of the health systems filed by UPSC aspirants urging the court to
denudedour ability to focus on health. postpone the upcoming civil services exam.
256. ‘They’ represent a plural number of 262. The word ‘from’ is wrong here because
living things. It must be followed by a plural here, we are showing to give a loan and to
helping verb only. The correct option is D show how much of the amount of the loan, we
Use Code: (AVP10) for 10% India’s No.1 Teacher in Bank Exams for English Language and Editorial 81
additional discount on All Website: www.vishalparihar.com | Follow on Instagram: @vishalthetrainer
Unacademy courses
1000+ Error Detection for All Exams
Last 20 years (Previous Year Papers)
English By Vishal Parihar

need to use ‘of’ and not ‘from’. The correct 267. No correction required. The correct
option is C Chief Minister Shivraj Singh option is E That Covid-19 has disrupted
Chouhan said that a scheme to provide an education in an unprecedented way is
interest-free loan of Rs 10,000 to rural street illustrated by the fact that schools reopened
vendors of Madhya Pradesh has been after a gap of six months on Monday to a
implemented. lukewarm response.
263. ‘Across’ is a wrong preposition here. The 268. ‘Being’ is used here to show a simple past
sentence shows monitoring of something by a sentences and not a continuous tense.
government. Therefore, the correct Therefore, it must be followed by ‘contracted’
preposition is ‘by’. The correct option is D The and not ‘contract’. The correct option is D The
Islamic scholar Zakir Naik, who has courted schools, on their part, are not taking chances
controversy for his regressive remarks, is and have made mandatory the consent of
being closely monitored by the Narendra guardians, besides putting in place the Covid
Modi-led government. protocol to rule out chances of the infection
264. No correction required. The correct being contracted.
option is E. The decision to withdraw tenders 269. Has’ when used for perfect sentences is
for the supply of ammunition ranging from followed by 3 rd form of Verb and when used
rifles, fired grenades to artillery rounds is for perfect continuous, it is followed by ‘been
sure to dampen spirits in the industry. + verb (ing)’. ‘Has + emphasises’ is wrong. The
265. Here, we are not showing the numbers in correct option is A As the UN has emphasised,
plural. It is being used for an adjective to a getting students safely back to the classroom
noun. And when used like this, we use ‘year’ must be a top priority as we confront a
and not ‘years’. The correct option is B After generational catastrophe that could waste
90 years of safekeeping, a Pakistani Sufi untold human potential.
organisation has transferred 110-year old 270. ‘Decisions’ which is the subject of the
copies of rare manuscripts of the Guru Granth sentence is a plural noun and therefore, the
Sahib to a gurdwara in Sialkot, the media helping verb related to it must be a plural
reported on Thursday. 40. Modals like verb or plural helping verb. Therefore,
‘will/shall/may/might etc’ are always ‘require’ is correct here. The correct option is
followed by a verb in its basic form. B Decisions that impact the lives of billions
Therefore, ‘come’ must be written. The require scrutiny and fixing of accountability,
correct option is D For the beleaguered yet dealing with a once-in-a-century
hospitality industry, the brisk business pandemic is a tough call.
reported by hotels at the tourist hotspot of 271. No correction required. The correct
Kasauli will come as a relief. option is E While most question the severity
266. ‘Capital’ is a singular noun and therefore, of the shutdown, some fail to comprehend the
must be followed by a singular helping verb urgency to unlock trade and business activity.
only. Therefore, ‘has’ must be used here. The 272. To’ when followed by a Verb makes an
correct option is B Wherever agriculture is ‘infinitive verb’ and here, it is always followed
being opened to markets, the big capital has by Verb in its 1st form only. The correct
successfully managed to push out the option is C The only sector in the Indian
majority farming population and economy that survived the Covid contraction
concentrated its control over food. is now being forced to suffer needless anxiety,

Use Code: (AVP10) for 10% India’s No.1 Teacher in Bank Exams for English Language and Editorial 82
additional discount on All Website: www.vishalparihar.com | Follow on Instagram: @vishalthetrainer
Unacademy courses
1000+ Error Detection for All Exams
Last 20 years (Previous Year Papers)
English By Vishal Parihar

with the government refusing to address Q.1 – Sentence applies. „To‟ must be followed
farmers’ legitimate apprehensions. by 1st form of verb. Therefore, here we will
273. Here, we need to show fatality rate of write “express” in the place of “expressed”.
state. And to represent the same, we need to For sentence B, the word „relevant‟
use apostrophe + s (‘s) after ‘state’ to show (adjective) should be replaced with
possession. The correct option is C The Covid- “relevance” (noun) as per the requirement of
19 pandemic has worryingly been relegated the sentence. For sentence C, we will replace
to the sidelines in Punjab, even though the „attacked‟ with “attack” as before this word,
state’s fatality rate continues to be among the we have „an‟ which requires a noun. But
highest in the country. „attacked‟ is a verb. • Within the bounds of
274. ‘From of’ makes no sense and is never law, liberal democracies ensure that citizens
used together. To show the exigencies arising, enjoy the right to express themselves in every
we need to use ‘out of’ here. The correct conceivable manner, including the right to
option is D IN a show of austerity, Parliament protest, and express dissent against
has cut the salaries and allowances of prevailing laws. • What is of utmost relevance
ministers and MPs to meet the exigencies today is our ability and commitment to
arising out of Covid-19 preserve, conserve and build on the rich
275. The subject of the sentence is ‘truth and pluralistic history that we have inherited. •
accuracy’ which is a plural noun. Therefore, it Dissenting voices cannot be labeled
must be used in plural forms only in the „antinational‟ or „anti-democratic‟ and such
further parts of the sentence. Therefore, assertions are an attack on people‟s
‘Casualties’ is correct here. The correct option commitment to protecting constitutional
is B Truth and accuracy are the prime values.
casualties when TV anchors and reporters 278. The correct option is B - I & III. The
stoop to new lows just to grab eyeballs. sentence shows a simple statement of past
276. The correct option is B – I & II. „Force‟ tense and sentences of simple past uses 2nd
will be replaced by „Forced‟ in sentence A as form of verb. But here, we have “gone” which
it seems to be of passive voice which implies is a 3rd form of “go”. Therefore, we will
that an action is done on something. And for replace it with “went”. “Of” when followed by
writing passive voice, we use 3rd form of verb a verb, takes the verb in its –ing form.
after “was/were”. „To‟ when followed by a Therefore, “throw” will be replaced by
verb always takes the verb in its 1st form. “throwing”. • The video went viral on social
This “to + Verb1” shows a secondary verb in a media platforms on Friday, but caught
sentence which does not show an active police‟s attention when right-wing activists
action. There it is never written in any past approached the college seeking action against
forms. • An Air India aircraft was forced to get the students on • Quite often, disasters are
airborne earlier than planned at Pune airport proof of our own failings, man-made
to avoid colliding with an IAF jeep that was transgressions of order, of throwing caution
dangerously near the airstrip. • Credit flow is to the winds.
reviving slowly and steadily and is set to 279. The correct option is B – Only I. In
improve on the back of steps taken by the sentence A, “issue” should be replaced with
government and RBI. “issued”. Refer to same reason in Q1. – A. •
277. The correct option is C – None is correct. The government ordered an inquiry, but
For sentence A, same reason as written for
Use Code: (AVP10) for 10% India’s No.1 Teacher in Bank Exams for English Language and Editorial 83
additional discount on All Website: www.vishalparihar.com | Follow on Instagram: @vishalthetrainer
Unacademy courses
1000+ Error Detection for All Exams
Last 20 years (Previous Year Papers)
English By Vishal Parihar

members of the opposition wanted to know 284. The correct option is B – I & II In
why the circus company was issued a license sentence A, the first word „Embattle‟ is wrong
to operate in such an unsafe place. as here, it is used as an adjective and when a
280. The correct option is B – I & II In verb is used as an adjective, it is written in its
sentence A, “wing” should be replaced with past forms. Therefore, „Embattled‟ must be
“winged” as here it is used as participle form put there. In sentence B, we have „has‟ which
of verb to show a verb (in 2nd form) used as shows a perfect tense, and „has‟ is always
an adjective. “What” in sentence B should be followed by the 3rd form of verb. Therefore,
replaced with “whom” as here the position „make‟ must be replaced with „made‟. •
adjective is to be used for “devotees” which Embattled telecom provider Vodafone Idea is
are living beings and for them, we use ready to pay the government dues related to
“whom”. • Though the place is devoid of large the Supreme Court‟s AGR judgment, but said
winged scavengers, the shrub forest hosts that its ability to remain a going concern
many tiny birds whose chirping rent the air, hinges on a leniency in the payment schedule.
especially during mornings and evenings. • • While Vodafone mulls the amount that it will
Every Saturday, the temple is thronged by be paying, Sunil Mittal‟s Airtel has already
villagers and other devotees – many of whom made clear its intentions to pay, as well as the
cover long distances – to pray to the deity. schedule.
281. The correct option is A – II & III The 285. The correct option is A – None is correct
sentence B shows evolution of paper notes In sentence I, „time‟ must be replaced with
before British rule, which needs a verb „times‟ as it is preceded by „five‟ which is a
“evolved” to show evolution in the past. plural number. And, the noun after a plural
“Losted” is no word. “Lost” is the correct noun is always written in its plural form. In
word. sentence II, „Align‟ is always followed by
282. The correct option is B – I & II In „with‟ to form a phrasal verb „align with‟
sentence A, “Has” when followed by a verb which means to do proceedings matching to
always takes it in 3rd form. Therefore, something else. The sentence III shows
“began” here must be replaced with “begun”. transfers of surplus by RBI. „Transfers‟ must
In sentence B, „technology‟ is followed by two indeed have the preposition „to‟ to show the
names. This implies that there isn‟t a single receiving organization. • RBI has reduced
technology mentioned. Therefore, policy rates five times since February 2018
“technology” should be replaced with and there are indications that banks are
“technologies”. • Every western country has passing on the cuts now. • The RBI has
begun appreciating yogic sciences and is proposed a change in its JulyJune accounting
taking it as a holistic approach towards health year to align with the government‟s financial
after consulting right doctors. • Our doctors year of April-March. • The RBI would be able
need to equip themselves with latest to provide better estimates of the projected
technologies like Artificial Intelligence and surplus transfers to the government for the
machine learning to match international financial year of budgeting purposes.
standards. 286. The correct answer is E - No correction
283. The correct option is E - No correction required.
required. 287. The correct option is B – Only II
Quantifier „some‟ is always followed by a

Use Code: (AVP10) for 10% India’s No.1 Teacher in Bank Exams for English Language and Editorial 84
additional discount on All Website: www.vishalparihar.com | Follow on Instagram: @vishalthetrainer
Unacademy courses
1000+ Error Detection for All Exams
Last 20 years (Previous Year Papers)
English By Vishal Parihar

plural noun as it is used to show plural with a plural helping verb. There „have‟ must
number (persons or things). Therefore, in be used there instead of „has‟. „Filtering out‟
sentence B, „fisherman‟ must be replaced is a phrasal verb used in sentence III. To
with „fishermen‟. highlight a verb, we use an adverb. The word
288. The correct option is D – Only I „Allege‟ „disproportionate‟ is wrong here as here, it is
in sentence II needs to be replaced with used as an adverb and the adverb form of it is
„Alleged‟ as here it is used as an adjective to „disproportionately‟. • Walnuts contain
extortion (noun) and when a verb is used as omega-3 fatty acids and polyphenols, which
an adjective, it is written in its past form. • have been found to counteract oxidative
The agency recovered emails of Iqbal Memon stress and inflammation, both of which are
and analysed allege extortion and drug drivers of cognitive decline. • While
proceeds laundered from India to Dubai algorithms may speed up the process of
where he ran a few hotels to cover up the narrowing the pool of job candidates, they are
dirty money. often not great at finding the most qualified
289. The correct option is C – None is correct. ones, and instead, end up disproportionately
„Build‟ is the first form of a verb which is used filtering out people in those categories.
to show incidents of past. Sentence I shows 291. The correct option is D – None is correct.
building of a dam in . The sentence I seem to be an action of simple
Therefore, to show past, we will use „Built‟ past. Simple past is used to write sentences
instead of „Build‟. „Communities‟ being a where we remember or tell a past action in a
plural noun must be followed by a plural general way. And for writing simple past, we
helping verb. Therefore, we will use „are‟ in use 2 nd form of verb. Therefore, „becomes‟
place of „is‟ in the sentence II. The sentence must be replaced with „became‟. The
III shows a research and a research is an sentence II is a sentence of passive voice
obtained result of something. Therefore, to where a sea change is observed. For a singular
show the result, we use „found‟ and not „find‟. subject like „sea change‟ we use a singular
• Built in 1920 to tame the flood fury of River helping verb. Therefore, „has‟ must be
Musi, Osman Sagar has served the drinking written there. The subject of the sentence III
water needs of a million people in parts of is a singular noun, and hence must be
Hyderbad for 100 years. • During his talk on followed by a singular helping verb.
„Hues that make India‟ Justice Chandrachud Therefore, „were‟ must be replaced with
called on audience to include communities „was‟ to make it grammatically correct. •
that are pushed to the fringe of the Though the POSCO act became law in 2012,
mainstream for being minorities – based on changes in infrastructure and judicial staff
their language, faith, culture or gender. • A training took a couple of years and the results
research found that Walnut consumption by can be seen now with testimonies of minors
healthy, elderly adults had little effect on that have ended in convictions. • There has
cognitive function over two years, but it had been a sea change in how subtitles are
greater effect on elderly adults who had lower written, because of streaming platforms and
baseline neuropsychological test scores. new markets. • The growth momentum of the
290. The correct option is A – I & III In first seven years was remarkable, but
sentence I, the subject is „Walnuts‟ which is a economies have ups and downs and there
plural noun. Therefore, it must be followed was a downturn in the last three years.

Use Code: (AVP10) for 10% India’s No.1 Teacher in Bank Exams for English Language and Editorial 85
additional discount on All Website: www.vishalparihar.com | Follow on Instagram: @vishalthetrainer
Unacademy courses
1000+ Error Detection for All Exams
Last 20 years (Previous Year Papers)
English By Vishal Parihar

292. The correct option is A – I & II The 295. The correct option is A – Only III The
subject in statement I is „policy‟ which is a sentence III contains a phrasal verb „gave
singular noun. It must be followed with a into‟ which means „overlook. But the
singular helping verb. Therefore, „has‟ must sentence intends to show „submission‟ of
be used here. In sentence II, „Auction‟ must be National Democratic Front. Therefore, it must
replaced with „auctioning‟. • The decision to be replaced with „gave up‟ which means
raise import duties is a major reversal of the „submission to someone/something‟. • The
policy of gradually reducing import duties, National Democratic Front of Bodoland gave
which has been in place for 30 years and was up arms marking an end to a 34-year armed
followed by several governments. • The CAG struggle for a separate Bodoland state.
came up with unrealistically large estimates 300. The correct option is C – I & II The
of revenue losses and public opinion came to sentence I shows refusal of the Supreme
the view that not auctioning spectrum was a Court for a BSP MP. Furthermore, the
big mistake. sentence gives reference to the MP of being
293. The correct option is B – Only II The accused of rape. And for making a reference
subject of sentence II is „He‟ which represents to the MP, the pronoun that should be used is
a male. But in the last part of the sentence, the „who‟ and not „which‟. In sentence II, the
possessive adjective relating to it is „her‟ word „evacuate‟ is wrong here as here we
which is incorrect. Therefore, „her‟ must be need to show a noun i.e. „evacuation‟. • The
replaced with „his‟ to make it grammatically Supreme Court refused to stop BSP MP, who
correct. • He was a well-behaved boy, is accused of rape and is in custody, from
drawing a personality sharply divergent from taking oath as Member of Parliament eight
the one his Facebook friends saw: of months after he got elected as a candidate of
undisguised hatred towards one community SP-MSP coalition. • While Beijing was yet to
and his violent intentions. approve India‟s request to operate two
294. The correct option is A – I & II As stated aircraft to fly back Indian nationals, the Indian
in many questions above, a verb when used as side was hoping evacuation will finally begin
an adjective acts as a past participle and here, January 31st evening.
the verb is written in its past form. Therefore, 301. The correct option is A – II & III
„ripe‟ must be replaced with „ripen‟ to make „Balanced‟ in sentence II, is preceded by an
it grammatically correct. The word „execute‟ article „a‟ and an article is always followed by
is preceded by their, which means that indeed a noun. Therefore, the word „balanced‟ must
there need to be a noun. The word „execute‟ be corrected with „balance‟ to make it
is a verb which is incorrect here, therefore it grammatically correct. The sentence III is in
must be replaced with „execution‟. • simple present tense, but the verb „awaited‟
Deploring the use of chemicals to ripen fruits, is incorrect here as it indicates a past time.
Delhi high court equated it to poisoning the Therefore, it must be replaced with „awaits‟.
consumer and called for a launch of criminal • Balance between the right to protect, and
prosecution against such culprits. • A Delhi ensuring that they don‟t cross the line to
court issued notice to Tihar Jail authorities illegality is a balance difficult to maintain. •
asking them to respond to a plea of the death Urns containing the ashes of deceased
row convicts seeking stay on their execution members of Hindu families in Pakistan are
slated for February 1. piling up in temples and crematoriums there

Use Code: (AVP10) for 10% India’s No.1 Teacher in Bank Exams for English Language and Editorial 86
additional discount on All Website: www.vishalparihar.com | Follow on Instagram: @vishalthetrainer
Unacademy courses
1000+ Error Detection for All Exams
Last 20 years (Previous Year Papers)
English By Vishal Parihar

as the community awaits restoration of train Maharashtra CM Thackrey said there was no
and bus links with India to perform the final need to fear the CAA, but asserted his
ritual. government will not allow the proposed NRC
302. The correct option is D – Only I „Vehicle‟ to be implemented as it would “impact people
is a singular noun and thus must have a of all religions”. • Cabinet‟s decision to
singular helping verb. Therefore, „don‟t‟ in approve MTP (Amendment) Bill, 2020, will
sentence I is incorrect and must be replaced truly address the needs of gender justice
with „doesn’t‟. • Vehicle owners will now through the prism of reproductive rights,
receive messages from the transport providing a solution which women in our
department if their vehicle doesn’t have third country have sought for decades.
party insurance or the policy has expired. 305. The correct option is A – Only II „To‟
303. The correct option is C – None is correct when followed by a verb is always written in
The subject of sentence I is „candidate‟ which its basic form (1st form). Therefore, the word
is a singular noun. Therefore, it must be „used‟ must be replaced with „use‟. • In the
followed by a singular helping verb. Indian context, anticipatory bail was found
Therefore, „get‟ must be replaced with „gets‟. necessary because of incessant targeting of
The subject of sentence II is „illegals‟ which political rivals and the tendency of police to
being a plural noun must be followed by a use its power of arrest even in cases which
plural noun. Therefore, „lies‟ must be require no custodial interrogation.
replaced with „lie‟. In sentence III, the word 306. The correct option is A – I & III The word
„convict‟ must be replaced with „convicts‟ as „revolution‟ in Sentence I is a noun and
it is preceded by „four‟ which needs a plural „learn‟ is a verb. Here, the verb is acting as a
noun after it and not a singular noun. • Before gerund (Verb + ing). Therefore, it must be
a presidential candidate gets their name on replaced with „learning‟. „Economic
the ballot for election day, they must survive attraction‟ being a singular noun must be
a nomination process, which whittle down replaced with a singular helping verb.
the firld of candidates to one for each party. • Therefore, „have‟ must be replaced with
Sceptics argue that every census undercounts „has‟. • Educationists fear that a ban will cut
the illegals since Bangladeshis, fearful of off Indian children from the learning
detention or deportation, lie about their revolution happening in the world, especially
birthplace. • Within hours of Delhi high court digital learning, and disconnect them from job
refusing permission for the execution of the opportunities in the knowledge economy. •
four Nirbhaya case convicts separately, the Demographic and economic trends in the two
Centre challenged it in the Supreme Court. countries suggest that India‟s economic
304. The correct option is B – I & III „Will‟ attraction for Bangladeshi migrants has
being a modal is always followed by the first diminished, especially in Assam which is
form of verb because modals do not highlight among the poorest and least economically
an action verb. Therefore, „allowed‟ in dynamic of Indian states.
sentence I must be replaced with „allow‟. The 307. The correct option is D – Only I In
subject in the last clause of sentence III is sentence I, „to‟ must be replaced with „for‟. •
„women‟ i.e. a plural noun and it must be Illustrating the heavy traffic congestion on
followed by a plural helping verb. Therefore, Indian roads, a report released recently has
„has‟ in it must be replaced with „have‟. • ranked three Indian cities in the top five

Use Code: (AVP10) for 10% India’s No.1 Teacher in Bank Exams for English Language and Editorial 87
additional discount on All Website: www.vishalparihar.com | Follow on Instagram: @vishalthetrainer
Unacademy courses
1000+ Error Detection for All Exams
Last 20 years (Previous Year Papers)
English By Vishal Parihar

among 416 cities across 57 countries, for functional terms, as the ability to learn new
having the worst traffic gridlocks. skills and solve various types of problems.
308. The correct option is B – Only II In 313. The correct option is B – Only I „Can + be‟
sentence II, „should‟ being a modal, must be is always followed by the 3rd form of verb.
followed by a basic form (1st form) of verb. Therefore, „replicate‟ must be replaced with
Therefore, „refrained‟ must be replaced with „replicated‟. • Once a machine achieves
„refrain‟. • Russian priests should refrain superhuman competence in any given
from the practice of blessing nuclear weapons domain, that competence can be rapidly
and other weapons of mass destruction that replicated multiple times.
can inflict indiscriminate loss of life. 314. The correct option is B – II & III„Friend‟
309. The correct option is D – Only II In in sentence II is incorrect as here we need an
sentence II, „to‟ is followed by „ensured‟ adverb while „friend‟ is a noun. Therefore it
which is incorrect. „To‟ when followed by a must be replaced with „friendly‟. The
verb must have verb in its basic form. preposition „in‟ in the end of sentence III
Therefore, it must be replaced with „ensure‟. must be replaced with „for‟ to make it
• Atalji‟s NDA government made serious grammatically correct. • Don‟t be friendly
efforts to ensure safer abortions, by with people who are complaining and
eliminating abortion by untrained persons negative in their mindset because if you are
and in unhygienic conditions, thus reducing friendly with them, you also get into that
maternal morbidity under the aegis of the mode. • Doesn‟t matter if it‟s a fast food joint
MTP. or an upscale diner, a cup of coffee or a round
310. The correct option is E - No correction of beer, indulging in eating and drinking is a
required. favourite pastime for many.
311. The correct option is B – Only II 315. The correct option is C – I & III „Called
„Refugee‟ is an inappropriate word here as off‟ in sentence I is incorrect as it shows
here we need a verb and „refugee‟ is a noun. finishing or cancelling something and must be
Therefore, it must be replaced with „refuge‟. replaced with „called on‟ to show the call or
• The bill seeks to strengthen provisions for step taken by India. The subject of the
protecting the dignity and privacy of women sentence III is „rights‟ which needs a plural
who seek the refuge of law when confronted helping verb after it. Therefore, „is‟ must be
with such a life altering decision. replaced with „are‟. • India called on Sri
312. The correct option is B – II & III „Stepped Lanka to fulfill the aspirations of the island
down‟ is an incorrect word in sentence II. nation‟s Tamil minority by taking forward
Therefore, it must be replaced with „stepped the process of reconciliation and
up‟. „Can‟ being a modal must be followed by implementing a constitutional provision
a verb in its basic form (1st form). Therefore, aimed at the devolution of powers. • It would
„been‟ must be replaced with „be‟. • be useful to remind oneself that the rights
Recognising the burgeoning numbers of which citizens cherish deeply are
undertrial prisoners in jails, initiatives to free fundamental – it is not the restrictions that
those not accused of heinous offences on are fundamental.
personal bail need to be stepped up. • 316. The correct option is D – Only II In
Intelligence can perhaps be described in sentence II, „over‟ is an incorrect preposition
and must be replaced with „across‟. • The

Use Code: (AVP10) for 10% India’s No.1 Teacher in Bank Exams for English Language and Editorial 88
additional discount on All Website: www.vishalparihar.com | Follow on Instagram: @vishalthetrainer
Unacademy courses
1000+ Error Detection for All Exams
Last 20 years (Previous Year Papers)
English By Vishal Parihar

Supreme Court directed that Cinema halls 322. The correct option is D – I & II
across the country must play the National „Maliciously‟ in sentence I must be replaced
Anthem before the screening of a film and with „malicious‟ as here we require an
people should stand up as a mark of respect. adjective while „maliciously‟ is an adverb.
317. The correct option is E - No correction „Touched on‟ is a phrasal verb which means
required. affecting something. But the sentence needs
318. The correct option is B – I & II The to show „closure/finish‟ of something.
subject of sentence I is „bravado‟ which is an Therefore, it must be replaced with „touched
uncountable noun. And an uncountable noun off‟. • The great American tragedy happened
is always followed by a singular helping verb. when US bombs, rained on Baghdad, glowing
Therefore, „camouflage‟ must be replaced like malicious fireflies on TV screens, CNN
with „camouflages‟. In sentence II, „should‟ bringing humanity‟s first televised war,
being a modal must always be followed by a palpable excitement ruffling its
verb in its basic form (1st form). Therefore correspondents‟ immaculate scarves. • The
„became‟ must be replaced with „become‟. • Supreme Court order for playing national
The bravado of instruction to BJP leaders that anthem in cinema halls has touched off an old
they should file expenditure statements for debate on whether forcing someone to sing
the period Nov 8 to Dec 31, 2016, camouflages the anthem infringes fundamental rights.
the question of whether there was selective 323. The correct option is A – II & III „Make‟
advance leakage of information of in sentence II must be replaced with
demonetisation. • Promises made during „making‟. „Woman‟ in sentence I must be
elections are to be backed up by performance, replaced with „women‟. • Assurance that
and results of good governance should rights are secure tends to diminish fear and
gradually become visible to the public. jealousy of strong government, and by
319. The correct option is D – Only II In making us feel safe to live under it makes for
sentence II, „uphold‟ must be replaced with its better support. • A latest study found
„upheld‟ as it is incorrect. • It‟s the Supreme anxiety about appearing “rude” or
Court which has often protected and upheld “ungrateful” was stopping women from
the rights and liberties of the individual and requesting more money – with around a
the minority against attempts by the state to quarter of women saying they were fearful
encroach on them, often in the name of the that contesting pay could endanger benefits
majority‟s mandate. like maternity leave or flexible working.
320. The correct option is B – Only II 324. The correct option is B – None is correct
„Effectively‟ in sentence II is incorrect as it is „Aggression‟ in sentence I must be replaced
preceded by „more‟ which requires a verb. with „aggressive‟ because here we need an
Therefore, it must be replaced with adjective and not a noun. „To‟ when followed
„effective‟. • If we can work out where healthy by a verb always uses the verb in its basic
cells normally live and what makes them form (1st form). Therefore, in sentence II
expand when someone stops smoking, „exiting‟ must be replaced with „exit‟. In
perhaps we have opportunities to make them sentence III, „waves‟ is incorrect as it is
even more effective at repair. preceded by an article „a‟ which always
321. The correct option is E - No correction requires a singular noun. Therefore, it must
required. be replaced with „wave‟. • The aggressive

Use Code: (AVP10) for 10% India’s No.1 Teacher in Bank Exams for English Language and Editorial 89
additional discount on All Website: www.vishalparihar.com | Follow on Instagram: @vishalthetrainer
Unacademy courses
1000+ Error Detection for All Exams
Last 20 years (Previous Year Papers)
English By Vishal Parihar

expansion of Chinese smartphone-makers in discovery of the longest smuggling tunnel


India has helped the country dethrone the US ever found on the southwest border,
to become the second largest smartphone stretching more than three quarters of a mile
market in the world. • The Centre offered to from now to industrial site in Tijuana, Mexico,
exit Air India and Air India Express by selling to the San Diego area.
its entire stake, instead of the 76% it had 328. The correct option is D – II & III In
offered two years ago, apart from the 50% it statement II, the subject is a singular noun
owns in ground handling joint venture AI- „Lipton‟ and hence, must be followed by a
SATS. • The coronavirus outbreak has stoked singular helping verb, Therefore, „focus‟ must
a wave of anti-China sentiment around the be replaced with „focuses‟. In sentences III,
globe, from shops barring entry to Chinese we find that „be‟ is written after „has‟. The
tourists, online vitriol mocking the country‟s sentences of perfect tense are written with
exotic mean trade and surprise health checks „has/have/had‟, followed by a 3rd form of
on foreign workers. verb. Therefore, „be‟ is wrong here and must
325. The correct option is E - No correction be replaced with „been‟. • The iconic “Taj
required. Mahal” tea under Brooke Bond, which is
326. The correct option is A – II „Quick‟ is marketed as a brand closely linked to the
incorrect in sentence II as we need an adverb heritage culture, while Lipton focuses on
but „quick‟ is an adjective. Therefore, it must emerging consumer tastes such as green tea.
be replaced with „quickly‟. • It is in terms of • The finance ministry has summoned senior
modest lethality and human ability to react Infosys executives to find out why the
quickly; some of the dynamic with regards to technology major has been unable to fix
spread of coronavirus may be different with glitches in the GST Network system, which is
the increasing migration and mobility of causing hardship to lakhs of taxpayers.
people and products, which explains the 329. The correct option is B – None is correct
massive quarantines underway in China. In sentence I, the subject is „businesses‟
327. The correct option is C – None is correct which is a plural noun. And, it must be
„Instant‟ is incorrect in sentence I and must followed by a plural helping verb. Therefore,
be replaced with „instance‟. „Both‟ is always „has‟ must be replaced with „have‟ to make it
paired with „and‟. Therefore, „or‟ in sentence grammatically correct. In the second clause of
II must be replaced with „and‟. „Founded‟ is sentence II, the subject is „analysts‟, which is
no word. Therefore, it must be replaced with a plural noun. Therefore, as states above for
„found‟ in sentence III. • While Bajaj had sentence I, it must also be followed by a plural
already taken a back seat when it came to helping verb. Therefore, „does‟ must be
taking active business decisions years ago, replaced with „do‟. „Has‟ is never followed
this would be the first instance when he just by a verb in its –ing form. It becomes a
officially gets into a nonexecutive position. • sentence of Present Perfect Continuous tense,
Both Bengal and Delhi fancied their chances which has the following combination –
of collecting three points, but in the end it was “Has/have + been + Verb (1st form) + -ing”.
the weather which had the final say as the Also, a sentence of perfect continuous tense
teams settled for a point each from their needs a time or duration because this tense
drawn Ranji Trophy Elite „A‟ match at the defines how long has an action taken place.
Eden Gardens. • US authorities announced the And it is advised to use „perfect tense‟ when
the sentences of present perfect continuous
Use Code: (AVP10) for 10% India’s No.1 Teacher in Bank Exams for English Language and Editorial 90
additional discount on All Website: www.vishalparihar.com | Follow on Instagram: @vishalthetrainer
Unacademy courses
1000+ Error Detection for All Exams
Last 20 years (Previous Year Papers)
English By Vishal Parihar

tense which do not mention any time frame or plural form, i.e. ―convicts‖ in place of
duration. But, in this sentence, we don‟t have ―convict‖. Therefore, the correction is in (A)
any time frame or duration. Therefore, here part of the sentence. The correct option is B.
sentence III, „has forcing‟ must be replaced One of the four death row convicts in the
with „has been forced‟ to make it Nirbhaya gang rape and murder case, Vinay
grammatically correct. • While businesses Sharma, approached the Supreme Court on
have been complaining of problems with the Tuesday challenging the rejection of his
platforms for several months, it is only now mercy petition by the President.
that the government has begun 332. The sentence tells about help done by
acknowledging the concerns. • While BBC to four generations. Part (D) has written
Unilever has initiated a strategic review of its a plural noun ―live‖ preceded by a possessive
global tea business, Industrial analysts do not adjective ―their‖. Therefore, it must be
expect the Indian subsidiary Hindustan replaced with ―lives‖. The correct option is B
Unilever to evaluate selling its tea business in The BBC did help at least four generations of
India. • The Narendra Modi government Indians to meet their goals, improve their
battling an acute economic slowdown has lives, and find their place in the world.
been forced to announce a series of steps to 333. The subject of the sentence is ―theft‖
accelerate activity. which is a singular noun, which must be
―Trends‖ is followed by a singular helping followed by a singular verb (or singular
verb ―has‖, which is grammatically incorrect. helping verb). But, the verb ―affect‖ is not
There may be two corrections to it. Either it written after singular nouns. It must be
will be ―trends have‖ or ―trend has‖. replaced in (B) to ―affects‖ to make the
Therefore, the error is in part (A) of the sentence grammatically correct. The correct
sentence. The correct option is B. The initial option is B Online copyright theft is a criminal
trends have shown that AAP is set to repeat enterprise which also affects consumers, and
for the third time with Chief Minister Arvind is susceptible to malware, identity theft and
Kejriwal heading the government. ransomware.
334. The sentence tells an incident when a
330. ―To‖ when followed by a verb uses the warning is done for an impulsive action to
verb as an infinitive form of verb to show a something. ―Warn‖ in sentence (A) must be
secondary verb/action in a sentence. The replaced with ―warned‖ to make the
verb that is written after ―to‖ is always sentence grammatically correct. The correct
followed by the 1st form of verb. Therefore, option is A Former IAF chief B S Dhanoa
the correction is in (B) part of the sentence. warned that the Indian government could hit
The correct option is C. The World Health cross-border terrorists harder if they
Organization convened outside experts to launched a strike similar to the one in
fast-track promising tests, drugs and vaccines Pulwama on February 14, 2019.
to help slow the outbreak of a new virus that 335. The sentence tells an incident of past in
emerged in China that has killed more than a general way. This takes the form of Simple
1,000 people and spread to two dozen other Present Tense to write the verb. The word
countries. ―took‖ is used for simple past only.
331. “Numeral + of‖ when followed by any Therefore, it should be changed to its passive
noun, always takes a plural noun after it. Here form i.e. ―taken‖ to be written in Simple
also, after ―one of‖ the noun should be in Present Tense. The correct option is B
Use Code: (AVP10) for 10% India’s No.1 Teacher in Bank Exams for English Language and Editorial 91
additional discount on All Website: www.vishalparihar.com | Follow on Instagram: @vishalthetrainer
Unacademy courses
1000+ Error Detection for All Exams
Last 20 years (Previous Year Papers)
English By Vishal Parihar

Millions of people across the globe come Civil Code (UCC) will be introduced on
together to bring about positive changes Tuesday.
online raise awareness and take part in 341. The connector used in (D) must be
events and activities. ―between‖ instead of ―at‖ as the sentence
336. No correction required. The correct highlights things done between two
option is E The fact is that the state of countries. The correct option is C A statement
J&K would not have come into existence had from the White House said that the trip will
it not been for the formidable combination of ‗further strengthen the United States-India
diplomacy and valour shown by the Dogras strategic partnership and highlight the strong
under Maharaja Gulab Singh (1792-1858). and enduring bonds between the American
337. Same as explained in Question 2, and Indian people‘.
―authenticate‖ should be written instead of 342. The helping verb ―Did‖ itself narrates
―authenticates‖. The correct option is B about the sentence to be of past. So, writing
While technology offers several solutions to the main verb also in past form will make it
authenticate the original product, the same superfluous i.e. more than what is required.
technological tools, particularly artificial Therefore, ―portrayed‖ in (B) must be
intelligence, help create clones. replaced with ―portray‖ to make the
338. All modal are used in a sentence to show sentence grammatically correct. The correct
strength, capacity, certainty, possibility or option is C The woman alleged that Chopra
ability of being of an action. It doesn‘t actually commercialized the entire issue of Kashmiri
show any action happening at any time. Pandit exodus of the 1990s and did not
Therefore, these are always followed by the portray the actual suffering of the community
basic form of verb (1st form of verb). including the genocide, mass rapes and mass
―Notified‖ in (D) must be replaced with murders committed by Islamic radical groups
―notify‖ to make the sentence grammatically in his film.
correct. The correct option is A The 343. We use ―is‖ to show a state of being in a
trademark owners should be given the option sentence. For example, ―Ram is a boy‖ says
to register with an e-commerce platform so the state of Ram i.e. a boy. But, when we show
that wherever a trademark product is possession, we use ―has/have/had‖. For
uploaded for sale on the platform, it should example, ―I have a pen‖. With the same, the
notify the respective trademark owner. sentence states the state of Chennai to hold
339. No correction required. The correct the worst record of pollution. Therefore, ―is‖
option is E Unlike the problem of physical should be replaced with ―has‖ to make it
piracy and counterfeiting, the piracy of grammatically correct. The correct option is A
content is easy and damaging. 340. The Chennai has the second-worst record of
subject of the sentence ―BJP‖ is a singular vehicular pollution in India after Delhi,
noun and the possessive adjective of it should generating 3,200 tonnes of carbon dioxide
be singular and not plural. Therefore, ―their‖ (CO2) per day, a primary greenhouse gas,
in (A) must be replaced with ―its‖ The according to Centre for Science and
correct option is B With the ruling BJP issuing Environment (CSE).
a whip to its members in both houses of 344. The sentence is not showing the action in
Parliament, there is a huge buzz on social perfect continuous form (has + been + V-ing).
media that a bill on the controversial Uniform The sentence states a perfect sentence of
present of giving citizenship to Adnan Sami.
Use Code: (AVP10) for 10% India’s No.1 Teacher in Bank Exams for English Language and Editorial 92
additional discount on All Website: www.vishalparihar.com | Follow on Instagram: @vishalthetrainer
Unacademy courses
1000+ Error Detection for All Exams
Last 20 years (Previous Year Papers)
English By Vishal Parihar

Therefore, ―awarding‖ should be replaced Ashok Bhushan and A S Bopanna, said the
with ―awarded‖ to make it grammatically pendency of the appeal filed by the Centre and
correct. The correct option is D As the Centre the Delhi government before it would not be
announced that singer Adnan Sami, who was an impediment for the trial court in issuing
born in Pakistan but later took Indian fresh date for execution of the convicts. 349.
citizenship, has been awarded Padma Shri The sentence narrates about the impact of
this year, Union Aviation Minister Hardeep Corona Virus and the possessive adjective
Singh Puri said that he hoped protestors at used for it should be ―its‖ and not ―their‖
Shaheen Bagh were "listening". before ―impact‖ in (C). The correct option is
345. The subject of the sentence (a leader) is C Amid the lockdown and travel bans
a singular noun and therefore, must follow a triggered by the 2019 novel coronavirus
singular verb (or helping verb). ―Have‖ in (C) outbreak in China, fears have been expressed
must be replaced with ―has‖ to correct the about its impact on the health of the global
same. The correct option is A A leader from economy as a whole.
Pakistan Prime Minister Imran Khan's party, 350. Whenever a verb is written after ―was‖,
who put up banners with a slogan offensive to it is either written in ―V-ing‖ form to show a
the minority Hindus in the country, has continuous tense or in ―Verb-3 rd form‖ to
apologised after he came under fire from show a passive voice. This sentence needs a
netizens as well as the party. passive voice form. Therefore
346. The correct phrase used in English ―conceptualize‖ must be replaced with
language is ―in the wake of‖ and not ―in the ―conceptualized‖ to make the sentence
waked of‖ as written in the sentence, as grammatically correct. The correct option is A
―wake‖ here is used as a noun and not a verb It was conceptualized with an aim to identify
to be written in any other forms. The correct the most promising startups via a competitive
option is B Actor Margot Robbie's latest anti- format, reward them and create an assured
hero ensemble feature "Birds of Prey" has path for success by eliminating hurdles and
reportedly being marketed by Warner Bros challenges typically faced by most founders.
with a new title in the wake of dismal opening 351. Same reason as previous question
weekend score. ―Will‖ must be followed by ―amount‖ and
347. ―Despite‖ is never followed by ―of‖. not ―amounts‖. The correct option is D A
The correct option is A Despite the fires from single page in a normal text book will amount
the CAA and NRC are still burning across the to 4 pages in Braille script and the advanced
country, the BJP is choosing to become more machine is fed with Baden Software to print
combative by pushing through another the books.
contentious issue which is a sensitive issue 352. ―Apart‖ is followed by ―from‖ and not
for the minority community. ―of‖. The correct option is C The Press,
348. The sentence describes about the functioning under Directorate for the Welfare
comprising of a three-judge bench and is not of Disabled and Senior Citizens, also prints
an event that happens occasionally. and distributes Braille books on general
Therefore, ―comprises‖ which makes it a knowledge, music, Kannada and English
regular action is incorrect and should be grammar apart from a calendar.
replaced with ―comprising‖. The correct 353. Same reason. ―Will‖ must be followed
option is A A three-judge bench, headed by by ―exceed‖ and not ―exceeding‖. The
Justice R Banumathi, comprising justices correct option is B On the overall coal
Use Code: (AVP10) for 10% India’s No.1 Teacher in Bank Exams for English Language and Editorial 93
additional discount on All Website: www.vishalparihar.com | Follow on Instagram: @vishalthetrainer
Unacademy courses
1000+ Error Detection for All Exams
Last 20 years (Previous Year Papers)
English By Vishal Parihar

scenario in the country, the official said the choosing to separate educational quotas from
production this year will exceed that of last jobs, where efficiencies depend on merit and
year, though there was some impact due to performance.
rains, by the end of the current fiscal it will be 358. ―Had‖ in the sentence followed by verb
covered up. ―reject‖ shows that this is a sentence of past
354. The sentence shows that a pledge was perfect tense. ―Rejects‖ in the sentence must
taken by Uddhav Thackeray. Pledge is not an be changed to ―rejected‖ to make it
action verb that continuous for long. grammatically correct/ The correct option is
Therefore, it can‘t be written in the form of D A Mumbai court had on February 5 rejected
―perfect continuous tense‖ i.e. ―has been the anticipatory bail application filed by
pledging‖. Verbs like this are always written Chudawala, who was recently booked in a
in perfect tense. Therefore, ―pledging‖ must sedition case for allegedly raising "anti-
be replaced with ―pledged‖ to make the national slogans" in support of JNU student
sentence grammatically correct. The correct Sharjeel Imam.
option is A Chief Minister Uddhav Thackeray 359. No correction required. The correct
has pledged to the public that the State option is E The 24-year-old woman lecturer
government will prove the crime of the succumbed to injuries a week after she was
accused as soon as possible and the accused set ablaze by a jilted lover near the
will be hanged soon. Hinganghat area in Maharashtra's Wardha
355. The sentence tells an incident that district, on February 3.
happened at sometime is past, i.e. a demand 360. ―Pour it‖ is a phrase that takes the
made by opposition. This requires a simple preposition ―out‖ and not ―to‖. The correct
present tense. So, ―demanding‖ should be option is B Once Hanumanji has decided that
replaced with ―demanded‖ to make the the dal fry, which was destined for you is
sentence grammatically correct. The correct ready, he will pour it out onto a small stainless
option is A Understandably this has stirred up steel bowl – filled to the brim and a bit more.
a hornet‘s nest with the issue being raised in 361. Replace ―from‖ with ―on‖. The correct
the Parliament and the Opposition Congress option is B Both DDT and new income tax
demanding that the Union Government either proposals will not have any adverse impact
file a review petition in the apex court or on the mutual fund industry
amend the Constitution to make reservation a 362. The subject of the sentence is ―she‖ and
fundamental right. the possessive pronoun for it is ―her‖ and not
356. The sentence is of a perfect tense to show ―me‖ which is the possessive pronoun of
something to have happened. To write this, ―I‖. The correct option is B Being a voracious
we need ―been‖ after ―have‖. The correct reader, she sat down for studying whenever
option is D Plus, this is a constitutional matter she got time but it was her family which
that should have been heard by the full helped her stay focused as they went out of
Constitution Bench and not just a twojudge their way to make her comfortable.
Bench of the apex court. 363. The sentence is of present sentence of an
357. Same reason ―Separate‖ to be written urgent need for something before anything
in place of ―separated‖ after ―to‖. The happens in future. ―Became‖ which shows a
correct option is B While the quotas have past tense must be replaced with ―becomes‖
been politicized over the years, the apex court to make it grammatically correct for present
has merely tried to make them practical, tense. The correct option is B There is an
Use Code: (AVP10) for 10% India’s No.1 Teacher in Bank Exams for English Language and Editorial 94
additional discount on All Website: www.vishalparihar.com | Follow on Instagram: @vishalthetrainer
Unacademy courses
1000+ Error Detection for All Exams
Last 20 years (Previous Year Papers)
English By Vishal Parihar

urgent need to galvanise the concerned 369. The sentence shows a fact of present for
departments into action before the price for the role of media. Therefore, no verb in the
inaction becomes too much to bear and takes same sentence highlighting the fact can be
the form of a public health emergency. written in past tense. Therefore, ―enabled‖
364. The sentence shows restoring faith for must be replaced with ―enables‖ to make it
people and to help them. Therefore, ―it‖ after grammatically correct. The correct option is B
help in (C) must be changed to ―them‖. The In any election campaign, the media plays an
correct option is C The ―Health at your important role in dissemination of
Doorstep‖ initiative of the Government will information and, thus, enables the people to
go a long way in restoring faith of the people make a well-informed choice.
and help them understand the importance of 370. No correction required. The correct
health parameters. option is E While we do not have concrete
365. ―A corporation‖ is a singular subject data on complications and deaths caused by
and therefore, must be followed by a singular malfunctioning equipment, it is not hard to
verb. ―Know‖ must be changed to ―knows‖ imagine a scenario where treatments are
to make the sentence grammatically correct. prescribed based on faulty readings, leading
The correct option is D Once a corporation to catastrophic consequences.
knows its value — which can be obtained 371. ―Government‖ is a singular subject and
through a reputed valuation service — it can the verb after it must be singular to make it
negotiate with the buyer and can reject the grammatically correct. Therefore, ―have‖
deal, if the offer made is less than what is must be replaced with ―has‖. The correct
required. option is D At present, the country imports
366. No correction required. The correct over 80 per cent of its medical devices and the
option is E. Many corporations measure Government has made its intention clear to
economic value creation for the company as a make India one of the global hubs for medical
whole, periodically, using different devices, both in manufacturing and
techniques to measure performance against distribution/
planned targets and suitably reward 372. The sentence describes about the
managers on achieving the same. probable deals after the visit of Donald Trump
367. ―Between‖ is always followed by ―and‖ to India and not from India. The correct
to connect two persons or things. The correct option is A Ahead of US President Donald
option is B India has asked Vietnam to open Trump‘s maiden visit to India, the two
its pharmaceutical market for the Indian countries are eyeing to finalise a raft of mega
generic drugs and sought better market defence deals including procurement of a
access for several other products during the batch of military helicopters by Indian Navy
delegation level talks between India and from American defence major Lockheed
Vietnam. Martin at a cost of USD 2.6 billion.
368. Justice is sought ―for an appeal‖ and not 373. ―Weather its‖ are mistyped words in
―of an appeal‖. The correct option is D Liquor place of ―whether it‘s‖. The correct option is
baron Vijay Mallya arrived at the Royal Courts B The requirement comes as governments
of Justice in London for his appeal against around the world are trying to hold social
being extradited to India to face fraud and media companies more accountable for the
money laundering charges amounting to Rs content that circulates on their platforms,
9,000 crores.
Use Code: (AVP10) for 10% India’s No.1 Teacher in Bank Exams for English Language and Editorial 95
additional discount on All Website: www.vishalparihar.com | Follow on Instagram: @vishalthetrainer
Unacademy courses
1000+ Error Detection for All Exams
Last 20 years (Previous Year Papers)
English By Vishal Parihar

whether it‘s fake news, child porn, racist 379. 3; Replace ‘ benchmarking ’ with
invective or terrorism-related content. ‘benchmark’
374. No correction required. The correct 380. 1; Replace ‘ taking’ with ‘taken ’
option is E Market benchmark Sensex rallied 381. 3; Replace ‘ constantly’ with ‘constant’
over 300 points in opening session on 382. 3; Replace ‘ than’ with ‘ from’
Wednesday driven by gains in HDFC twins, 383. 2; Replace ‘ creating’ with ‘ create’
RIL, ICICI Bank and HUL ahead of the release 384. 4; Replace ‘ rating’ with ‘ rate’
of inflation and factory output data. 385. 5
375. Same reason as stated for Question no. 2. 386. 1; Replace ‘ witnessed’ with ‘ having
―conferring‖ must be replaced with witnessed’
―confer‖ to make it grammatically correct. 387. 4; Replace ‘ causing’ with ‘ cause’
The correct option is A The BJP on Monday 388. 2; Infinitive without ‘ to’ follows
staunchly defended its decision to confer wouldn’t/shouldn’t
Padma award to singer Adnan Sami by 389. 1; Replace ‘ bring’ with ‘ bringing’
claiming that Sonia Gandhi too should be 390. 5; No error
subject of discussion if everyone has to be 391. 4; Replace ‘ on’ with ‘ for’
judged by his or her parents‘ deeds. 392. 5; No error
376. ―Be‖ is always followed by the 3rd form 393. 2; Replace ‘ on’ with ‘ at’
of verb when followed by a verb. ―Describe‖ 394. 2; Replace ‘on’ with ‘in’
must be replaced with ―described‖. The 395. 3; Replace ‘abide’ with ‘abidin’ and
correct option is C Prime Minister Narendra ‘subject’ with ‘subjected’
Modi hit out at the Congress in Lok Sabha over 396. 4; Replace ‘have’ with ‘has’
its opposition to the CAA and asked if first 397. 5
premier Jawaharlal Nehru could be described 398. 4; Replace ‘to’ with ‘in’
as "communal" for seeking citizenship for 399. 4; Replace ‘its’ with ‘theirs’
religious minorities from Pakistan. 400. 3; Replace ‘require’ with ‘requires’
377. The sentence is in passive form. 401. 3; Replace ‘on’ with ‘in’
Therefore, ―took‖ must be replaced with 402. 4; Replace ‘supervise’ with ‘supervising’
―taken‖ to make it grammatically correct. 403. 5
The correct option is D The verdict of putting 404. 3; Place the word ‘file’ before ‘electronic’
Hafiz Saeed in jail for 7 years came four days 405. 4; Replace the word ‘to’ with ‘on’
before the Financial Action Task Force‘s 406. 1; Omit the word ‘of’
(FATF) plenary meeting in Paris to assess 407. 4; Replace ‘been’ with ‘being’
action taken by Pakistan to counter terror 408. 3; Put ‘a’ before ‘discreet’
financing and money laundering. 409. 2; Replace the word ‘mid’ with ‘amid’
378. ―Pulled out‖ must be followed by 410. 5
―from‖ instead of ―in‖ to mean withdraw 411. 5
from some place. The correct option is B The 412. 3; The correct word is ‘unproductive’
Hummer, which was first marketed in 1992 413. 1; Use the word ‘as’ before ‘the’
and then pulled out from the market in 2008 414. 4; Replace “ next” with “near”
following questions over its viability after the 415. 2; Replace “have been planting” with
economic slowdown, is making a comeback, “have planted”
albeit with some 'thunderous' modifications. 416. 5
417. 4; Replace “through” with “in”
Use Code: (AVP10) for 10% India’s No.1 Teacher in Bank Exams for English Language and Editorial 96
additional discount on All Website: www.vishalparihar.com | Follow on Instagram: @vishalthetrainer
Unacademy courses
1000+ Error Detection for All Exams
Last 20 years (Previous Year Papers)
English By Vishal Parihar

418. 3; Replace “things ” with “thing ” 476. 5


419. 3; Put the word “ were” before “being ” 477. 1; “ The commenrce ministry has not
420. 5 only fxed ”
421. 2; Replace “between ” with “ through” 478. 4; “ severely overweight”
422. 5 479. 5; No error
423. 4; Replace “ began” with “begun ” 480. Interestingly, this sentence has three
424. 2; Replace “worse ” with “ worst” errors; Replace ‘avoid’ with ‘avoiding in (1)
425. 1; Replace “is ” with “ has” ‘on’ with ‘in’ in (3); and ‘are’ with ‘is’ in (4)
426. 4; Replace “ for” with “ in” 481. 4; Substitute “ at ” with “ an”
427. 1; Put the word “soon” before “make ” 482. 1; Substitute “ for ” with “ of ”
450. 3; Replace “ on” with “in ” 483. 5
451. 4; Replace “ their ” with “ its ” as it is used 484. 3; Substitute “ is ” with “through”
for ‘ airline ’, which is singular 452. 1; Replace 485. 3; Substitute “ from ” with “ through ”
“ began ” with “ begun ” as the 3rd form of the 486. 1; Replace “ on ” with “ in ”
verb is used in the present perfect Tense 487. 3; Replace “ screen ” with “screened ”
453. 3; Replace “ Confident ” with “ 488. 2; Replace “ differing ” with “ different ”
Confidence ” 489. 5
454. 1; Replace “ Inspite ” with “ Despite the 490. 1; Replace “ which ” with “ who ”
fact ” 491. 2; Insert “ is ” before “ alleged “
455. 4; Replace “ invested ” with “ investing ” 492. 4; “ undergo “
456. 5 493. 2” forced “
457. 2; Replace with “likely to cause of 494. 1; Replace “ this ” with “ these ”
inflation ” 495. 5
458. 3; Replace “ have ” with “ has ” as it is 496. 3; Replace “enjoyed ” with “ enjoy ”
used for ‘ nationalization of banks ’,which is 497. 1; Replace “ on ” with “ in ”
singular 498. 4; Replace “ bringing ” with “ bring ”
459. 2; Replace “ currently ” with “current ” 499. 4; Replace “ for ” with “ to ”
460. 5 500. 3; “ have been ”
461. 3; Replace “ was go ” with “ may go ” 601. 4; should be “who”
462. 1; Replace “ angry(adj)” with “anger(n) ” 602. 2; Replace “ necessary ” by “necessarily ”
463. 3; Replace “ were ” with “ had ” 603. 4; Replace “ was ” by “ were ”
464. 5 604. 1; Replace “ begin ” by “ begun ”
465. 2; Replace “ ever – grow ” with “ever – 605. 2; Replace “ to ” by “ too ”
growing ” 606. 2; Replace “ lowest ” by “ lower ”
466. 4; Replace “ probable ” with “ probably ” 607. 3; Replace “ with ” by “ among ”
467. 4; Replace “ above ” with “ about ” 608. 4; Replace “ since ” by “ from ”
468. 5 609. 3. Replace “ rate for ” by “ the rate of ”
469. 1; Replace “ easy “ (adj) with “ eased ” (v) 610. 2; Replace “ lying ” by “ underlying ”
470. 2; Replace “ linkage” with “ linked ” 611. 4; Replace “ among ” by “ along ”
471. 4; Replace “ tackle ” with “tackling ” 612. 1; Substitute extensive
472. 3; “ instance of illegal ” 613. 2; Substitute government - sponsored
473. 2; Replace “ was ” with “ were ” 614. 4; Substitute of
474. 3; Replace “ youths ” with “youth ” 615. 2; Substitute indication
475. 5 616. 4; Substitute because for why
Use Code: (AVP10) for 10% India’s No.1 Teacher in Bank Exams for English Language and Editorial 97
additional discount on All Website: www.vishalparihar.com | Follow on Instagram: @vishalthetrainer
Unacademy courses
1000+ Error Detection for All Exams
Last 20 years (Previous Year Papers)
English By Vishal Parihar

617. 1; Substitute will have 661). 1; Substitute to


618. 2; Substitute has been promoted 662). 2; Substitute drawing
619. 2; Substitute many for much 663). 2; Substitute sticking
620. 1; Substitute to for into 664). 5;
621. 3; Substitute for selling 665). 4; Substitute to
622). 1; Substitute to start 666). 3; Substitute about
623). 3; Substitute their 667). 1; Substitute to for on
624). 4; Substitute on 668). 3; Substitute have for had
625). 5; 669). 5;
626). 2; Delete of 670). 4; Substitute ways
627). 3; Substitute need 671). 3; Substitute to define
628). 4; Substitute to be taken 672). 3; Substitute when for as
629). 2; Substitute finalizing 673). 4; Substitute would grow
630). 2; Substitute million 674). 4; Substitute asked
631). 3; Substitute for 675). 4; Substitute surface
632). 2; Substitute leading manufacturer 676). 1; Substitute in for On
633). 1; Substitute of 677: 2; Substitute comply
634). 3; Substitute persons 678: 4; Substitute between
635). 5; 679: 1; Substitute damaged
636). 5; 680: 2; Delete of the
637). 4; Substitute reached 681: 3; Substitute talking
638). 2; Delete has 682: 4; Substitute end
639). 3; Substitute 683: 5
640). 3; Substitute regulate 684: 3; Substitute its
641). 2; Delete most 685: 1; Substitute has
642). 5; 686: 2; Substitute similar
643). 5; 687. 5
644). 4; Substitute which 688. 3 ; Substitute ones for talks
645). 2; Substitute consider 689. 1 ; Substitute of
646). 2; substitute consideration 690. 2 ; Substitute who
647). 3; Substitute for 691. 3 ; Delete do not
648). 2; Substitute produced 692. 3 ; Substitute is
649). 5; 693. 4 ; Substitute one another
650). 2; Substitute are for will be 694. 4 ; Substitute favour
651). 4; Substitute profitable 695. 3 ; Substitute when
652). 1; Substitute when for which 696. 1 : Substitute was
653). 1; Substitute on 697. 4 ; Substitute fluent
654). 3; Substitute consumption 698. 1 : Substitute was attempting
655). 3; Substitute is considering 699. 5
656). 3; Substitute headed 700. 5
657). 1; Substitute do 701. 4; inexhaustible
658). 4; Substitute adopting 702. 1; students
659). 3; Substitute announced 703. 2; the
660). 5; 704. 3; who
Use Code: (AVP10) for 10% India’s No.1 Teacher in Bank Exams for English Language and Editorial 98
additional discount on All Website: www.vishalparihar.com | Follow on Instagram: @vishalthetrainer
Unacademy courses
1000+ Error Detection for All Exams
Last 20 years (Previous Year Papers)
English By Vishal Parihar

705. 4; Substitute “weakness the power..” 748. 5; 249. 3; Substitute make


706. 3; Delete most 750. 5; Substitute help
707. 3; Substitute managed 751. 3; Substitute was
708. 4; Substitute remains 752. 5;
709. 2; Substitute lower 753. 3; Substitute were
710. 3; Substitute “among the voters’ 754. 1; Substitute A certain amount
711. 4; Substitute “from tomorrow” 755). 3
712. 3; Substitute “the rate of “ 756). 4
713. 1; Substitute a for the 757). 2
714. 4 758). 3
715. 3; Substitute “was to go” 759). 4
716. 1; Substitute anger 760). 5
717. 3; Substitute had for were 761). 4
718. 5; 762).1; Replace ‘scheduled’ with ‘organizing’
719. 2; Substitute ever – growing or ‘holding’.
720. 4; Substitute probably. 763). 4; Change ‘heavily’ to ‘heavy’
721. 4; Substitute regarding for above 764). 5
722. 5; 765). 2; Delete ‘in’ after ‘invest’
723. 1; Substitute eased 766). 3; Change ‘dependence’ to ‘dependent’
724. 2; Substitute linked 767). 1; Change ‘taken’ to ‘was taking’
725. 4; Substitute was 768). 2; Change ‘to spend none’ to ‘spending
726. 1; Substitute when diving into deep sea some’.
727. 5; 769). 4; Replace ‘of’ with ‘to’.
728. 1; Delete “The reason behind” 770). 2; Change ‘waiting for’ to ‘waiting’.
729. 1; Substitute had 771). 1; Change ‘when’ to ‘whether’
730. 1; Substitute are 772). 1; ‘Our customers are always visit’
731. 3; Substitute had should be replaced with ‘our customers
732. 2; Substitute is. always visit’.
733. 1; Substitute had been 773). 5
734. 4; Delete other 774). 2; ‘arrive’ should be replaced with
735. 1; Substitute begun ‘arrived’.
736. 1; Substitute “Inspite of the 775). 1; ‘A habit of’ or ‘The habit of’ is the
organizations being aware” correct usage.
737. 1; Insert the before interlinking 776). 5
738. 4; Substitute investing 777). 4; ‘it lacks resources’ should be replaced
739. 4; Substitute its with ‘they lack resources’.
740. 5; 778). 3; ‘quickly should be replaced with
741. 2; Substitute the current. ‘quick’.
742. 3; Insert the before headquarters 779). 2; ‘has lend’ should be replaced with
743. 2; Substitute to cause for the cause of ‘has lent’.
744. 3; Substitute has 780). 4; ‘our self’ should be replaced with
745. 3; Substitute forgotten ‘ourselves’.
746. 4; Substitute rethinking 781). 3; ‘have’ should be deleted.
747. 2; Substitute is 782). 2
Use Code: (AVP10) for 10% India’s No.1 Teacher in Bank Exams for English Language and Editorial 99
additional discount on All Website: www.vishalparihar.com | Follow on Instagram: @vishalthetrainer
Unacademy courses
1000+ Error Detection for All Exams
Last 20 years (Previous Year Papers)
English By Vishal Parihar

783). 4 scarcely…when./ As soon as , Here in the part


784).5 (2) ‘when’ should be replaced with than.
785).3 Examples –
786).1 No sooner had I entered the room than
787). 2; Replace ‘in’ with ‘on’
788). 3; Replace ‘abide’ with ‘abiding’ and Sheetal called.
‘subject’ with ‘subjected’ Hardly had he reached the station when the
789). 4; Replace ‘have’ with ‘has’
790). 5; train came.
791). 4; Replace ‘to’ with ‘in’ As soon as I arrived at the bus stop, she came.
792). 2; It should be “equipped not only with”
instead of “not only equipped with”.
813. Solution: 1
793). 1; Here, as we are comparing two
Summons is a noun while summon is a verb.
methods for a single purpose, the sentence
Summons (n) – A notice summoning someone
should start as – ‘No other method’.
to appear in court.
794). 3; Views should always be followed by
singular – summons , plural- summonses .
‘on’ instead of ‘for’.
Summon (v) – to ask someone to
795). 3; Delete ‘I’.
come/attend.
796). 4; It should be ‘has been’ instead of
Turn up – make an appearance, appear, be
‘have been’.
present.
797). 1; Replace ‘a’ with ‘the’.
Use plural form summonses instead of
798). 5
summons.
799). 3; Replace ‘with’ with ‘about’
800). 5
814. Solution: 3
801). 2; Change the order from ‘for holding
Replace ‘is it’ with ‘it is’ as the given sentence
not only’ to ‘for not only holding’ 802). 4;
is not a question .
Replace ‘since’ with ‘for’
803). 2; The sentence is in past tense, so it
815. Solution: 2
should be ‘was’ in place of ‘is’.
Replace ‘amusing’ with ‘amused’. Amused
804). 4; It should be “….how one ‘looks’ at it”.
at/by something- thinking that someone or
805). 3; It should be ‘are’ in place of ‘is’.
something is interesting, so that you smile or
806). 3; The work has been going for several
laugh.
years continuously, so it should be ‘have been’
in Plac Example- We were amused at her jokes.
811 Solution: 1
Replace ‘quarter results’ with ‘quarterly
results’.
‘Quarter’ is a noun and ‘quarterly’ is 816. Solution: 2
adjective/ adverb. The noun ‘results’ should Replace ‘on’ with ‘up’.
be preceded by adjective ‘quarterly’. Pick on- to harass or bother.
812. Solution: 2 Pick up- to grasp something (as with one’s
If the second event occurs immediately after hands).
the first, we can express that idea using the
structure no sooner … than. / hardly or
Use Code: (AVP10) for 10% India’s No.1 Teacher in Bank Exams for English Language and Editorial 100
additional discount on All Website: www.vishalparihar.com | Follow on Instagram: @vishalthetrainer
Unacademy courses
1000+ Error Detection for All Exams
Last 20 years (Previous Year Papers)
English By Vishal Parihar

817. Solution: 3 to suggest that one thing happened very soon


Preposition ‘to’ is superfluous in the part (3). after another.
818. Solution: 2 Structure: – “hardly ……. when / before.”
Replace ‘for’ with ‘against’ “Scarcely ……. when / before.”
Discriminate against something “no sooner ……. Than”
819. Solution: 1 Example- Sheetal had no sooner seen one
Replace ‘as to’ with ‘due to’. ”As to” – about, picture than she saw another.
regarding, concerning…ex- There is some 830. Solution: 3
doubt as to whether the information is totally “On call” – If someone is on call, they are ready
accurate. to go to work at any time if they are needed,
“Due to” – because of something. especially if there is an emergency. Replace
Example- The Company’s financial losses ‘by’ with ‘on’
were due to weak management. 831. Solution: 1
820. Solution: 2 Replace ‘my’ with ‘our’
“difficult” should be followed by infinitive 832. Solution: 1
with ‘to’. Replace “little” with “a little”
replace ‘decided’ with ‘to decide’ 833. Solution: 2
821. Solution: 2 denounce – criticise, condemn. Oppression-
Replace ‘on’ with ‘in’ harassment, coercion.
822. Solution: 1 Replace ‘justice’ with ‘injustice’
Replace ‘seen’ with ‘see’ (present indefinite 834. Solution: 1
tense) Replace ‘a wiser’ with ‘as wise’ (competent-
823. Solution: 3 capable, सक्षम)
Replace “for drive” with “for driving/to 835. Solution:4
drive”. Generally preposition should be Next to- beside, by, near, close to .
followed by V4 form and when ‘to’ is the part Example- Sheetal sat down next to him on the
of infinitive, it should be followed by the base sofa.
form of the main verb. 836. Solution: 2
824. Solution: 3 This is a conditional sentence – use ‘were’
Use definite article ‘the’ before the name of instead of ‘was’ example- Sheetal always
ocean (the Atlantic Ocean) treats him as if he were a child.
825. Solution: 1 replace ‘was’ with ‘were’.
Replace ‘had been gained’ with ‘has gained’, 837. Solution: 1
as the given sentence is in active voice. Active The English represents ‘The people of
voice structure: “has + V3” or “has+ been + V- England not
ing”. Passive voice structure: “has + been + English language’ . Replace ‘the English’
V3”. with ‘English’.
826. Solution: 3 838. Solution: 1
Replace ‘his’ with possessive case of ‘one’ – ‘full of something ’ – containing a lot of
“one’s” something.
Example- One must take care of one’s health. Example- The room was full of girls.
827. Solution:1 The team members are full of energy.
Expressions “hardly, scarcely and no sooner” Replace ‘with’ with ‘of’.
– can be used (often with a past perfect tense) 839. Solution: 2
Use Code: (AVP10) for 10% India’s No.1 Teacher in Bank Exams for English Language and Editorial 101
additional discount on All Website: www.vishalparihar.com | Follow on Instagram: @vishalthetrainer
Unacademy courses
1000+ Error Detection for All Exams
Last 20 years (Previous Year Papers)
English By Vishal Parihar

This is The type 3 conditional which is used to Here the doer is – we and the action is – saw.
refer to an unreal past condition and its The correct form should be – we saw not only
probable past result. Structure:- the president (noun) + but also the chief
If + subordinate clause in past perfect (Or had minister (noun)……
+ sub. + past participle.)+ principle clause – Example- Rohit is not only foolish but also
would / could / should + have + past mad. (adjectives)
participle. He was not only beaten but also fined. (verbs)
should have – to say that a different action 835. Solution: 3
was recommended in the past. could have – to Explanation :- use plural form ‘pairs’ instead
talk about possibilities if something had been of singular form ‘pair’
different in the past.
would have – to imagine a result (if something 836. Solution: 3
had been different in the past) when two subjects are joined by not
replace ‘should’ with ‘would’ only………but also / either…..or /
840. Solution: 1 neither…..nor. – The verb should be used
The type 1 conditional refers to a possible according to the nearest subject.
condition and its probable result. In these Example- Neither Rohit nor his friends are
sentences – the ‘if clause is in the simple capable of doing this work within three days.
present, and the main clause is in the simple Use singular verb ‘finds’ instead of plural verb
future’. Structure:- If … Simple Present …, … ‘find’.
Simple Future.
Omit ‘will’ in the part (1) 837. Solution: 4
841. Solution: 3 no error
Expressions- coupled with, as well as, along
with, together with, with etc., when we use
these expressions to join one singular subject 838. Solution: 2
of a sentence with another noun or pronoun, No sooner … than, hardly/scarcely…when.
consider it as a singular subject . Example- Hardly had I arrived home when my
Example- Mr. Puneet along with his wife phone rang. Use ‘when’ instead of ‘then’.
wants to purchase the beach house. Replace 839. Solution: 3
‘were’ with ‘was’ in the part (3) subject ‘demand’ is singular – use singular
842. Solution: 3 verb ‘is’ instead of ‘are’
In the part (C) remove definite article ‘the’ 840. Solution: 4
Example- They are going to visit the No error.
construction work in June next year. We may use ‘a + countable noun + of +
843. Solution: 4 uncountable noun.
“in” – for months, years, centuries and long Example – A drop of honey/water.
periods 840. Solution: 2
“On” for days and dates ( example – “in Replace ‘master’ with ‘master’s’ .( to do a
1947”, ‘in August’, “on monday”, “on 12th master’s degree )
May, 2000′) 841. Solution: 2
834. Solution: 1 when two nouns “possess” the same entity,
“not only…….but also” – both the parts should only the second takes an apostrophe ( ’ ):
be followed by same part of speech.
Use Code: (AVP10) for 10% India’s No.1 Teacher in Bank Exams for English Language and Editorial 102
additional discount on All Website: www.vishalparihar.com | Follow on Instagram: @vishalthetrainer
Unacademy courses
1000+ Error Detection for All Exams
Last 20 years (Previous Year Papers)
English By Vishal Parihar

Example – Mr. Sharma and his son’s shop. Use 998. Solution: 1
Rajesh instead of Rajesh’s. Sentence is based on the structure –If
842. Solution: 3 not……then means अगर नह ीं तो
Review (verb) – assess (something) formally Part (1) should be If not too long then
with the intention of instituting change if 999. Solution: 1
necessary. example- it’s a good idea to review Unless means if this condition is not met
your class notes. review (noun) – a formal (Unless you work hard , you won’t pass. ),
assessment of something with the intention of whereas until means before or up to the time
instituting change if necessary. (I lived with my parents until I was 21.)
Example – an annual review of company Do not use negative word in the clause
performance. starting with conjunctions until and unless.
843. Solution: 3 The correct usage should be unless you obey
“accessible to” – easy to get , use or in part (1).
understand. 1000. Solution: 2
Example- This television programme aims to If the second event occurs immediately after
make history more accessible to children. the first, we can express that idea using the
Replace ‘with’ with ‘to’ structure – hardly or scarcely…when.
844. Solution: 2 replace where with when.
For two – each other Example- Hardly had Himesh left when his
And for more than two – one another father came.
Here in this sentence the subject is “The two e of ‘are’
men” reciprocal pronoun one another should 1001).1
be replaced by each other

845. Solution: 1
This / that kind of something – for singular ;
these / those kinds of something – for plural
Example – I have given up those kinds of
ideas. (plural) or I have given up that kind of
idea. (singular)
The correct subject should be These kinds of
clothes

FREE YouTube Classes daily on Bankersway Youtube


996. Solution: 1
Channel
climb (noun) – the act or process of moving
upward ➢ The Hindu Editorial Analysis Daily at 7 AM
Example – Himesh made a rapid climb to the ➢ English Class Daily at 9 AM
zenith of success.
Replace upside with to.

997. Solution: 4
no error

Use Code: (AVP10) for 10% India’s No.1 Teacher in Bank Exams for English Language and Editorial 103
additional discount on All Website: www.vishalparihar.com | Follow on Instagram: @vishalthetrainer
Unacademy courses

You might also like